Sei sulla pagina 1di 119

IES VIRGEN DEL CARMEN (JAN)

DEPARTAMENTO DE LENGUA CASTELLANA Y LITERATURA

CURSO ACADMICO 2009-2010

CUADERNO DE ACTIVIDADES 1 BACHILLERATO

NOMBRE Y APELLIDOS: _____________________________________________________ CURSO Y GRUPO_____________

El siguiente Cuaderno de actividades est organizado de acuerdo con el orden de los temas del libro de texto. De esta manera, las actividades se distribuyen en dos grandes bloques: 1) La variedad de los discursos, el tratamiento de la informacin y el conocimiento de la lengua. Se corresponde con las actividades pertenecientes a las lecciones 1 a 10 2) El discurso literario Se corresponde con el resto de actividades y que pertenecen a las lecciones 11 a 20. Dependiendo de los objetivos de la asignatura de Lengua castellana y Literatura que no hayas alcanzado, debers trabajar unas actividades u otras. En la ficha de Lengua, que se te ha dado junto al boletn de calificaciones, se detalla el bloque o los bloques de contenido que has de trabajar durante el verano y de los que se te preguntarn en el examen extraordinario del mes de septiembre. Trabaja las actividades con la ayuda del libro de texto, ya que en el examen de septiembre habr ejercicios similares a los que en el cuaderno se incluyen. Junto a estas actividades, cada una de las lecciones del libro de texto incluye en su ltima pgina unas actividades de evaluacin. Resultar muy conveniente que lo trabajes ya que suponen una tarea de sntesis muy aprovechable de los contenidos bsicos de cada leccin.

EL DEPARTAMENTO DE LENGUA CASTELLANA Y LITERATURA

ACTIVIDADES

REFUERZO

1. La comunicacin. Funciones del lenguaje


Los marcos de referencia permiten establecer distintas situaciones de comunicacin y el papel que desempean en ella los participantes. En el caso de los textos literarios, es posible reconocer dos procesos comunicativos: uno externo, en el que participamos como lectores, y otro interno, que se establece en la ficcin creada en ellos. Adems, en estos textos pueden manifestarse las distintas funciones que cumple el lenguaje.

Arxemiro
En ese instante, despert Alguien me hablaba. Me pareci que despertaba de otro mundo. Delante de m haba un fantasma, un fantasma que haba visto muchas veces. Era Arxemiro, que tena el candil encendido en la mano. Le pregunt: Qu te pasa? Te ha venido a visitar el enemigo? Has visto a la Santa Compaa? Por qu ests tan plido? Peor que eso me contest. Me vi muerto. No volvera a mi cuarto por nada del mundo. La voz de Arxemiro pareca que vena de muy lejos. Yo no saba qu hacer. Me estallaba la cabeza. Por fin, le dije: Era el vino, hombre, era el vino. Vulvete a acostar. l me contest: No, Bieito. Era la muerte! Escucha y despus, juzga. Cuando llegu a mi cuarto, encend el candil para acostarme. Me fui desnudando poco a poco, dejando la ropa en el suelo. Ya me haba quitado la guerrera y la camisa. Todava la tena en la mano. No s cmo mir para la cama. All haba un hombre durmiendo con la cabeza tapada por la manta. Qu hara yo? Le quit la ropa A Arxemiro le castaeaban los dientes como a un hombre desnudo una maana de helada. Yo me sent en la cama. Su voz ya no era que viniese de lejos: era la voz de otro. Hablaba tartamudeando y me dijo: Era yo, que estaba muerto! Tena que quitarle aquel miedo, fuera como fuera. Ahora mismo le dije vamos all.
nxel FOLE en De cmo me encontr con el demonio en Vigo, Trama

MATERIAL FOTOCOPIABLE / Oxford University Press Espaa, S. A.

1 A qu tipo de texto pertenece el fragmento segn su intencin comunicativa? 2 Reconoce los elementos de la comunicacin que se observan en el texto. 3 Razona por qu hay intencin comunicativa en el dilogo. 4 En la segunda y en la tercera pregunta que formula el primer interlocutor, se menciona al enemigo y a la Santa Compaa. Qu tipo de contexto aclara las interrogaciones? 5 Arxemiro menciona la guerrera. Explica por qu el significado de esta palabra lo aclara el contexto lingstico y brinda un ejemplo en el que dicho trmino adquiera otro significado.

6 Qu intencin tiene el interlocutor de Arxemiro al decirle Era el vino, hombre, era el vino? 7 Has analizado el proceso comunicativo interno, el que se establece en el texto. Explica ahora los elementos del proceso comunicativo externo. 8 Indica, con ejemplos, las funciones del lenguaje que se pueden observar en el fragmento. 9 Seala algunos rasgos lingsticos caractersticos presentes en cada una de ellas. 10 Si no has encontrado alguna de las funciones del lenguaje, escribe enunciados que las ejemplifiquen.

1.

La comunicacin

23

ACTIVIDADES

REFUERZO

2. Tipos de signos
Recuerda que signo es aquello que est en lugar de otra cosa y que los signos pueden clasificarse en funcin de diferentes criterios: fuente de emisin, intencionalidad y conciencia del emisor, aparato receptor y vnculo con el referente (clasificacin de Peirce).

1 Indica de qu tipo son los signos representados en las imgenes en funcin de los criterios que figuran en la pgina 13 de tu libro de texto. 2 Seala, en todos los casos, la realidad a la que sustituyen dichos signos. 3 Considerando que esos signos se han representado por medio de dibujos, qu relacin guardan las imgenes con la realidad que pretenden representar?

4 A qu tipo de signo corresponde esa relacin? Recuerda los criterios de clasificacin de la pgina 13 de tu libro. 5 Cita otros signos que correspondan a cada una de las tipologas representadas en las imgenes. 6 Qu tipos de signos de los que figuran en el cuadro de la pgina 13 no se incluyen en los dibujos? Cita ejemplos de cada uno de ellos.

24 1.

La comunicacin

MATERIAL FOTOCOPIABLE / Oxford University Press Espaa, S. A.

ACTIVIDADES

REFUERZO

3. Los cdigos no verbales


Las siguientes imgenes y el fragmento de Adelaida Garca Morales registran diversos signos pertenecientes a los cdigos paralingstico, cinsico y proxmico, que constituyen sistemas de comunicacin que funcionan junto con el lenguaje verbal.

La ausencia de Bene
Qu pasa? dijo con sobresalto al escuchar mi voz. Tengo miedo le respond, deseando que recordara un tiempo ya pasado en el que yo le despertaba por las noches con esa misma frase. Pero esta vez me respondi fastidiado: Todava tienes miedo? Con lo mayor que eres! Estoy asustada por Bene. Me parece que le est pasando algo en este momento dije, tratando de justificarme y segura de que aquellas palabras le despertaran de una vez. Qu dices?! me contest irritado, pero mostrando al mismo tiempo una gran preocupacin. Bene no est en su habitacin dije lentamente, como si le notificara algo muy grave. Qu tontera! me respondi. Estar en el cuarto de bao. No, no est all, ni tampoco en el jardn. La he buscado por todas partes, tambin en la torre. No est en ningn sitio.
MATERIAL FOTOCOPIABLE / Oxford University Press Espaa, S. A.

Y a ti qu te importa dnde est Bene? me dijo malhumorado, y despus aadi: Vete ya a dormir y deja de espiarla o te llevars un susto. Por qu? Por nada, nia. Pareces tonta.
Adelaida GARCA MORALES El sur & Bene, Anagrama

1 A qu cdigos no verbales corresponden los rostros, los grupos de personas y las referencias del texto? 2 Explica qu sentimientos expresa cada una de las caras dibujadas en esta pgina. 3 Cmo se refleja la relacin existente entre las personas en su disposicin en el espacio?

4 Seala las referencias paralingsticas del texto de Adelaida Garca Morales e indica qu tipo de informacin comunican. 5 Escribe de nuevo ese texto aadiendo otras referencias paralingsticas e incorporando elementos relativos a la cinsica y a la proxmica.

1.

La comunicacin

25

ACTIVIDADES

REFUERZO

4. Variedades de la lengua
Cada lengua presenta diferencias o variedades internas: diatpicas, que constituyen sus dialectos, diastrticas, que se manifiestan en sociolectos, y diafsicas, que se plasman en distintos registros de lengua. En los siguientes textos puedes observar algunas de estas variedades.

Texto 1
PADRE.tonces, hijo? O quers decir algo ms? NIO.S. PADRE.Qu quers decir? NIO.No s. PADRE.A ver, pens. NIO.No s. PADRE.Y, dec cualquier cosa, lo primero que se te ocurra NIO.Qu? PADRE.Una palabra, por ejemplo. NIO.rbol!
Andrs NEUMAN Una vez Argentina, Anagrama

Texto 2
PEPE.Ahora lo que yo deploro vivsimamente es haber venido a producir a ustedes esta molestia suntuaria, este trasiego ornamental ANASTASIA.No, seor; no faltara otra cosa. Muchsimo gusto. Lo que usts se merecen y naa ms. PEPE.Oh! No diga usted eso; tanto agasajo nosotros, dos personas tan ANASTASIA.Y una lo que siente es no haber sabo antes lo que eran usts. PEPE.Oh, eso no, por Dios! Pero qu es lo que somos nosotros, diga usted? Haga el favor de decrmelo! Qu somos nosotros? ANASTASIA.Toma, pues meno! Digo Nada! Una friolera! Y por qu no han quero usts decirlo al llegar? PEPE.Pues no lo hemos querido decir porque francamente porque no lo sabamos que aqu se nos estimase de una manera tan halagea. ANASTASIA.Aqu, crea el seor, que, aunque esto es un humilde pueblo, se sabe tratar a las presonas de categora, como son los excelentsimos seores. (Aparte.) Voy a ver si son melitares. (Alto.) Y usts de qu son? PEPE.(Palpndose con asombro.) Cmo que de qu somos?
Carlos ARNICHES El santo de la Isidra. El amigo Melquades. Los caciques, Alianza

Texto 3
Pues yo me curro un ritmo guapo y luego le meto la meloda que me va saliendo, medio inventada. Yo no tengo ni guarra de solfeo, sabes, yo saco tres acordes, un ritmo y ya est; luego metemos la batera y el bajo, y as las canciones salen como churros. Cuando no tengo muchas ganas de currarme una letra, le digo al cantante que se la curre l, sabes Qu vas a hacer este verano? Currar para ver si me saco unas pelas para agosto. Voy a ver si puedo sacarme un curro como socorrista y luego puede que vaya a ver a Santi, el batera, a la Manga.
Jos ngel MAAS Historias del Kronen, Destino

Texto 4
La teora de la relevancia est basada en la tesis de que la atencin y el pensamiento humano obedecen a una constante bsqueda de informacin relevante, o sea, informacin que produzca efectos cognoscitivos adecuados a cambio de un esfuerzo de procesamiento mnimo. A este principio general sobre la cognicin humana, Sperber y Wilson, que presentaron la teora de la relevancia en 1986, y, en versin revisada, en 1995, aaden un segundo principio general acerca de la comunicacin, que puede enunciarse as: Cada acto de comunicacin ostensiva comunica la presuncin de su propia relevancia ptima. Este segundo principio es el ms importante en pragmtica, y en lo que sigue lo llamaremos simplemente: principio de relevancia.
Graciela REYES, Elisa BAENA y Eduardo URIOS Ejercicios de pragmtica (I), Arco Libros

1 Qu variedad interna del castellano presenta el texto 1? Dnde se registra? Seala los rasgos lingsticos presentes en el texto caractersticos de esta variedad. 2 Explica la diferencia en el habla de cada uno de los interlocutores del texto 2. Cmo se denominan estos usos? Indica los rasgos especficos de las palabras de Anastasia; cmo denominaras a su manera de hablar?

3 Qu tipo de variedad se observa en el texto 3? Indica sus rasgos caractersticos. Transforma el texto pensando en que la primera persona que habla est en una entrevista de trabajo. 4 Caracteriza el texto 4 segn su variedad. Seala vocablos que permitan tal caracterizacin. Qu tipo de lector exige este fragmento? 5 Redacta un texto sobre las variedades del castellano.

26 1.

La comunicacin

MATERIAL FOTOCOPIABLE / Oxford University Press Espaa, S. A.

ACTIVIDADES

ORTOGRAFA

5. Uso de la tilde
Para el correcto uso de la tilde debes recordar: Las reglas generales de acentuacin. Las palabras agudas llevan tilde cuando terminan en -n, -s o vocal (redaccin, ciprs, llev; laurel, radar, cantad); las llanas, cuando no acaban en esas letras (cncer, husped, rquiem; cantan, respetas, escriba); las esdrjulas y sobreesdrjulas siempre llevan tilde (nica, dgaselo). En las llanas, un caso especial lo constituyen aquellas palabras que acaban en -s precedida de otra consonante, las cuales llevan tilde: bceps, frceps. Los hiatos formados por una vocal abierta (a, e, o) tona y una cerrada (i, u) tnica, o viceversa, siempre llevan tilde: raz, bal, redo, odo, tena, huamos. En los adverbios en -mente, si el adjetivo base de derivacin lleva tilde, la palabra derivada la mantiene: corts > cortsmente. Por regla general, los monoslabos no llevan tilde: sol, di, dio, fue, fui La tilde diacrtica permite diferenciar palabras con los mismos sonidos pero con valores gramaticales y significados diferentes: an (Adv, todava) / aun (Conj, incluso); d (Verbo dar) / de (Prep); l (Pron) / el (Art); ms (Adv) / mas (Conj); m (Pron) / mi (Adj Pos); s (Verbo ser o saber) / se (Pron); s (Adv de afirmacin) / si (Conj); t (Sust) / te (Pron); t (Pron) / tu (Adj Pos). Los pronombres, adjetivos y adverbios interrogativos y exclamativos son palabras tnicas y llevan tilde: qu, quin, cul, cmo, dnde, cundo, cunto. El uso de la tilde es potestativo en el caso de solo (Adv, solamente) y este / a / s (Pron Dem), salvo cuando pueda haber ambigedad, en cuyo caso deben llevarla: Trabaja solo por las noches (Adj) / Trabaja slo por las noches (Adv); Esta maana los traer (Adj Dem) / sta maana los traer (Pron Dem). Las maysculas siempre llevan tilde: ngel, vila, CRDOBA

1 Coloca la tilde cuando corresponda. El proceso de indiferencia


La inferencia es el proceso por el que se reconstruyen los vinculos que permiten ligar la seal indicial y el contenido al que esta se refiere. Tipicamente, la recuperacion de este vinculo esta basada en la utilizacion de los conocimientos extralingisticos, que permiten salvar la distancia que media entre el estimulo utilizado y su contenido. [] La inferencia es, basicamente, un proceso por el que se integran los contenidos de diferentes representaciones internas, que pueden proceder, a su vez, de diferentes fuentes. Cuando procesamos datos lingisticos, combinamos la informacion obtenida a partir de la descodificacion del mensaje linguistico con otra informacion contextual.
MATERIAL FOTOCOPIABLE / Oxford University Press Espaa, S. A.

Son, por tanto, inferenciales los procesos que nos permiten especificar los contenidos codificados lingisticamente, como, por ejemplo, los que llevamos a cabo para identificar los referentes a los que aluden las expresiones linguisticas (a que se refiere la palabra casco en la frase Tendras que limpiar el casco?), o los que especifican contenidos vagos (cuando decimos El mio es mas rapido queremos decir que es mas rapido con respecto a que?). Y lo son tambien los que nos conducen a determinar que es exactamente lo que el interlocutor quiso comunicarnos, o en que sentido hay que tomar sus palabras (lo que dijo, era una sugerencia, o un consejo, o una orden, o una amenaza?). Sin embargo, no hay que perder de vista que los procesos inferenciales no son exclusivos del procesamiento de enunciados lingisticos: se dan tambien en la interpretacion de datos de otras fuentes, tanto si han sido producidos intencionalmente como si no (como cuando inferimos datos sobre la procedencia geografica de alguien a partir de su acento). Esto indica, por tanto, que los procesos inferenciales tienen lugar con independencia de que haya comunicacin; como vimos, tendremos que deducir informacion a partir de cualquier clase de datos disponible; la principal diferencia entre las seales producidas intencionalmente y la observacion de datos fortuitos radica en que solo las primeras se interpretan desde el supuesto de que quien las produce las respalda.
M. Victoria ESCANDELL VIDAL La comunicacin, Gredos

1.

La comunicacin

27

Evaluacin
El sueco
En qu puedo servirle? Quiero pasar asever el sueco con voz temblorosa. Vacil unos instantes, pero acab franquendole el paso, ya que se trataba de un cliente de mi hermana, autodenominado novio, por ms seas, y no me convena de modo alguno enemistarme con ella. Pens que quiz quera discutir algn asunto de familia y que, siendo yo el varn, me consideraba el interlocutor idneo para ello. Esta fineza, ya anacrnica, y algo en el aspecto del sueco me decan que estaba en presencia de un hombre de bien, y no menoscab mi estima el hecho de que sacara un pistoln de la faltriquera y me encaonara con l al tiempo que se sentaba en la cama. Pero me dan miedo las armas, o no habra tomado mi carrera delictiva tan corto vuelo, y as se lo hice saber. Veo, caballero dije lentamente, con profusin de ademanes y procurando vocalizar bien para que la barrera del idioma no fuera bice a nuestro entendimiento mutuo , que algo le impuls a desconfiar de m: quiz el natural recelo que inspira mi facha, quiz un rumor de esos a cuya divulgacin son dadas las malas lenguas. Sin embargo, puedo asegurarle por mi honor, el de mi hermana, sister, y el de nuestra santa madre, que Dios haya en su gloria, que no tiene usted nada que temer de m. Soy perspicaz y, aunque no tenga el placer de conocerle salvo superficialmente, no he dejado de advertir que es usted hombre de principios, instruido, cabal y de buena cuna, a quien acaso reveses de fortuna han lanzado a una vida desasosegada en pos de ms amplios horizontes, del olvido, incluso.

Discusin en la noche, por Edward Hopper.

Mi llaneza no pareca hacer mella en su obstinacin. Segua sentado en la cama, con los ojos clavados en m y el rostro inexpresivo, perdidos sin duda sus pensamientos en quin sabe qu recuerdos dolorosos, qu visiones indescriptibles, qu melancola.
Eduardo MENDOZA El misterio de la cripta embrujada, Seix Barral

1. Resume el contenido del texto y reconoce los elementos de la comunicacin que se establecen en l. Diferencia estos elementos de los del proceso externo de la comunicacin que se constituye al leer el texto.

6. Caracteriza los conceptos de lengua histrica y lengua


funcional y pon ejemplos tomados del texto. Seala, adems, los tipos de variedades que presenta cualquier lengua histrica. Lengua histrica Lengua funcional

2. De qu tipo de texto se trata segn su objetivo comunicativo? Seala otras clases posibles, atendiendo al mismo criterio de clasificacin.

7. Escoge una palabra del fragmento y explica, a partir de


ella, la nocin de signo lingstico. Signo lingstico

3. Qu intencin comunicativa manifiesta el protagonista


al proferir el largo parlamento que comienza con Veo, caballero ?

8. Qu tipo de signo es, segn la clasificacin de Pierce, la


palabra que has escogido en la actividad 7? Elige una de las opciones, justifica tu respuesta y explica la diferencia entre este tipo de signo y los otros dos. a) Smbolo. b) Icono. c) ndice.

4. Localiza en el fragmento de Eduardo Mendoza algunas


menciones de signos visuales.

5. En el texto se hace referencia a signos no verbales.


Copia las expresiones correspondientes y clasifcalas en un cuadro como este.

CDIGOS NO VERBALES
Paralingstica Cinsica Proxmica

9. Reconoce las funciones del lenguaje presentes en el texto


y brinda ejemplos para las que no encuentres.

10. Redacta un texto en el que expliques las caractersticas


de la facultad del lenguaje y de las lenguas humanas. No olvides cuidar la ortografa y la presentacin de tu escrito. 23 1.
La comunicacin

ACTIVIDADES

REFUERZO

1. La conversacin
La conversacin en la que se emplea la lengua coloquial es el discurso prototpico de la comunicacin oral. A veces, los textos escritos imitan o reproducen la variedad coloquial con distintos objetivos.

Texto 1
CHUSA.Dnde estn estos? JAIMITO.Se han largado. CHUSA.Adnde? JAIMITO.(Sale del lavabo y se acerca.) Se han largado del todo; se han abierto, ta. Se han llevado sus cosas Quedan esas cajas de ah; van a venir luego a por ellas. En eso han quedado. (De pronto ella toma contacto con la realidad. Ve las cajas. Luego las cosas que faltan y el cambio en la habitacin.) CHUSA.(Deja de guardar la ropa y se sienta muy afectada.) Pero, cmo? Qu ha pasado? JAIMITO.Se han largado, juntos, los dos. Los dos y sus madres. Los cuatro. Bueno, y el padre. Se van a casar. Han cogido un piso en Mstoles. El da que yo sal del hospital, y te cogieron a ti, fue todo un lo. CHUSA.Qu tal sigue tu brazo? JAIMITO.(Sacndole y metindole del pauelo con que se le sujeta al cuello.) Bueno, mira. Le puedo mover ya. Maana o pasado me quito esto. Pues nada, que se han ido. CHUSA.Alberto tambin? JAIMITO.No te digo que se han ido los dos juntos?
Jos Luis ALONSO DE SANTOS Bajarse al moro, Ctedra

Texto 2
S1: t qu Csar? que como nunca has fumao ni nada no tienes problemas/// t no has fumao nunca C1: si/ fumaba S2: mm? C2: fumaba yy- y lo dej/ tam(b)in/ me senta mal del pecho// yy lo dej/ yy- lo que pasa es que noo// yo hasta los dieciocho aos no fumaba J: yo hasta los diecinueve
MATERIAL FOTOCOPIABLE / Oxford University Press Espaa, S. A.

C3: yy luego he estao fumando hastaa hacee dos aos o as / peroo no- no me gustaba nadaa/ as paraa/ yo de pequeo haba tenido bronquitis/ y cosas de este tipo/ y ahora to(d)avaa/ mm-cada poco tengo de la garganta o dee/ y entonces lo del fumar era/ era una/ era una pesadez
Antonio BRIZ GMEZ El espaol coloquial en la conversacin, Ariel

1 Qu tipo de informacin aportan las acotaciones en el primer texto? A qu elementos del acto comunicativo se refieren? 2 Hay informacin del mismo tipo en el segundo texto? Qu diferencias encuentras en el registro de habla de los interlocutores? 3 Lee el primer texto aportando los aspectos prosdicos y los elementos no verbales propios de la comunicacin oral.

4 Distingue qu tipo de conversacin (oral coloquial o escrita que imita la variedad coloquial) se representa en cada uno de los fragmentos. 5 Reconstruye una versin escrita del segundo texto similar al anterior. 6 Reconoce las caractersticas de la lengua coloquial presentes en los textos.

2.

La comunicacin oral y escrita

37

ACTIVIDADES

REFUERZO

2. La entrevista
La entrevista es una conversacin que informa sobre las opiniones de una persona. Con frecuencia, las noticias o los reportajes de la prensa incluyen declaraciones, dilogos con el periodista, que no tienen la forma de una entrevista larga y formal. En este caso, podemos distinguir tres tipos de representacin escrita del dilogo: una que comienza con la presentacin del entrevistado y contina con la modalidad pregunta-respuesta; otra que contiene la reproduccin de las respuestas del entrevistado, con la intercalacin de informacin sobre su vida y obra y comentarios o descripciones del entrevistador; y finalmente la que mezcla ambas formas.

Texto 1
Empieza la conversacin hablando de cmo encuentran los temas que llevan a los escenarios. No somos especialmente originales, reconoce Fernndez. Tomamos los temas de aquello que nos choca de la propia realidad, cosas que vemos en la prensa o a nuestro alrededor. Luego est la historia. Y, no puedo evitarlo, soy fillogo, la revisin de la literatura. En mi caso, a veces surge de la imaginacin de forma azarosa y en otras ocasiones del deseo de tratar un tema que me lleva a algo concreto para lo que tengo que buscar la forma, explica Mayorga. La tortuga de Darwin procede directamente de esta foto (muestra un recorte de peridico []) que me dispar la imaginacin por un personaje que ha sido testigo de la Revolucin de Octubre y la perestroika, ha conocido a doce papas y a treinta y cinco presidentes de Estados Unidos. Me pareci interesante y busqu la forma teatral. [] P.Y qu echan de menos en la escena espaola? J. MAYORGA.Una escritura original que consiga un teatro completamente actual a la vez que intemporal. Espacios, como en Inglaterra y Francia, que desafen a los autores para que sus obras no sean la rplica de lo que sale en los peridicos. J. R. FERNNDEZ.Que las obras puedan reposar y madurar en los escenarios. La mirada de los jvenes autores y la de los mayores.
El Cultural, 7 de febrero de 2008

Texto 2
S, es guapsima. Ojos de mar, pmulos redondos, labios llenos, piel de melocotn, una preciosidad. Cierto que llega maquillada de un acto de las marcas a las que vende cara su imagen. Pero la luz que emana su rostro supera las prestaciones del mejor iluminador cosmtico. Parece que se acaba de tragar, encendida, una bombilla de cien vatios. No puedes dejar de mirarla. P.Cundo fue consciente del efecto que causaba en los dems? R.De nia. Tendra doce aos. Los chicos empezaron a dejarme notitas diciendo que estaban por m. Y me lo cre, me puse chulita, me crec. Pero mis amigas me pusieron en mi sitio. Me dejaron de lado, completamente sola. Fue muy duro, me di cuenta de que gustar es agradable, pero hay otras cosas mucho ms importantes. Aprend la leccin.
El Pas Semanal

Texto 3
El Libro Guinness de los rcords tiene registrada como la mujer ms anciana del mundo a Edna Parker, de Illinois (EE. UU.), con 115 aos. Obviamente, no tienen constancia de la existencia de la abuela de El Vacie (Mara Daz Corts, 116 aos, de una barriada de Sevilla). A Mara esas historias no la interesan. Est cansada de las televisiones y los peridicos. Yo qu soy, un divertimento na ms?, protesta. Son las dos. Aunque es la hora de comer, la abuela Mara decide volverse a la cama. Estoy cansada y se me va la memoria, se disculpa. Su hija y su nieta la cogen en volandas. Es tan menuda que apenas las cuesta levantarla. Antes de desaparecer, mira desconfiada a la visita: Qu dice esa paya? Qu me van a dar la casita o qu?, pregunta ceuda. Manuel se acerca a su bisabuela, con un chupete en la boca, y ella, desde arriba, le mira fijamente. Pero qu guapo est mi nio, presume a gritos. Entonces la preguntan que cmo se siente ella. Y Mara sentencia, resuelta: Pues igual de joda que siempre, hija. Cmo voy a estar?.
El Pas, 3 de febrero de 2008

1 Cmo se estructuran los dos primeros textos? 2 Indica los temas principales del primer texto. Cmo se sealan las declaraciones de los dramaturgos? 3 En qu tipo de entrevistas incluiras el segundo texto? Por qu? 4 La intervencin del periodista del texto 2, aporta informacin sobre el personaje?

5 El tercer texto constituye una entrevista? En qu gnero periodstico incluiras el texto? 6 Identifica rasgos de la lengua coloquial en estos textos. 7 Realiza una entrevista a un compaero o compaera que desarrolle una actividad literaria, artstica o deportiva. Utiliza la modalidad pregunta-respuesta y reelabora la informacin siguiendo el modelo del texto 2.

38 2.

La comunicacin oral y escrita

MATERIAL FOTOCOPIABLE / Oxford University Press Espaa, S. A.

ACTIVIDADES

ORTOGRAFA

3. Signos de exclamacin, interrogacin y comillas


Recuerda que los signos de interrogacin y exclamacin se utilizan para delimitar enunciados interrogativos y exclamativos en estilo directo. Son dos (apertura y cierre) y se colocan al principio y al final del enunciado. Despus de los signos no se escribe nunca punto. El signo de principio de pregunta o exclamacin se coloca donde esta empieza, aunque no coincida con el comienzo del enunciado: Pero entonces, por qu no se lo dijiste?; Lo esper en vano, se haba ido sin m! Si se escriben varias preguntas o exclamaciones seguidas y breves, se pueden considerar oraciones independientes, con mayscula cada una y sus signos respectivos: Eso te dijo? Por qu no le contestaste?; Silencio! Empieza la funcin! Si forman parte de un mismo enunciado, hay que separar las oraciones por comas o por puntos y comas y solo la primera se escribir con maysculas: Qu ha pasado?, quin ha sido?, por qu?; Qu calor!, abre la ventana!, no puedo respirar! Las comillas, dobles () o simples ( ) se utilizan en los siguientes casos: Para reproducir citas textuales. Los chicos opinan: No es tan guapo como dicen. En los textos narrativos en ocasiones se utilizan las comillas para reproducir los pensamientos de los personajes, a diferencia de las rayas que transcriben el discurso directo. Para indicar que una palabra o expresin es impropia, vulgar o de otra lengua, se utiliza de forma irnica o con un significado especial (Hacerlo? Pa qu?; Hoy tambin ha faltado, est enferma), o cuando se comenta o se trata una palabra en particular. En estos dos ltimos casos, en textos impresos, se utiliza la letra cursiva o se cambia el tipo de letra.

Texto 1
Era la segunda vez en mi vida que alguien me llamaba escritor, y me abrum una mezcla inextricable de vergenza y de orgullo, y tambin una oleada de afecto por Rodney. No dije nada, pero, como el hombre no pareca dispuesto a invitarme a entrar, ni a deshacer el silencio, por decir algo le pregunt si era el padre de Rodney. Me dijo que s. Luego volv a preguntarle por Rodney y me respondi que no saba dnde estaba.

Texto 3
Qu es lo que queras preguntarme [] Quin es Tommy Birban La cara de Rodney no se alter, y yo no supe leer la mirada de su ojo nico, o quiz es que no haba nada que leer en ella. Cuando habl a continuacin consigui que su voz sonara natural De dnde has sacado ese nombre Lo mencion tu padre Dijo que antes de que te marchases de Urbana t y l hablasteis por telfono, y que por eso te marchaste No te dijo nada ms Qu ms debera haberme dicho [] En aquel momento anunciaron por megafona la llegada inminente del tren de Atocha Tommy era un compaero dijo Rodney Lleg a Luang Nai cuando yo era un veterano, y nos hicimos muy amigos Nos marchamos de all casi al mismo tiempo, y desde entonces no he vuelto a verle
Javier CERCAS La velocidad de la luz, Tusquets

Texto 2
MATERIAL FOTOCOPIABLE / Oxford University Press Espaa, S. A.

A Rodney no le interesaba discutir el argumento de mi libro, que era en cambio el punto que ms me preocupaba a m, sino quin desarrollaba el argumento. Las historias no existen, me dijo una vez. Lo que s existe es quien las cuenta. Si sabes quin es, hay historia; si no sabes quin es no hay historia. Entonces yo ya tengo la ma, le dije. Le expliqu que lo nico que tena claro en mi novela era precisamente la identidad del narrador: un tipo exactamente igual que yo que se hallaba exactamente en las mismas circunstancias que yo. Entonces el narrador eres t mismo?, conjetur Rodney. Ni hablar, dije, contento de ser ahora yo quien consegua confundirle. Se parece en todo a m pero no soy yo.

1 Transforma el texto 1 en un dilogo en estilo directo. 2 Coloca las comillas que faltan en el segundo texto.

3 Pon los signos de interrogacin y los puntos que faltan en el texto 3. 4 Escribe una narracin (sin dilogos) del dilogo citado.

2.

La comunicacin oral y escrita

39

Evaluacin 2
Compromiso con mayo
Al seor me lo he encontrado en el portal cuando yo entraba dijo Consuelo. Iba de mala leche. O, bueno, no s si es que la tiene tomada conmigo. No mujer. Es que trabaja mucho. Jolines, pero tambin ganar pasta. El que quiere la col, quiere las hojitas de alrededor, no? Ahora, eso s, lo que est es muy moderno. Se da un flash a Mario Conde. Luego me pregunt que si anoche habamos estado de fiesta. S, pero no aqu, en casa de otra gente. Una casa a todo tren. Te acuerdas de aquel seor alto que nos arregl el cuarto de bao? Pues all. El del Escorial? Vaya que si me acuerdo. Estaba como para hacerle padre, no cree usted? Yo no. Pero gustos son gustos. La nevera estaba pelada. Me tom el caf y le dej dinero a Consuelo para que hiciera una buena compra y les preparara algn guiso rico a las chicas, que seguramente se quedaran a comer. Qu pasa? Que usted se larga? P ues s, hija, me largo. Es uno de mayo y me voy por ah a celebrarlo a mi manera. Es el aniversario de su boda? De mis bodas con mayo. Has visto qu da hace? Aqu estoy de ms y mayo me echa de menos. Consuelo se qued mirndome con los ojos muy abiertos. Qu fuerte!, mayo me echa de menos. Lo ha inventado usted?
Carmen MARTN GAITE Nubosidad variable, Crculo de Lectores La mujer de la casa, por Marcel Gromaire.

1. Redacta un resumen del contenido del texto de Carmen


Martn Gaite.

6. En qu se distinguen los gneros orales monologados


de los dialogados? Expn las diferencias entre unos y otros brevemente.

2. Identifica las modalidades textuales que aparecen en


ese fragmento y reconoce las funciones del lenguaje predominantes. Modalidades textuales Funciones del lenguaje

7. Existe una oposicin clara entre el habla y la escritura?


Cita ejemplos de textos escritos con rasgos orales.

8. Enuncia tres criterios de clasificacin de los textos escritos. 9. Confecciona una tabla con las principales diferencias
entre el habla y la escritura.
! )# u $ ' ! )# u ! . -% /

3. En este texto se imita un gnero oral. Indica qu caractersticas propias de una conversacin cotidiana observas en las palabras de las dos mujeres.

4. Indica el registro y el nivel lingstico utilizados por los


personajes. Cul de ellos emplea ms expresiones coloquiales? Menciona algunas de ellas.

5. Cita los principales gneros orales dialogados y enumera


las caractersticas de cada uno.

10. Escribe una narracin (sin incluir ningn dilogo) acerca


de lo que ocurre y se dice en el fragmento Compromiso con mayo. 37 2.
La comunicacin oral y escrita

ACTIVIDADES

DOCUMENTACIN

1. Lenguas y legislacin
El carcter plurilinge del Estado espaol est reconocido en la legislacin. La Constitucin espaola de 1978, en su artculo 3. (reproducido en la pgina 39 del Libro del alumno) establece la oficialidad de las lenguas. Adems, cada una de las comunidades autnomas bilinges ha legislado al respecto en sus estatutos de autonoma.

Estatuto de Aut
ARTCULO 5.

onoma de Gal

icia

de Catal Estatuto de Autonoma

ua

ARTCULO 3. Catalua es el cataln. 1) La lengua propia de as el oficial de Catalua, 2) El idioma cataln es el o tod en l cia ofi castellano, como tambin lo es el Estado espaol. cial izar el uso normal y ofi 3) La Generalidad garant rias esa nec as did me ptar las de los dos idiomas, ado dicon las ar cre y to en mi para asegurar su conoci en ad ald anzar su plena igu ciones que permitan alc ciu los de res rechos y debe lo que se refiere a los de dadanos de Catalua.

1) La lengua pr opia de Galicia es el gallego. 2) Los idiomas gallego y cast el lano son oficia Galicia y todo les en s tienen el de recho a cono usarlos. cerlos y 3) Los podere s pblicos de Galicia garant uso normal y izarn el oficial de los do s idiomas y po rn la utilizaci tencian del gallego en todos los r de la vida pbl denes ica, cultural e informativa, y drn los medio dispons necesarios pa ra su conocim 4) Nadie podr iento. ser discrimin ado por razn lengua. de la

Estatuto de Aut

ARTCULO 3. La lengua cata lana, propia de las Illes Balears, junto con la ca tendr, stellana, el car cter de idioma y todos tienen oficial, el derecho de co nocerla y utiliza rla.

onoma de las

Illes Balears

Estatuto de Autonoma

del Pas Vasco

de Estatuto de Autonoma Valenciana


MATERIAL FOTOCOPIABLE / Oxford University Press Espaa, S. A.

la Comunitat

ARTCULO 7. Autciales de la Comunidad 1) Los dos idiomas ofi nen tie os Tod o. lan y el castel noma son el valenciano s. usarlo derecho a conocerlos y uso ciana garantizar el en Val t 2) La Generalita me las r pta ado y as s lengu normal y oficial de las do gurar su conocimiento. didas necesarias para ase su criminado por razn de 3) Nadie podr ser dis lengua. recuproteccin y respeto a la 4) Se otorgar especial peracin del valenciano.

ARTCULO 6. pia del pueblo vasco, ten 1) El euskera, lengua pro en l cia ofi carcter de lengua dr, como el castellano, de ntes tienen el derecho ita hab Euskadi; todos sus guas. conocer y usar ambas len es de la Comunidad Au un com 2) Las instituciones nioli soc d ida nta la divers tnoma, teniendo en cue antizarn el uso de ambas gar co, Vas s Pa gstica del y cter oficial y arbitrarn lenguas, regulando su car ase a par s medios necesario regularn las medidas y gurar su conocimiento. la criminado por razn de 3) Nadie podr ser dis lengua. ia, gua Vasca, Euskaltzaind 4) La Academia de la Len cta pe oficial en lo que res es la institucin consultiva al euskera.

Ley orgnica de
ARTCULO 9.

l rgimen Fora

l de Navarra

1) El castellano es la lengua of icial de Navarra 2) El vascuenc . e tendr, tambi n , carcter de le oficial en las zo ngua nas vascoparla ntes de Navarra .

3.

La realidad plurilinge de Espaa

49

ACTIVIDADES

DOCUMENTACIN

2. Leyes de normalizacin lingstica


Todas las comunidades autnomas con lengua propia han promulgado leyes que desarrollan los artculos de los estatutos referidos a las lenguas. A continuacin, citamos algunos artculos de esas leyes.

Ley de normalizacin lingstica de TTULO I

Galicia (1983)
Ley de normalizacin lingstica de Catalua (1983) TTULO PRELIMINAR
Artculo 1. 1) La presente ley tiene por objeto el desarrollo del artculo 3.o del Estatuto de Autonoma de Catalua para llevar a cabo la normalizacin del uso de la lengua catalana en todos los mbitos y garantizar el uso normal y oficial del cataln y el castellano. 2) Dada la situacin lingstica de Catalua, son, pues, objetivos de esta ley: a) Amparar y fomentar el uso del cataln por parte de todos los ciudadanos. b) Dar efectividad al uso oficial del cataln. c) Normalizar el uso del cataln en todos los medios de comunicacin social. d) Asegurar la extensin del conocimiento del cataln.

Artculo 1. los gallegos tienen el deber de El gallego es la lengua propia de Galicia. Todos . conocerlo y el derecho de usarlo Artculo 2. al del gallego y del castellano, Los poderes de Galicia garantizarn el uso norm lenguas oficiales de la Comunidad Autnoma. Artculo 3. medidas oportunas para que Los poderes pblicos de Galicia adoptarn las nadie sea discriminado por razn de lengua. tribunales para obtener la proLos ciudadanos podrn dirigirse a los jueces y a. lengu la teccin judicial del derecho a emplear

TTULO II
Artculo 4. a oficial de las instituciones 1) El gallego, como lengua propia de Galicia, es lengu de la Administracin local cin, nistra Admi su de oma, de la Comunidad Autn Comunidad Autnoma. y de las entidades pblicas dependientes de la

TTULO I
Artculo 5. 1) El cataln, como lengua propia de Catalua, lo es tambin de la Generalidad y de la Administracin territorial catalana, de la Administracin local y de las dems corporaciones pblicas dependientes de la Generalidad.

TTULO III
Artculo 12. tambin lengua oficial en la 1) El gallego, como lengua propia de Galicia, es tivos. enseanza en todos los niveles educa Artculo 13. anza en su lengua materna. 1) Los nios tienen derecho a recibir la primera ense oma arbitrarn las medidas 2) Las autoridades educativas de la Comunidad Autn en la enseanza. o galleg encaminadas a promover el uso progresivo del Artculo 14. torio en todos los niveles edu1) La lengua gallega es materia de estudio obliga cativos no universitarios.

TTULO II
Los nios tienen derecho a recibir la primera enseanza en su lengua habitual, ya sea esta el cataln o el castellano. La Administracin debe garantizar este derecho y poner los medios necesarios para hacerlo efectivo. [] 3) La lengua catalana y la lengua castellana deben ser enseadas obligatoriamente en todos los niveles y grados de la enseanza no universitaria. 4) Todos los nios de Catalua, cualquiera que sea su lengua habitual al iniciar la enseanza, deben poder utilizar normal y correctamente el cataln y el castellano al final de sus estudios bsicos.

TTULO IV
Artculo 18. de radio y televisin y en los El gallego ser la lengua usual en las emisoras a gestin o competencia de dems medios de comunicacin social sometidos oma. las instituciones de la Comunidad Autn

TTULO III
Artculo 21. 2) El Consejo Ejecutivo de la Generalidad debe reglamentar la normalizacin del uso de la lengua en los medios de comunicacin social sometidos a la competencia o gestin de la Generalidad, con el objetivo de asegurar la comprensin y mejorar el conocimiento de la lengua catalana teniendo en cuenta la situacin lingstica de cada medio en concreto.

Ley de normalizacin lingstica de las Illes Balears (1986) TTULO PRELIMINAR

Artculo 1. La presente ley tiene por objeto desarrollar el artculo 3.o del Estatuto de Autonoma en lo que respecta a la normalizacin de la lengua catalana como propia de las Illes Balears en todos los mbitos y garantizar el uso del cataln y del castellano como idiomas oficiales de la Comunidad Autnoma.

50 3.

La realidad plurilinge de Espaa

MATERIAL FOTOCOPIABLE / Oxford University Press Espaa, S. A.

Ley sobre el uso y la enseanza del valenciano (1983) TTULO PRELIMINAR


Artculo 1. 1) La presente ley tiene por objeto genrico dar cumplimiento y desarrollar lo dispuesto en el artculo 7. del Estatuto de Autonoma, regulando el uso normal y oficial del valenciano en todos los mbitos de la convivencia social, as como su enseanza. 2) En base a ello son objetivos especficos de la presente ley los siguientes: a) Hacer efectivo el derecho de todos los ciudadanos a conocer y usar el valenciano. b) Proteger su recuperacin y garantizar su uso normal y oficial. c) Regular los criterios de aplicacin del valenciano en la Administracin, los medios de comunicacin social y la enseanza. d) Delimitar los territorios en los que predomine el uso del valenciano y del castellano. e) Garantizar, con arreglo a principios de graduabilidad y voluntariedad, el conocimiento y uso del valenciano en todo el mbito de la Comunidad.

DOCUMENTACIN

ACTIVIDADES

TTULO I
Artculo 7. 1) El valenciano, como lengua propia de la Comunidad Valenciana, lo es tambin de la Generalidad y de su Administracin pblica, de la Administracin local y de cuantas corporaciones e instituciones pblicas dependan de aquellas.

TTULO II
Artculo 18. 1) La incorporacin del valenciano a la enseanza en todos los niveles educativos es obligatoria. [] 3) El valenciano y el castellano son lenguas obligatorias en los planes de enseanza de los niveles no universitarios. [] Artculo 19. 2) [] Al final de los ciclos en que se declara obligatoria la incorporacin del valenciano a la enseanza y cualquiera que hubiera sido su lengua habitual al iniciar los mismos, los alumnos han de estar capacitados para utilizar, oralmente y por escrito, el valenciano en igualdad con el castellano.

TTULO III
Artculo 25. 1) El Consejo de la Generalidad Valenciana velar para que el valenciano tenga una adecuada presencia en aquellas emisoras de radio y televisin y dems medios de comunicacin gestionados por la Generalidad Valenciana, o sobre los que la misma ley tenga competencia, de acuerdo con lo dispuesto en la presente ley.

Ley bsica de no TTULO PRELIMINAR


Artculo 2. La lengua propia de

rmalizacin de

l uso del eusker a (1982)

scuence Ley foral del va 6) 98 en Navarra (1 TTULO PRELIMINAR


Artculo 1. enciales: 2) Son objetivos es s ho de los ciudadano a) Amparar el derec de y ce en el vascu a conser var y usar rlo ce ha ra pa s to finir los instrumen . vo cti efe lo eracin y el desarrol b) Proteger la recup do lan varra, sea del vascuence en Na fomento de su el ra pa s las medida uso. l o y la enseanza de c) Garantizar el us de os ipi inc pr a lo vascuence con arreg ualidad y respeto, voluntariedad, grad realidad sociolinde acuerdo con la . gstica de Navarra es del vascuence tal lec 3) Las variedades dia peto jeto de especial res en Navarra sern ob y proteccin.

MATERIAL FOTOCOPIABLE / Oxford University Press Espaa, S. A.

llano. Artculo 6. 1) Se reconoce a to dos los ciudadanos el derecho a usar tan relaciones con la Ad to el euskera como ministracin pblica el castellano en sus en el mbito territori atendidos en la len al de la Comunidad gua oficial que elijan Autnoma, y a ser . Artculo 15. Se reconoce a todo alumno el derecho a recibir la enseanz los diversos niveles a tanto en euskera educativos. como en castellano A tal efecto, el Parla en mento y el Gobiern o adoptarn las me cin progresiva del didas oportunas ten bilingismo en el sis dentes a la general tema educativo de izala Comunidad Aut Artculo 17. noma del Pas Vasco . El Gobierno adoptar aquellas medidas en ca minadas a garantiz igualdad de condici ar al alumnado la po ones, de poseer un sibilidad real, en conocimiento prctic finalizar los estudio o suficiente de amba s de enseanza ob s lenguas oficiales al ligatoria y asegurar del mismo vehculo el uso ambiental de de expresin norm l euskera, haciendo al, tanto en las activ actuaciones y docu idades internas como mentos administra externas y en las tivos.

TTULO II

l Pas Vasco es el eu skera. Artculo 3. Las lenguas oficiales en la Comunidad Au tnoma del Pas Va sco so

n el euskera y el caste

FUENTE: Maitena ETXEBARRA ARSTEGUI, El bilingismo en el Estado espaol, FBV.

3.

La realidad plurilinge de Espaa

51

2. Leyes de normalizacin lingstica

ACTIVIDADES

AMPLIACIN

3. Lenguas en contacto
Los siguientes textos tratan consecuencias del contacto de lenguas en las comunidades vasca y catalana.

Texto 1
En los ltimos aos se detecta un aumento del inters por la calidad lingstica de la produccin en euskera. Euskaltzaindia (1994) ha mostrado su preocupacin porque se est produciendo en los medios de comunicacin y entre los hablantes ms jvenes una ruptura con la tradicin de proporciones notables. Tambin en el mbito de la institucin escolar existen voces que sealan que la lengua est sufriendo un desgaste considerable, sobre todo debido a la introduccin masiva del euskera en mbitos castellano-hablantes, a travs de programas de inmersin. En opinin de algunos, se ha centrado ms la atencin en la produccin en euskera que en la calidad de la misma. [] Los usos incorrectos son debidos en gran parte, sin duda, a las interferencias y calcos provocados por la lengua dominante respectiva (castellano o francs), que afectan a todos los niveles de la lengua (fontico-fonolgico, morfosintctico y lxico-semntico). En cuanto a la forma que est adquiriendo la lengua estndar en los ltimos aos, el acadmico Ibon Sarasola ha sealado que se est produciendo un divorcio preocupante entre las variedades del Sur de los Pirineos y las del Norte. Sarasola, que trabaja sobre todo en el mbito del lxico, apunta que existe el riesgo de que el vasco quede reducido a la condicin de lengua criolla del castellano, y propone tender al equilibrio entre las variedades peninsulares y las continentales. En el campo de la sintaxis tambin hay varias propuestas para el desarrollo de la variedad estndar, observndose en todas ellas una preocupacin comn: la de que el euskera no se convierta en una traduccin del castellano ms torpe (debate que afecta de lleno a los medios de comunicacin). En resumen, cabe sealar que en los ltimos aos est fortalecindose, desde diferentes sectores, el inters por aumentar no solo la cantidad de produccin en euskera, sino tambin la calidad de la misma; ello significa un intento de mantener una relacin ms estrecha con los usos tradicionales de la lengua, y la bsqueda de autonoma en su desarrollo con respecto, sobre todo, al castellano.
Julin MAIA LARRETXEA La lengua euskera en la enseanza Textos de didctica de la lengua y de la literatura, n. 18

Texto 2
En el interior de la sociedad, las interferencias, los prstamos y la alternancia en el uso del cataln y el castellano constituyen un hecho. Se puede constatar en el cataln hablado, condicionado secularmente por las estructuras del castellano a niveles cultos, ms modernamente por el peso de muchos aos de escolarizacin exclusiva en castellano, y ms recientemente por la influencia del castellano coloquial de importantes sectores de poblacin procedentes de la inmigracin. Lo prueba la abundancia, entre otras, de obras de divulgacin sobre el uso de un cataln correcto []; todo ello con una tradicin (obsesin) que se remonta ya a las primeras pocas de la moderna normativizacin. La competencia oral en castellano de los catalanohablantes es suficiente para permitirles, por ejemplo, cuando narran en cataln, reproducir textualmente intervenciones de terceros en lengua castellana. El cataln hablado en todo el territorio delata a la otra lengua en contacto, ms o menos segn cada hablante: pues (doncs), bueno (b), bandeja (safata), relleno (farcit), tiburn (taur), ojal (tant de bo) Pero hay que decir que los prstamos del cataln al castellano estn vivos y arraigados incluso entre monolinges castellanohablantes: paleta (albail), tornavs (destornillador), enchegar (poner en marcha), rachola (baldosa), enracholar (alicatar), enracholador (alicatador), collar (apretar), no cal (no es necesario), plegar (dar de mano), baldufa (peonza), pencar (currar), charrar (charlar), colla (grupo, pandilla), no me veo (no veo), adeu (adis) Se trata de catalanismos a menudo difciles de sustituir en los registros coloquiales del castellano hablado en los territorios de lengua catalana. Estos fenmenos, producidos de manera absolutamente indiscriminada y espontnea, tienen ahora un regulador en el sistema educativo de las tres comunidades, al cual, por primera vez en amplias zonas, o de forma mucho ms generalizada en otras, le compete la informacin y la formacin en ambas lenguas.
Luis LPEZ DEL CASTILLO La lengua catalana en la enseanza Textos de didctica de la lengua y de la literatura, n. 18
MATERIAL FOTOCOPIABLE / Oxford University Press Espaa, S. A.

1 Cul es la mayor preocupacin respecto al uso del euskera? A qu factor social se debe esta situacin? 2 Indica las causas de los usos incorrectos. 3 Menciona las ideas principales que se desarrollan en la conclusin del texto de Julin Maia Larretxea. 4 Qu fenomenos de contacto de lenguas son habituales en el uso del cataln y el castellano? Explica en qu consisten. 5 Indica a qu registro de lengua del cataln afectan principalmente y cules son los factores histricos y sociales que han favorecido su desarrollo. 6 Cita ejemplos de interferencias lxicas en ambas lenguas. 7 Explica la conclusin a la que llega el autor del texto 2.

52 3.

La realidad plurilinge de Espaa

ACTIVIDADES

ORTOGRAFA

4. Uso de ll e y
El dgrafo ll representa el fonema lateral palatal /l /. Se escriben con ll: Las palabras de uso general terminadas en -illo, -illa: chiquillo, bombilla, campanilla. La mayor parte de los verbos terminados en -illar, -ullar y -ullir: acuchillar, apabullar, bullir. La letra y puede representar dos fonemas distintos: uno equivalente al representado por la letra i en palabras como estoy, voy; y otro consonntico, el fonema lateral palatal sonoro, en voces como bueyes, cuyo. Se escriben con y: Las palabras que terminan con el sonido correspondiente a /i / precedido de una vocal con la que forma diptongo, o dos con las que forma triptongo: soy, rey, Paraguay. Las palabras con sufijos ad-, dis- y sub-: adyacente, disyuntiva, subyacente. Formas verbales de caer, recaer, leer, creer, poseer, proveer, sobreseer; y de los verbos acabados en -or y -uir: cay, ley, creyeron, oyendo, huy, contribuyeron. Las palabras con la slaba -yec-: proyectaron, inyectar. Los plurales de los sustantivos terminados en -y: ley, leyes. El gerundio del verbo ir: yendo En algunas zonas de habla castellana, se produce el fenmeno del yesmo, es decir, la pronunciacin como /y/ de palabras con ll y con y: vaya y valla se pronuncian igual, lo cual conduce a numerosas incorrecciones grficas.

MATERIAL FOTOCOPIABLE / Oxford University Press Espaa, S. A.

1 Completa con ll o y las palabras siguientes: foeto apoaron pro__ecto jeringuia panta__a ampoa re__enar o__ente subra__ar arro__uelo atribu__ enro__arse cn__uge ro__o in__eccin desfa__ecer desma__o ca__eron desarro__aron arro__o pi__aste ensa__ ha__arn influ__ente a__udaron pe__orativo fo__etn ve__o be__o exclu__ disminu__a ta__aron fo__n o__a 2 Escribe halla o haya, segn corresponda: a) Cuando alguna novedad, te llamaremos. b) la solucin a estos problemas. c) Quien te lo dicho, te ha mentido. d) No se a gusto en esta ciudad. e) No creo que llegado nadie an.

3 Escribe los gerundios de los siguientes verbos. ir or huir influir destruir construir incluir caer leer atraer extraer retraer contraer proveer atribuir retribuir concluir excluir 4 Conjuga los siguientes verbos en tercera persona del singular y del plural del pretrito perfecto simple de indicativo y del presente del subjuntivo: disminuir, influir, constituir, obstruir, diluir. 5 Completa las siguientes oraciones con las formas verbales correspondientes:
a) Ten cuidado, se est

el lquido! maana;

(Caer)
b) No estoy seguro de que

estamos muy cansados. (Ir)


c) No se sabe nada del joven que ayer

de su casa. (Huir)
d) Silencio! No estis

lo que os dice. los accidentes de

(Or)
e) El mes pasado

trfico. (Disminuir)
f) Ayer todos

las actividades programadas. (Concluir)

3.

La realidad plurilinge de Espaa

53

Evaluacin 3

Danzas en Egaray, por Martnez Ortiz.

La lengua de los vizcanos


La lengua que oy se habla en Castilla, de la qual vosotros queris ser informados tiene parte de la lengua que se usava en Espaa antes que los romanos la enseoreassen, y tiene tambin alguna parte de la de los godos, que sucedieron a los romanos, y mucha de la de los moros, que reinaron muchos aos, aunque la principal parte es de la lengua que introduxeron los romanos, que es la lengua latina, ser bien que primero esaminemos qu lengua era aquella antigua que se usava en Espaa antes que los romanos viniessen a ella. Lo que por la mayor parte los que son curiosos destas cosas tienen y creen es que la lengua que oy usan los vizcanos es aquella antigua espaola. Esta opinin confirman con dos razones harto aparentes. La una es que, ass como las armas de los romanos quando conquistaron la Espaa no pudieron passar en aquella parte que llamamos Vizcaya, ass tampoco pudo pasar la lengua al tiempo que, despus de averse hecho seores de Spaa, quisieron que en toda ella se hablasse la lengua romana. La otra es la disconformidad que tiene la lengua vizcana con cualquiera de todas las otras lenguas que el da de oy en Espaa se usan.
Juan DE VALDS Dilogo de la lengua, Castalia

1. Contesta las siguientes cuestiones:


Qu informacin da el autor sobre el origen y formacin de la lengua espaola? Con qu lengua actual se identifica la antigua que se usava en Espaa antes que los romanos viniessen? Qu datos aporta sobre ella?

5. Nombra las lenguas oficiales de Espaa, en qu comunidades tienen ese carcter y sus variedades dialectales.

6. Cules son los llamados dialectos histricos? Indica


sus caractersticas ms destacadas.

7. Explica los objetivos de las leyes de normalizacin lingstica y seala los mbitos a los que se refieren.

2. Resume los hechos histricos que han originado la


diversidad lingstica de la Espaa actual.

8. Qu fenmenos de las lenguas en contacto pueden


darse en las comunidades bilinges? En qu consisten?

3. El texto de Juan de Valds, escrito en el siglo XVI, reproduce de modo aproximado caractersticas fnicas del castellano de la poca. Cita algunos ejemplos.

9. Realiza un esquema de las variedades geogrficas del


espaol con sus caractersticas y las relaciones que existen entre algunas de ellas.

4. Qu establece la Constitucin de 1978 respecto de la


situacin lingstica de Espaa?

10. Redacta un texto expositivo sobre la importancia del


respeto a la pluralidad lingstica y cultural de Espaa. 55 3.
La realidad plurilinge de Espaa

ACTIVIDADES

REFUERZO

1. El anlisis fnico
El nivel fnico de estudio de la lengua se ocupa de las unidades relacionadas con el aspecto articulatorio y auditivo, distinguindose dos disciplinas, la fontica y la fonologa. Lee el siguiente texto y responde las cuestiones que se te plantean a continuacin.

El seor prisionero
SR. TEPN.No vaya usted ahora a andar con vergenzas con nosotros. Si quiere que le soltemos las ligaduras, dganoslo. SRA. TEPN.Usted pngase lo ms cmodo que pueda. ZEPO.Bueno, si se ponen as, sultenme las ligaduras. Pero solo se lo digo por darles el gusto. SR. TEPN.Hijo, qutaselas. (ZAPO le quita las ligaduras de los pies.) SRA. TEPN.Qu, se encuentra usted mejor? ZEPO.S, sin duda. A lo mejor les estoy molestando mucho. SR. TEPN.Nada de molestarnos. Usted, considrese como en su casa. Y si quiere que le soltemos las manos, no tiene ms que pedrnoslo. ZEPO.No. Las manos, no. Es pedir demasiado SR. TEPN.Que no, hombre, que no. Ya le digo que no nos molesta en absoluto. ZEPO.Bueno entonces, destenme las manos. Pero solo para comer, eh?, que no quiero yo que me digan luego que me ofrecen el dedo y me tomo la mano entera. SR. TEPN.Nio, qutale las ligaduras de las manos. SRA. TEPN.Qu bien, con lo simptico que es el seor prisionero, vamos a pasar un buen da de campo. ZEPO.No tiene usted que decirme seor prisionero, diga prisionero a secas. SRA. TEPN.No le va a molestar? ZEPO.No, en absoluto. SR. TEPN.Desde luego hay que reconocer que es usted modesto.
Fernando ARRABAL Pic-Nic. El triciclo. El laberinto, Ctedra

1 Caracteriza la situacin planteada en este fragmento de la obra teatral de Fernando Arrabal. Argumenta tu respuesta.
MATERIAL FOTOCOPIABLE / Oxford University Press Espaa, S. A.

7 Qu diferencia los fonemas de los sonidos lingsticos? Qu disciplina lingstica se ocupa de unos y de otros? 8 Los fonemas voclicos funcionan como centro silbico. Busca en el texto ejemplos de slabas abiertas y cerradas. 9 Extrae del texto distintos tipos de diptongos y de hiatos. 10 Explica las relaciones posibles entre fonemas y grafas atendiendo a las siguientes palabras del texto. Incorpora otros ejemplos para completar tu explicacin. ahora vergenzas buen cmodo quiere ofrecen 11 Localiza en el texto ejemplos del texto de distintos tipos de palabras acentuadas e inacentuadas. 12 Extrae ahora ejemplos de palabras oxtonas, paroxtonas y proparoxtonas.

2 La seora Tepn formula dos preguntas a Zepo. Cmo es su entonacin? Compralas con otros enunciados del fragmento. 3 Realiza la transcripcin fonolgica de las dos interrogaciones que la seora Tepn plantea al prisionero. 4 Caracteriza los fonemas que intervienen en ambas teniendo en cuenta sus rasgos mnimos. 5 Seala el o los rasgos que diferencian los siguientes pares de fonemas. /i/, /u/ /i/, /e/ /i/, /a/ /p/, /b/ /p/, /t/ /p/, /f/ /r/, /l/ /b/, /m/ /m/, /n/ /s/, /x/ 6 Explica qu rasgos se tienen en cuenta para definir los fonemas voclicos. Y para definir los fonemas consonnticos?

4.

Fontica y fonologa

61

ACTIVIDADES

ORTOGRAFA

2. Uso de la h
En la lengua castellana existe una letra que no representa ningn fonema, la h. Esta grafa puede aparecer al principio de palabra (hora), en su interior (ahora) y, solo en algunas interjecciones, al final (ah!, oh!, eh!, uh!, bah!). Debes recordar que se escriben con h: Las formas de los verbos haber, hacer, hallar, hablar y habitar: hemos, habis, hacan, he hecho, hallamos, hallarn, habl, hablaba, habitaste, habitaremos. Del mismo modo, tambin llevan h las palabras derivadas de algunos de dichos verbos: deshacer, hallazgo, hablador, habitacin. Las palabras que comienzan por los diptongos ia, ie, ue y ui: hiato, hierba, huelga, huida. Si el diptongo ue est precedido de vocal, se escribe h intercalada: cacahuete. Las palabras que comienzan con las races clsicas hecto-, helio-, hema- / hemato- / hemo-, hemi, hepta, hetero-, hidra- / hidro-, higro-, hiper-, hipo-, holo-, homeo-, homo-: hectogramo, helioterapia, hemate, hematfago, hemofilia, hemiplejia, heptaedro, heterogneo, hidratacin, hidroavin, higrometra, hiperglucemia, hiponimia, holocausto, homeotermia, homogneo. Por regla general, las palabras que empiezan por histo-, hosp-, hum-, horm-, herm-, hern-, holgy hog-: historia, hospitalario, hmedo, hormign, hermoso, hernia, holgazn, hoguera. Es necesario, adems, que tengas en cuenta palabras homfonas con h y sin h: ha / a / ah; a ver / haber; habra / abra; asta / hasta; aya / haya / halla; desecho / deshecho; echo / hecho; errar / herrar; ojear / hojear; ola / hola; onda / honda; ora / hora; reusar / rehusar, as como palabras parnimas como hay / ah / ay.

1 Explica el significado de las races clsicas citadas en la informacin superior, as como el de los ejemplos dados. hecto-: cien veces. 2 Construye oraciones con las palabras homfonas y parnimas mencionadas en la informacin superior. 3 Indica si estn correctamente escritas las siguientes oraciones. Si no son correctas, seala cul es el error en cada una de ellas.
a) Ah ay un error: lo descubr al ojear la

revista.
b) Juan se ha hechado novia; haber cundo

haces lo mismo.
c) Hallad la superficie de esta figura; cuando

ayis acabado, levantad la mano.


d) Ojal haya fresas en el mercado! Nunca

las allo cuando las busco.


e) Marcos se hech a rer cuando le dije que

no lo haba hecho.
f) Cuando habri la puerta se encontr con

Mara; si lo hubiera sabido no abra vuelto.


g) ltimamente no ha habido lluvias

en esa regin; va a haber que acudir al riego artificial.


h) Hasta el da jueves la bandera debe hondear

a media hasta.

4 Incorpora la h cuando sea necesario en estas palabras. alel coete inchado armona aumar exagonal oquedad ovario alcool exibicin exortar exaltar exaustivo veemencia desauciar exuberante ueco aondar ilandera orror alcauete eniesto coercitivo anelar ovalado uelo olemos urfano orfandad ampa osamenta urtar idrulico idroterapia reabilitacin ipcrita eterosexual trasumante aijado iguera ren baa emos echo desaogar ematoma coartar almoada toalla inumacin echar de menos undir coaccin eliocntrico urao seo oler ule ilacin ediondo oloroso vado omgrafo incapi transente inspito error almoada eptgono mero retala trasumancia ueso

62 4.

Fontica y fonologa

MATERIAL FOTOCOPIABLE / Oxford University Press Espaa, S. A.

Evaluacin 4

Pareja, por Bou.

Sobre el ingeniero Pearson


As que usted es francs, eh? insisti la seora. En efecto. Soy de Pars. Nadie lo dira, oyndole hablar. Su castellano es perfecto. Dnde lo aprendi? Mi madre era espaola. Siempre me habl en espaol, de modo que puede decirse que aprend el espaol desde la cuna, incluso antes que el francs. Qu bien, verdad? A m me gustan los extranjeros. Son muy interesantes, cuentan cosas nuevas y distintas de las que omos cada da. Nosotros siempre estamos hablando de lo mismo. Y es natural, digo yo, eh? Vivimos en el mismo lugar, vemos a la misma gente y leemos los mismos peridicos. Por eso debe de ser que discutimos siempre: por no tener nada de qu hablar. En cambio con los extranjeros no hace falta discutir: ellos cuentan sus cosas y nosotros las nuestras. Yo me llevo mejor con los extranjeros que con los de aqu. Estoy seguro de que usted se lleva bien con todo el mundo. Ca, no lo crea. Soy muy gruona. Con los aos el carcter tambin se deteriora. Todo va de baja. Pero, hablando de extranjeros, dgame una cosa, conoci usted al ingeniero Pearson? Fred Stark Pearson? No, no le conoca, aunque o hablar de l con gran frecuencia. Era una gran persona, ya lo creo! Muy amigo de mi difunto esposo, que en gloria est.
Eduardo MENDOZA La verdad sobre el caso Savolta, Seix Barral

1. Resume el contenido de este fragmento de Eduardo


Mendoza. A qu tipo de texto corresponde segn su intencin comunicativa?

6. Reconoce los grupos fnicos de este otro enunciado


y adjudica a cada uno su tonema correspondiente. Vivimos en el mismo lugar, vemos a la misma gente y leemos los mismos peridicos.

2. Reconoce en el texto los elementos del proceso comunicativo y seala las funciones del lenguaje presentes en los enunciados.

3. Transcribe fonolgicamente el enunciado del recuadro,


incluyendo los acentos y la entonacin. Conoci usted al ingeniero?

7. Explica la relacin que se establece entre el fonema /x/


y sus grafas, ejemplificando con palabras del texto.

8. Qu rasgos distintivos comparten los siguientes pares


de fonemas? Por cules se oponen? /e/, /o/ /n/, /c/ /r/, /r /

4. Enumera los ragos distintivos de cada uno de los fonemas


que intervienen en tu transcripcin.

9. Localiza en el fragmento casos de palabras con hiatos


y diptongos (crecientes, decrecientes y mixtos).

5. Si el tonema de la actividad 3 fuera el contrario, qu


cambio se producira en el enunciado?

10. Redacta un texto explicativo sobre la produccin de los


sonidos lingsticos. Cuida la ortografa y la presentacin. 67 4.
Fontica y fonologa

ACTIVIDADES

REFUERZO

1. Morfologa flexiva
La morfologa flexiva se ocupa de los accidentes gramaticales de las palabras variables (sustantivos, adjetivos y verbos). Repasa los conceptos de tema y de morfemas flexivos (genricos y numricos; vocales temticas, caractersticas y desinencias), y despus contesta las cuestiones que se plantean sobre el texto.

El reloj
Cuando Curtis va por el Cantn, ve ya con toda claridad las nubes cenicientas, espesas, terrosas, como el vaho del rescoldo del fuego rumiante, que forman las humaredas de la Drsena. Cubierto tambin el cielo hacia la parte de Mara Pita. Sabe que ya no podr dar marcha atrs. Tiene que seguir adelante para verlo con sus propios ojos. Mira el reloj en lo alto del Obelisco. Recuerda: Su majestad, la Hora! Pero parece que lleva ah desde siempre, que las agujas no han completado an la vuelta a la esfera, que marca siglos. Sada tena razn. Debera ser un reloj de cuco. Si ahora saliese un cuco, pens Cutis, quiz todo sera distinto. Alzara la frente de los que caminan inquietos, contando interrogantes en las lozas del suelo como quien pone sus pasos en cuadrculas de ajedrez.Tal vez descompondra el andar marcial de otros, que trazan una lnea recta. Quiz el cuco detendra por un instante al joven uniformado con ese gorro de cresta colorada que parece galopar sobre las ancas de una lnea recta. Tuvo la impresin de verlo a l, a Sada, cuando pas por la plaza de Pontevedra, y all donde estaban empezando a formar las tropas de reclutamiento. El ejrcito golpista se haba impuesto en la ciudad y dominaba Galicia, que iba a ser uno de los territorios de retaguardia para lo que los sublevados denominaban la nueva reconquista de Espaa. S, le pareci que Sada estaba all. Tan alto, le resultaba difcil pasar desapercibido. Le pareci tambin que haba otros rostros conocidos, aunque en aquellos das no solo haba cambiado el humor, sino el rostro de las gentes, su presencia, sus rasgos fsicos.
Manuel RIVAS Los libros arden mal, Punto de Lectura

1 Extrae todos los sustantivos comunes y los adjetivos calificativos del texto.
MATERIAL FOTOCOPIABLE / Oxford University Press Espaa, S. A.

6 Extrae ahora las formas verbales personales simples, agrupndolas segn el tiempo verbal al que pertenezcan. 7 Atendiendo a su base lxica, di cules corresponden a verbos regulares e irregulares. En el caso de los irregulares, seala los distintos alomorfos del verbo y explica los tipos de variaciones que presenten. 8 Indica los morfemas flexivos (vocal temtica, caracterstica y desinencia) de todas las formas verbales del fragmento. 9 Qu desinencias no estn presentes en las formas verbales del texto? Brinda ejemplos de ellas conjugando verbos que figuren en tu lista, y selalas. 10 Reconoces algn verbo polirrizo en el fragmento? En caso afirmativo, seala sus distintas races.

2 Seala en ambas clases de palabras sus temas y los morfemas flexivos genricos y numricos que presenten. Indica si los temas son simples o complejos. 3 Reconoce entre los sustantivos que has sealado en la actividad 1, uno que sea comn en cuanto al gnero. 4 Escribe ejemplos de sustantivos epicenos, de sustantivos ambiguos y de sustantivos heternimos. 5 Escribe ahora las formas verbales no personales que encuentres en el texto, diferenciando su base lxica y sus morfemas flexivos correspondientes (vocal temtica y caracterstica).

5.

Morfologa

73

ACTIVIDADES

DOCUMENTACIN

2. Modelo de constituyentes
Los siguientes cuadros recogen las formas simples de los distintos temas verbales. En cada una se ha destacado la vocal temtica en negrita, la caracterstica en cursiva y la desinencia subrayada; el resto es el lexema o raz verbal.

TEMA DE PRESENTE Imperativo 1. conj. estudi-a estudi-a-d 2. conj. tem-e tem-e-d 3. conj. part-e part-i-d 1. conj. estudi-o estudi-a-s estudi-a estudi-a-mos estudi--is estudi-a-n Presente de indicativo 2. conj. tem-o tem-e-s tem-e tem-e-mos tem--is tem-e-n

TEMA DE PRESENTE Presente de subjuntivo 1. conj. estudi-e estudi-e-s estudi-e estudi-e-mos estudi--is estudi-e-n 2. conj. tem-a tem-a-s tem-a tem-a-mos tem--is tem-a-n 3. conj. part-a part-a-s part-a part-a-mos part--is part-a-n 3. conj. part-o part-e-s part-e part-i-mos part-s part-e-n

TEMA DE PRETRITO Participio 1. conj. estudi-a-do 1. conj. estudi-a-ndo 2. conj. tem-i-do Gerundio 2. conj. tem-ie-ndo 3. conj. part-ie-ndo 3. conj. part-i-do 1. conj. estudi-a-ba estudi-a-ba-s estudi-a-ba estudi-a-ba-is estudi-a-ba-n 1. conj. estudi-a-ra/-se estudi-a-ra/-se 2. conj. tem--a tem--a-s tem--a tem--a-is tem--a-n 2. conj. tem-ie-ra/-se tem-ie-ra/-se

TEMA DE PRETRITO Pretrito imperfecto de indicativo 3. conj. part--a part--a-s part--a part--a-mos part--a-is part--a-n 3. conj. part-ie-ra/-se part-ie-ra/-se-s part-ie-ra/-se Pretrito perfecto simple de indicativo 1. conj. estudi- estudi-a-ste estudi- estudi-a-mos estudi-a-ste-is estudi-a-ro-n 1. conj. estudi-a-re estudi-a-re-s estudi-a-re 2. conj. tem- tem-i-ste tem-i tem-i-mos tem-i-ste-is tem-ie-ro-n 2. conj. tem-ie-re tem-ie-re-s tem-ie-re tem-i-re-mos tem-ie-re-is tem-ie-re-n 3. conj. part- part-i-ste part-i part-i-mos part-i-ste-is part-ie-ro-n 3. conj. part-ie-re part-ie-re-s part-ie-re part-ie-re-is part-ie-re-n
MATERIAL FOTOCOPIABLE / Oxford University Press Espaa, S. A.

estudi--ba-mos tem--a-mos

Pretrito imperfecto de subjuntivo

Futuro de subjuntivo

estudi-a-ra/-se-s tem-ie-ra/-se-s

estudi--ra/-se-mos tem-i-ra/-se-mos part-i-ra/-se-mos estudi--re-mos estudi-a-ra/-se-is tem-ie-ra/-se-is part-ie-ra/-se-is estudi-a-re-is estudi-a-ra/-se-n tem-ie-ra/-se-n part-ie-ra/-se-n estudi-a-re-n

part-i-re-mos

TEMA DE FUTURO Infinitivo 1.a conj. estudi-a-r 2.a conj. tem-e-r 3.a conj. part-i-r 1. conj. estudi-a-r estudi-a-r-s estudi-a-r estudi-a-r-is estudi-a-r-n Futuro de indicativo 2. conj. tem-e-r tem-e-r-s tem-e-r tem-e-r-is tem-e-r-n 3. conj.

TEMA DE FUTURO Condicional 1. conj. estudi-a-ra estudi-a-ra-s estudi-a-ra estudi-a-ra-is estudi-a-ra-n 2. conj. tem-e-ra tem-e-ra-s tem-e-ra tem-e-ra-is tem-e-ra-n 3. conj. part-i-ra part-i-ra-s part-i-ra part-i-ra-mos part-i-ra-is part-i-ra-n

part-i-r part-i-r-s part-i-r part-i-re-mos part-i-r-is part-i-r-n

estudi-a-re-mos tem-e-re-mos

estudi-a-ra-mos tem-e-ra-mos

74 5.

Morfologa

ACTIVIDADES

REFUERZO

3. Morfologa lxica
La morfologa lxica se ocupa de los procedimientos de formacin de palabras. Repasa cada uno de estos procedimientos y responde despus a las cuestiones que se plantean sobre el siguiente fragmento.

En la cafetera
Grupos bulliciosos de jvenes se arracimaban, charlando y fumando, ante la barra de la cafetera, en un hervor humano, confuso y excitante. Por el suelo se entremezclaban desperdicios de marisco, huesos de aceitunas, puntas de cigarrillos, envolturas de azcar y servilletas de papel arrugadas. Vctor se situ en un pequeo hueco, en el extremo de la barra, junto a la caja. La muchacha ms vistosa una rubia de brazos pecosos y sonrosados de las cuatro que atendan el mostrador, se dirigi sonriente a Vctor al divisarle: Un vinito? pregunt. Un vinito, vale dijo Vctor. Puso un vaso en la barra, cogi una botella de la estantera y le sirvi: De viaje otra vez? Qu remedio! Siempre de viaje. Cmo marchan las cosas? Marchan, que no es poco. Por la puerta de cristales abierta entraba un vaho de humedad, pues apenas haban transcurrido cinco minutos desde el ltimo chaparrn. En las aceras, hmedas, se vean centenares de octavillas de colores, embarradas, pegadas al suelo. Por la calzada, pas un coche con un altavoz estridente, pero iba tan rpido que apenas pudo escucharse el comienzo de la alocucin antes de que sus voces fueran sofocadas por el rumor del resto de los automviles que circulaban por la amplia avenida.
Miguel DELIBES El disputado voto del seor Cayo, Crculo de Lectores

MATERIAL FOTOCOPIABLE / Oxford University Press Espaa, S. A.

1 En este fragmento de Miguel Delibes pueden localizarse tres de los procedimientos de formacin de palabras. Indica de cules se trata y explica en qu consisten a partir de ejemplos extrados del texto. 2 Qu procedimientos de formacin de palabras no aparecen en el pasaje? Defnelos y escribe algunos ejemplos. 3 Seala en el texto diez palabras en las que solo aparezcan morfemas lxicos y morfemas flexivos, y analiza su estructura interna. 4 Enumera las palabras derivadas del texto, explica su formacin y analiza su estructura interna.

5 Localiza en estas lneas de Delibes una palabra compuesta, indica cmo se ha formado y analzala morfolgicamente. 6 Aparecen palabras parasintticas en el primer prrafo del texto? Y en el ltimo? En caso afirmativo, realiza un anlisis morfolgico similar al planteado en las actividades 4 y 5 de esta pgina. 7 Al inicio del dilogo entre Vctor y la muchacha rubia del mostrador se da un caso de sufijacin apreciativa. Explica con qu valor se utiliza este procedimiento y escribe otros ejemplos con el mismo sufijo en los que este adquiera valores diferentes.

5.

Morfologa

75

ACTIVIDADES

ORTOGRAFA

4. Uso de b y v
El fonema /b/, bilabial sonoro, adopta la representacin grfica de b y v, lo que provoca muchos problemas ortogrficos. Para evitarlos te ser de gran ayuda recordar algunas reglas.

Se escriben con b: Los verbos beber, caber, deber, haber y saber: bebi, cabremos, debera, hubo, saben. Los verbos terminados en -buir y -bir, a excepcin de hervir, servir, vivir y sus compuestos: distribuirn, atribu, retribuir, escribiremos, recibi; pero hervimos, sirven, revivi. La caracterstica -ba- del pretrito imperfecto de indicativo: iba, cantaban, rezabais. Las palabras que contienen los prefijos bi-, bis-, biz- y los elementos compositivos biblio- , bio- / -bio, bien- / bene-: biangular, bisnieto, bizcocho, biblioteca, biologa, microbio, bienvenido, beneplcito. Las palabras que comiencen con las slabas bu-, bury bus-: buceador, burgus, buscar. Las palabras que acaban en -bundo / a y en -bilidad, a excepcin de movilidad, civilidad (ya que derivan de los adjetivos mvil y civil, respectivamente) y sus compuestos: vagabundo, amabilidad, inmovilidad. Las palabras en las que el fonema /b/ precede a otra consonante o est en final de palabra: abdicar, obtuso, absolucin, nabab.

Se escriben con v: Las palabras que empiezan con eva- (con excepcin de bano), eve-, evi- y evo-: evadir, evento, evitar, evolucin. Los adjetivos que acaban en -avo / a, -evo / a, -eve, -ivo / a, y las palabras derivadas de algunos de ellos: esclavo, esclavitud, esclavismo, nueva, novedad, leve, pasiva. Las palabras que empiecen con el prefijo vice- / viz- / vi-: vicedirector, vizconde, virrey. Las palabras que acaben en -voro / a, con excepcin de vbora: carnvoro, insectvora. Los verbos acabados en -olver: volvers, disolvieron. Los presentes de indicativo y subjuntivo y la segunda persona del singular de imperativo del verbo ir: vas, vayamos, ve. El pretrito perfecto simple del indicativo y el pretrito imperfecto y futuro del subjuntivo de los verbos andar, estar, tener y sus derivados: anduvimos, anduvieras, desanduviere, estuvo, retuviera, sostuviramos, tuve, retuviera, sostuviere, mantuviramos. Las palabras que contengan el fonema /b/ cuando va precedido por las slabas ad-, sub- y ob-: adversario, subversivo, obvio.

1 Coloca b o v en las siguientes palabras segn corresponda: reci__iremos __rtigo con__encer __ra__ucn acti__idad a__sol__er contri__uyente __ulgaridad en__idiosa __aga__undo retu__ieron permea__ilidad ca____amos go__ierno __l__ula ca__idad __urladero sucum__ieron __iscosidad con__irtieron __re__iario disol__ente o__iedad atri__uido a__ismo fri__olidad __est__ulo mantu__iste e__ita__le la__a__an coha__itacin __enda__al contu__ieron micro__io hir__iente __olumen con__idaste o__jeti__as decisi__a le__edad ca__ida com__inacin par__ulario __islum__rar contu__ieran omn__ora i__ais apro__aron hu__ieron __isla__as __iena__enturado sir__iente __olu__le con__ocatoria o__ser__aron a__sol__ieron ad__er__ial ca__aron de__ilitar __o__ino __oca__ulario a__iertas her____oras __ipolares con__alidar a__o__edada longe__idad a__negado __i__ienda __oracidad con__ulsin su__junti__o __ayan ci__ismo sa__an esta__le equ__oco re__erso gra__amen estu__imos

76 5.

Morfologa

MATERIAL FOTOCOPIABLE / Oxford University Press Espaa, S. A.

Evaluacin 5
El cambio climtico
Algunas consecuencias del cambio climtico son ya irreversibles, pero si actuamos con rapidez y firmeza podemos frenar el aumento y conseguir que las temperaturas no suban ms all de los dos grados. Si no actuamos y continuamos al ritmo actual, las temperaturas podran subir ms de seis grados centgrados durante este siglo, aumentando la magnitud de las consecuencias. Las inundaciones aumentarn dramticamente al producirse cada vez ms tormentas violentas y lluvias torrenciales. Los glaciares del planeta se derretirn a una velocidad incluso mayor de a la que ya lo hacen, provocando un aumento de agua en el mar y cambiando probablemente corrientes marinas, como la del Golfo de Mxico. La subida del nivel del mar es una de las principales amenazas de zonas costeras e islas, y afectar ms a los pases ms pobres. En las costas espaolas, para finales de este siglo se prev un aumento del nivel de mar de 50 centmetros a un metro; podr causar prdidas de un nmero importante de playas, sobre todo en el Cantbrico, e inundaciones de las zonas bajas costeras. A la vez que los mares recibirn ms agua de la fusin de los polos, los ros se secarn en muchas partes del mundo poniendo seriamente en peligro los suministros acuferos. Las sequas estn siendo cada vez ms frecuentes, por ejemplo en frica, Asia y la regin del Mediterrneo. Millones de personas se ven amenazadas por el hambre, especialmente en los pases ms pobres y se espera que se recrudezca en las prximas dcadas.

Playa de Acech, en el norte de Sumatra, el 26 de diciembre de 2004, despus del tsunami.

La extincin de especies se acelerar debido a la dificultad de animales, plantas y ecosistemas para adaptarse a los rpidos cambios climticos. Estos peligros son realmente serios para la fauna y la flora de los arrecifes de coral, bosques, sabanas, regiones polares y cordilleras montaosas. La comunidad cientfica teme que para 2050 se haya extinguido un tercio de todas las especies existentes.
Asensio RODRGUEZ Revista Greenpeace, n. 1, ao 2007

1. Resume el contenido del texto El cambio climtico,


publicado en la Revista Greenpeace.

6. Indica la categora gramatical de las palabras derivadas


del recuadro y la base de la que proceden. Ejemplo climtico: adjetivo denominal irreversibles inundaciones probablemente costeras velocidad existentes rapidez montaosas sequas

2. Reconoce en el fragmento los elementos del proceso de


la comunicacin. De qu tipo de texto se trata, segn su intencin comunicativa?

3. Reconoce en las siguientes palabras los morfemas flexivos y el tema. consecuencias marinas acuferos grados amenazas montaosas inundaciones bajas climticos

7. Analiza morfolgicamente las palabras ecosistemas,


dramticamente y acuferos. Indica por qu procedimiento / s se ha formado cada una y seala sus morfemas, indicando el significado lxico y / o gramatical.

4. Distingue el morfema lxico raz y los morfemas flexivos


en las siguientes formas verbales, indicando el valor de cada uno de ellos. son actuamos frenar podran aumentando aumentarn hacen acelerar extinguido

8. En el texto aparece la palabra recrudezca, que es una


forma flexionada del verbo recrudecer, que se conjuga de manera anloga a reblandecer. Indica su proce dimiento de formacin.

9. En qu consisten las siglas y los acortamientos? Pon


ejemplos de ambos procedimientos.

5. Qu verbos de la actividad 4 son irregulares? Seala los


alomorfos de cada uno y explica su variacin.

10. Redacta un texto sobre la composicin: concepto, tipos,


estructuras ms frecuentes. No olvides brindar ejemplos. 85 5.
Morfologa

ACTIVIDADES

REFUERZO

6
A continuacin, se ofrecen dos textos del escritor argentino Jorge Luis Borges sobre los que debes resolver las cuestiones sintcticas que se proponen luego.

1. Construcciones sintcticas

El amenazado
Es el amor. Tendr que ocultarme o que huir. Crecen los muros de su crcel, como en un sueo atroz. La hermosa mscara ha cambiado, pero como siempre es la nica. De qu me servirn mis talismanes: el ejercicio de las letras, la vaga erudicin, el aprendizaje de las palabras que us el spero Norte para cantar sus mares y sus espadas, la serena amistad, las galeras de la Biblioteca, las cosas comunes, los hbitos, el joven amor de mi madre, la sombra militar de mis muertos, la noche intemporal, el sabor del sueo? Estar contigo o no estar contigo es la medida de mi tiempo. Ya el cntaro se quiebra sobre la fuente, ya el hombre se levanta a la voz del ave, ya se han oscurecido los que miran por las ventanas, pero la sombra no ha trado la paz. Es, ya lo s, el amor: la ansiedad y el alivio de or tu voz, la espera y la memoria, el horror de vivir en lo sucesivo. Es el amor con sus mitologas, con sus pequeas magias intiles. Hay una esquina por la que no me atrevo a pasar. Ya los ejrcitos me cercan, las hordas. (Esta habitacin es irreal; ella no la ha visto.) El nombre de una mujer me delata. Me duele una mujer en todo el cuerpo.

A un gato
No son ms silenciosos los espejos ni ms furtiva el alba aventurera; eres, bajo la luna, esa pantera que nos es dado divisar de lejos. Por obra indescifrable de un decreto divino, te buscamos vanamente; ms remoto que el Ganges y el poniente, tuya es la soledad, tuyo el secreto. Tu lomo condesciende a la morosa caricia de mi mano. Has admitido, desde esa eternidad que ya es olvido,
MATERIAL FOTOCOPIABLE / Oxford University Press Espaa, S. A.

10

el amor de la mano recelosa. En otro tiempo ests. Eres el dueo de un mbito cerrado como un sueo.
Jorge LuiS BORGES Obras completas, Emec

1 Extrae los sintagmas nominales de los dos textos de Jorge Luis Borges y analiza sus estructuras. 2 Cules de dichos sintagmas funcionan como sujeto? 3 Reconoce en los textos otros tipos de sintagmas y de construcciones.

4 Reconoce y elabora un listado con las oraciones simples de ambos textos. 5 Explica la estructura jerrquica que establecen en cada una de ellas sus constituyentes. 6 Brinda ejemplos en las oraciones simples de los textos que sean constituyentes, pero no construcciones.

6.

Sintaxis (I). La oracin simple

87

ACTIVIDADES

REFUERZO

6
En los titulares de los peridicos se pretende condensar el contenido de la informacin de una noticia o de un artculo de opinin empleando pocas palabras. En ellos son frecuentes tanto las oraciones simples como los enunciados no oracionales. Obsrvalo en esta seleccin de titulares extrados de un peridico de tirada nacional.

2. Enunciados no oracionales y oracionales en el periodismo

Grammy al mejor disco de pop latino

Columbus se acopla con xito a la estacin espacial

Los lderes de Hams pasan a la clandestinidad por temor a un ataque

Madrid exige a Londres un sistema tripartito de vigilancia

Integrados sin derecho a voto

Las dudas sobre los creditos de AIG causan otra jornada bajista en el sector financiero

Escalofriante arresto en Bagdad

Nos esperan tres semanas duras

Prisin para el detenido por la cada de su novia

Cuatro facultades de Medicina ms en 2008

La Espaa igual y la plurinacional

7 000 firmas antiaeropuerto en Getafe


Vestidas con destellos de opulencia

La cartelera se queda en cueros

Muere un hombre en un banco en Getafe

Un fuego reduce a cenizas la Puerta de Namdaemun, el principal tesoro nacional de Corea del Sur

Sonrisas y jbilo bajo una intensa tromba de agua


La extraa dualidad entre EE UU y Europa

El adis ms refinado al rigor invernal

EE UU detiene a cuatro personas por espionaje industrial y militar para China

La venta de pisos de Metrovacesa para los prximos dos aos cae un 60%

Fusiones a la vista en la banca y el sector de medios de comunicacin

Microsoft corteja a los accionistas de Yahoo tras el rechazo de su OPA

El reptil volador ms diminuto

1 Confecciona una tabla y clasifica en ella los enunciados oracionales y los no oracionales que aparecen en estos titulares. 2 Analiza la estructura de los enunciados no oracionales y seala de qu tipo de construccin se trata en cada caso.

3 Reconoce el sujeto y el predicado de las estructuras oracionales atendiendo a la concordancia. 4 Analiza en cada oracin los constituyentes de los sujetos expresos y de los predicados. (Recuerda las tcnicas de reconocimiento de los distintos constituyentes.)

88 6.

Sintaxis (I). La oracin simple

MATERIAL FOTOCOPIABLE / Oxford University Press Espaa, S. A.

Una reforma oportuna de la Ley del menor

ACTIVIDADES

REFUERZO

6
El lenguaje literario brinda enunciados oracionales construidos cuidando la seleccin y la combinacin de las palabras que los configuran.

3. Oraciones simples en el lenguaje literario

1 Realiza el anlisis sintctico de las siguientes oraciones simples extradas de poemas de Juan Ramn Jimnez.
a) En la calma magnfica del parque, reson

r) Pero, acaso puede hablar de sus rosales

el beso con un eco largo.


b) Un ruiseor despierto lanz un dulce quejido

desgarrado.
c) Las avenidas se alargan entre la incierta

penumbra de la arboleda lejana.


d) Y en la onda transparente del cenit verdoso,

vagan misticismos de suspiro y perfume de plegarias.


e) En el balcn, un instante nos quedamos

los dos.
f) Desde la dulce maana de aquel da, ramos

novios.
g) El paisaje sooliento dorma sus vagos tonos,

bajo el crepsculo gris y rosa del crepsculo de otoo.


h) Mi alma es hermana del cielo gris y de las

hojas secas.
i) Los rboles del jardn estn cargados

de niebla.
j) El valle se queda ms solo y lbrego.
MATERIAL FOTOCOPIABLE / Oxford University Press Espaa, S. A.

k) La otra tarde se ha llevado el viento ms

hojas secas!
l) Qu pena tendrn los rboles, esta noche

sin estrellas!
m) La luna camina muerta, sin luz de besos

ni lgrimas, amarilla entre la niebla.


n) Y la noche, tibia, serena y callada, dormir

el mundo, a los rayos de su luna solitaria.


) Y, de repente, una voz melanclica

y distante, ha temblado sobre el agua, en el silencio del aire.


o) Luz y cristal su voz era en el surco removido. p) Qu difcil es lo fcil! q) Tu corazn y mi alma yerran solos por

la sombra de esta larga tarde azul, tarde doliente de aromas.

un corazn sepulcrado? s) Negro sobre el cielo liso, revolotea un murcilago. t) Por las verjas se ve luz en las casas. u) Los montes, con el confuso pinar de la soledad, parecen de los difuntos. v) En la paz del campo van dejando los troncos muertos un olor fresco y honrado a corazn descubierto. w) Los caminos de la tarde se hacen uno, con la noche. x) Por encima de los oscuros tejados, verde, lloroso de grillos y de esquilas, est el campo. y) El cielo le da lagunas azules, suaves caadas, llenas de nveos rosales y de abrigadas cabaas. z) Como una rueda visible del cielo, la luna roja va acarreando la noche sobre la campia sola. aa) Campana de Francia, lloras por mis amadas de Espaa? bb) La armona romntica del poniente de oro va resbalando sobre el ro vespertino cc) Huele a rosas pisadas. dd) Libra de sus penosas dudas a este constante asedio de mis penas. ee) El luto duro y refrescante de la hora hablaba a la ilusin de un imprevisto ambiente. ff) Un agudo cuchillo de luz agria y equvoca orna el medroso instante de un extrao esplendor, delirante y amarillo. gg) Dnde se han escondido los colores en este da negro y blanco? hh) Nos quedbamos solos en la hora serena. ii) La muerte no sabr de nuestra dulce alma. jj) Ya el rbol se ha resignado a su sereno morir. kk) Las flores azules huelen a imposible, entre dulces luces. ll) El celeste se torna azul sonoro. mm) Las estrellas empiezan a contemplar el mundo.

6.

Sintaxis (I). La oracin simple

89

ACTIVIDADES

REFUERZO

6
La publicidad hace uso de distintos tipos de estructuras sintcticas que atraigan la atencin de los receptores, los cuales combina con imgenes. El lenguaje verbal complementa, aclara o, incluso, contradice lo que muestra el anuncio.

4. Anlisis sintctico de anuncios publicitarios

El 70% de la superficie de la tierra es agua. Disfrtelo.

El Crculo OLAY lo hace posible.

Sonra. Tenemos grandes soluciones para su piel por solo 7 euros.

Nunca un tratamiento ha aportado tanto a la piel de una mujer: una nueva vida, un nuevo volumen, una nueva edad.

Con Braun la suavidad se hace irresistible. Acrcate a la suavidad.

Concede a tu pelo tres deseos.

Entre otras muchas cosas, aparca solo.

Dale color al momento.

Disfruta de un consumo responsable.

Solo en algunas ocasiones, unas memorias se convierten en un relato tan apasionante e indispensable.

Llena de fragancia toda la habitacin.

Qu le pides a tu seguro del hogar?

Todos tenemos recetas de familia. Cul es la tuya?

La vida se disfruta ms en compaa.

Zaragoza 2008, la Expo del agua y del desarrollo sostenible, cuenta con el apoyo de Bob Dylan.

Se acabaron las carreteras aburridas.

Dale un buen mordisco a la vida.

Qu parte de mi cuerpo te gusta ms?

Cuenta con Lizipaina

El plan Activia 15 das tambin es para ti.

Olvdate de la caspa.

Actimel ayuda a las defensas en pocas de sobreesfuerzo.

1 Reconoce en los enunciados el sujeto y el predicado, aplicando la regla de la concordancia. 2 Si el sujeto es explcito, analzalo internamente, estableciendo sus elementos constitutivos. 3 Analiza ahora el predicado, aplicando las tcnicas de reconocimiento de los distintos complementos verbales.

4 En cada predicado, diferencia complementos argumentales y no argumentales. 5 Identifica en los anuncios las perfrasis verbales y explica cul es su constitucin. 6 Indica el valor temporal o la modalidad subjetiva de las perfrasis que has localizado en la actividad 5.

90 6.

Sintaxis (I). La oracin simple

MATERIAL FOTOCOPIABLE / Oxford University Press Espaa, S. A.

Ahora con Ariel puedes conseguir una limpieza ms brillante.

Dentro de un Suzuki la vida se ve de otra manera.

Los caprichos de los grumetes ahora puedes hacerlos al horno.

ACTIVIDADES

ORTOGRAFA

5. Uso de g, gu, g y j
Recuerda: Las slabas /ga/, /go/ y /gu/ se escriben con g a / o / u: gato, goloso, gula. Las slabas /ge/ y /gi/ se escriben con gu e / i: guerra, guillotina. Las slabas /gue/ y /gui/ se escriben con g e / i: cigea, lingstica. Las slabas /xa/, /xo/ y /xu/ se escriben con j a / o / u: jalar, joya, jugar. Las slabas /xe/ y /xi/ se escriben en unos casos con g e / i y en otros con j e / i: general, jefe, gitano, jirafa.

1 Coloca g o gu segn corresponda. a) errero d) a obiante b) ila e) si ientes d) m ico f) itarra 2 Enuncia la regla por la que has optado por una u otra grafa en el ejercicio anterior. 3 Coloca diresis donde corresponda. a) pinguino e) ceguera b) cigueal f) linguista c) liguero g) desague d) halagueo h) aguita 4 Enuncia la regla por la que has colocado la diresis en algunas palabras de la actividad 3. 5 Coloca g o j, segn corresponda, en las palabras siguientes. eografa di eras introdu e irafa conser e fonolo a fin aborda e pea e contradi iste
MATERIAL FOTOCOPIABLE / Oxford University Press Espaa, S. A.

salva e mar en neolo ismo pere il parad ico bendi e maldi iste enco en gran ero estiones au e contra e infrin ir ma estad ter iversar transi ir ara e exi irn verti inosa ba eza re a

co ear enial ma ia re ional il uero ind ena sur en te i aprendiza e di estivo corre ir fu itivo o iva r ida intan ible vi encia con etura exi ente can ear li ereza morfolo a

apo eo eometra eringa cerra ero ersey fin ir ho ear aco ida su erencia a encia efi ie su ecin he emona nostal ia inete homena ear produ e cru iente le islacin ena enar in eniosa

eol ica tri simo presti ioso peda ico antolo a fin o reco i esco eremos condu e in erencia tra n pr imo mu ido irn erarquizar prote er emer ieron il tra edia pela e masa e

dedu imos exi ente infli ir mensa ero lison era in ertar ele iste cru ir ener a i ante extran ero

6 Despus de corregir esta ltima relacin de palabras, escribe nuevamente aquellas en que te hayas equivocado y forma voces derivadas de las mismas cuando sea posible. En los casos en que se trate de una forma verbal, escribe, adems, otras de su conjugacin con el mismo lexema.

6.

Sintaxis (I). La oracin simple

91

Evaluacin 6
El obispo
Una dbil claridad aparece en las altas vidrieras de la catedral. Es la hora del alba. A esta hora baja el obispo a la catedral. El palacio del obispo est unido a la catedral por un pasadizo que atraviesa la calle. A la hora en que el obispo entra en la catedral todo reposa en la pequea ciudad. La catedral est casi a oscuras; resuenan, de cuando en cuando, unos pasos; chirra el quicio de una reja. En la pequea ciudad la luz de la maana va esclareciendo las callejas. Se ve ya, en la plaza que hay frente a la catedral, caer el chorro del agua en la taza de la fuente; el ruido de esta agua, que haba estado percibindose toda la noche, ha cesado ya. El obispo est ciego, ciego como el dulce y santo obispo francs Gastn Adrin de Sgur. Entra en la catedral despacito, va sostenindose en un cayado; [ ] le van siguiendo dos familiares. La amplia capa cae en pliegues majestuosos hasta las losas. Se dirige el buen prelado hacia la capilla del maestre don Ramiro. De cuando en cuando se detiene, apoyado en su bastn, con la cabeza baja, como meditando. Su pelo es abundante y blanqusimo. Destaca su noble cabeza en el vivo morado de las ropas talares1. No puede ya ver el obispo su catedral, ni su ciudad. Pero desde su cuartito, l, todas las maanas, a la hora en que rompe el alba, espa todos los ruidos de la ciudad, que renace a la vida: el canto de un gallo, el tintn de una herrera, el grito de un vendedor, el ruido de los pasos.
Jos MARTNEZ RUIZ, AZORN Don Juan, Espasa Calpe Interior de la catedral de Mnster, por Lorenz Ritter.
1

talar: que llega hasta los talones.

1. Caracteriza el fragmento de Azorn atendiendo a los


siguientes aspectos: Intencin comunicativa. Modalidades del discurso.

7. Enuncia los argumentos exigidos por los siguientes verbos


del texto. aparecer ver ser cesar bajar dirigirse reposar espiar

2. Las siguientes construcciones han sido extradas del


texto. Indica de qu tipo es cada una de ellas, analiza sus constituyentes y establece la estructura jerrquica. a) En la pequea ciudad. b) El quicio de una reja. c) Frente a la catedral. d) En la taza de la fuente. e) Espa todos los ruidos de la ciudad.

8. Realiza el anlisis sintctico de las siguientes oraciones,


construidas a partir del texto. a) Un pasadizo une la catedral al palacio del obispo. b) Chirra estrepitoso el quicio de una puerta. c) Encontraba relajante el ruido del chorro del agua en la fuente. d) Todas las maanas, el obispo es seguido por dos familiares.

3. Localiza los verbos pronominales que aparecen en el


pasaje de Don Juan.

9. Escribe una oracin que responda a la estructura que se


indica en el recuadro. S (Det N Ady Ap) - P (N (Perf V) C Rg CD)

4. Seala las perfrasis verbales que aparecen en el texto,


explica su estructura e indica su valor.

5. Reconoce todas las oraciones simples que componen el


fragmento de Azorn.

10. Compara por escrito los complementos directo y de


rgimen: comienza por sus semejanzas y establece despus sus diferencias, atendiendo tanto a la estructura como al reconocimiento. Pon ejemplos de ambos. 101 6.
Sintaxis (I). La oracin simple

6. Analiza sintcticamente las oraciones que has identificado


en la actividad 5.

ACTIVIDADES

REFUERZO

1. Clases de oraciones simples


Las oraciones simples pueden clasificarse atendiendo a dos criterios: segn la actitud del hablante y segn sus relaciones sintcticas.

a) A qu hora llegarn los invitados? b) Se arrodill ante el altar. c) No ocurri nada importante. d) Los pisos superiores fueron rpidamente alcanzados por las llamas. e) No se la asusta fcilmente. f) Se ha avisado ya a la polica? g) Se sabe de memoria la leccin. h) Finalmente result ser la ganadora. i) Cada vez amanece ms temprano. j) No se puede comer todava el postre. k) Le va muy bien en sus estudios. l) Qu te falta en la maleta? m) S atenta con los invitados. n) Ojal no llueva este fin de semana. ) No se saludaron esta maana. o) En ese negocio se necesita un ayudante. p) Siempre hay mosquitos y lucirnagas por la noche junto al ro. q) Los rumores fueron divulgados por esa cadena de televisin. r) Por dnde se llega antes al pueblo? s) Se convoc a todos los alumnos. t) Ya se lo han dicho? u) Dos empleados fueron agredidos a la salida del trabajo. v) ltimamente se estudia muy poco.
MATERIAL FOTOCOPIABLE / Oxford University Press Espaa, S. A.

w) Desde nuestra ventana se puede contemplar el atardecer. x) Debis dejar limpias las aulas. y) Se trata de los estudios de su hijo.

1 Analiza sintcticamente las oraciones. 2 Clasifcalas atendiendo a la actitud del hablante. 3 En cada una de las oraciones con se, distingue el tipo de pronombre de que se trata.

4 Clasifica las oraciones atendiendo a las relaciones sintcticas: Sujeto-predicado / impersonales. Atributivas / predicativas. Transitivas / intransitivas. Pronominales / no pronominales.

7.

Sintaxis (II). Clases de oraciones. La coordinacin

115

ACTIVIDADES

REFUERZO

2. Oraciones coordinadas
Las siguientes oraciones, extradas todas ellas de una obra de Azorn, se ofrecen para practicar el anlisis sintctico de oraciones compuestas por coordinacin.

a) Parece unas veces perdida la mirada de la seora en una lontananza invisible; otras, pasa y repasa sobre el haz de las cosas a manera de silenciosa caricia. De pronto, un pensamiento triste conturba a la desconocida: la mirada se eleva y un instante resalta en lo trigueo de la faz lo blanco de los ojos. b) Desde el balcn se divisa un panorama de tejados; abajo culebrea la calle estrecha y pendiente. El piso de la estancia es de ladrillos rojos; algunas de estas losetas han perdido el barniz y se deshacen en un polvillo tenue. c) Doa Ins se levanta y se acerca a la puerta de la sala. No ha sido nada, reina el silencio. Los visillos del balcn son ladeados por la fina mano; la mirada pasea vagamente por el panorama de los tejados y baja hasta el fondo de la calle. No est intranquila la dama y no acaba de sentir perfecto sosiego. d) Doa Ins ha cogido la carta, la ha rasgado en cien pedazos y ha abierto el balcn. La mano fuera del balcn lanzaba los cien pedacitos de papel blanco. Los mltiples pedacitos de papel caan, volaban, revoloteaban en la luz penumbrosa del crepsculo, y una encendida estrella rutilaba en el cielo difano. e) Unos ojos negros tienen destellos de bondad, unas veces; otras, miran de hito en hito y misteriosos. Y unos brazos se levantan y dejan recortado en el fondo indefinido un busto firme y esbelto. f) En la ciudad todo se desenvuelve automticamente; todo obedece a la luminosidad de la hora y de la estacin. El sol con su luz viva suscita el bullicio y el estruendo de los moradores. Va declinando la viva luz solar: el estrpito y el trfago se van amortiguando. As, al par de la carrera del sol por el firmamento, crece o decrece en la ciudad el oleaje del tumulto y de los mil ruidos. g) Ya le dan de vestir a la seora; el palacio bulle de gente; ya le trae una camarera el blanco brial; en el palacio los caballeros y las damas van y vienen por corredores y galeras. Ya una camarera le ajusta el corpio a la dama, y otra la baa con aguas de olor. En el palacio los juglares ren y chancean con los caballeros. h) Doa Beatriz estaba muy triste. Abatida andaba por el palacio; sus camareras la miraban con melancola; no se hablaba nada en la cmara de la dama; pero todos tenan fijo el pensamiento en el trovador. i) Don Santiago me encanta por su bondad; no le he tratado mucho; pero adora apasionadamente a sus hijos y es amigo de todos los nios, y no puede verlos sufrir. j) Una valenciana vestida de negro es un prodigio. La tez es blanca, marfilea, de una blancura mate, suave; las facciones son llenas, con redondeces mrbidas; acaso en la blancura se advierten las ramificaciones sutiles de las venillas azules. Y lo negro hace resaltar la blancura maravillosa de esta piel y pone un poco de veladura melanclica en la languidez de la mirada. k) La vida entre aquella gente era grata y amena. Haban llenado de dibujos curiosos y de rtulos chocarreros las puertas, las paredes y las mesas. Yo me hice amigo de uno de aquellos pintores; le acompaaba algunos ratos en sus excursiones; tena un verdadero talento de paisajista. Nos levantbamos antes de amanecer; l gustaba de pintar la luz fina de la maana, y en esa luz, la hojarasca sutil de la primavera.
AZORN Doa Ins, Castalia

1 Localiza en los textos las oraciones simples. 2 Reconoce las oraciones compuestas e indica las que las forman. 3 Seala el tipo de coordinacin que une las oraciones e indica los nexos.

4 Qu tipo de coordinadas predominan en estas oraciones? 5 Qu coordinacin est ausente en esta relacin de oraciones? Construye cinco ejemplos de este tipo.

116 7.

Sintaxis (II). Clases de oraciones. La coordinacin

MATERIAL FOTOCOPIABLE / Oxford University Press Espaa, S. A.

ACTIVIDADES

ORTOGRAFA

3. Uso del parntesis, la raya y el guin


El parntesis encierra elementos aclaratorios o secundarios incluidos en un enunciado. Se utiliza en los siguientes casos: Cuando se intercala un inciso que constituye una aclaracin o una informacin secundaria: Quiere terminar el Bachillerato (que le brindar ms posibilidades en el futuro) y se esfuerza muchsimo. Para estos incisos tambin se pueden usar rayas. Para intercalar algn dato (fechas, lugares, significados de siglas, autor u obra citados): Rubn Daro (1867-1916) naci en Metapa (Nicaragua); Las actuaciones del FMI (Fondo Monetario Internacional) han sido discutidas; Lo bueno, si breve, dos veces bueno; y aun lo malo, si poco, no tan malo. (Gracin) Cuando se transcriben textos se utilizan tres puntos entre parntesis () para indicar que se omite un fragmento. Para las letras o nmeros que sealan enumeraciones o clasificaciones: Los parntesis se usan en los siguientes casos: a) Cuando se intercala un inciso. La raya se puede usar aisladamente o bien como signo de apertura y de cierre que incluye un elemento o enunciado. Se utiliza en los siguientes casos: Para intercalar incisos o aclaraciones (en este empleo pueden ser sustituidas por parntesis o por comas): Quiere abandonar el Bachillerato est muy desanimada y ponerse a trabajar. Para sealar cada una de las intervenciones de un dilogo: No puedes abandonar ahora. T crees que soy capaz de aprobar esas asignaturas? Para intercalar las intervenciones del narrador en el dilogo de los personajes. Piensa con calma en lo que vas a hacer dijo la profesora. Piensa en tu futuro. El guin es un trazo ms corto que la raya y se usa para separar los dos elementos de una palabra compuesta o para marcar la divisin de una palabra al final del rengln: terico-prctico; ren-gln.

1 Coloca los parntesis que faltan en los siguientes enunciados.


a) Las palabras que empiezan por los

MATERIAL FOTOCOPIABLE / Oxford University Press Espaa, S. A.

elementos compositivos xenoextranjero, xero- seco, rido y xilo- madera se escriben con x.
b) Jos Mara Merino A Corua, 1941 ha

obtenido numerosos premios literarios a lo largo de su trayectoria.


c) La novela picaresca se inici con

el Lazarillo de Tormes 1554.


d) Poderoso caballero es don Dinero Francisco

de Quevedo.
e) La letra x, al final de palabra o en posicin

intervoclica, representa el grupo consonntico ks o gs en pronunciacin relajada.


f) Cierta tarde de otoo fresca y lluviosa como

casi todas ese ao, haba tomado aquella drstica decisin.

2 Escribe los parntesis, rayas y comillas que faltan en estos enunciados (extrados de La magia de leer, de Jos Antonio Marina). a) Nacemos con una insaciable hambre de estmulos. En este sentido y en otros, por supuesto, nos parecemos mucho a nuestros primos los primates. b) Hay lectores a quienes les gusta releer el mismo libro Blanca, mi hija pequea, lleg a leerse el primer libro de Harry Potter cincuenta veces! y los hay vidos de novedades. c) Sin la ayuda del lenguaje somos estpidos, inarticulados, toscos e insociables. Posibilidades o mejor, imposibilidades tristes todas ellas. d) Otra fundacin, la FAD Fundacin de Ayuda contra la Drogadiccin, realiz, en 2004, una gua de lectura, basndose en el libro Adivina cunto te quiero, escrito por un maestro irlands, Sam McBratney. e) La revolucin francesa fue una revolucin de lectores: En Pars dice un testigo todo el mundo lee.

7.

Sintaxis (II). Clases de oraciones. La coordinacin

117

Evaluacin

Playa azul, por Marcel Gromaire.

Agosto
A esta hora la playa est ya bastante concurrida pero a las doce no se podr dar un paso. Tengo delante a una familia cosmopolita. La abuela es gallega, va vestida de negro y se cobija bajo una sombrilla. Es una estampa antigua y no se pierde detalle. C Ay, carallo , exclama de vez en cuando. La madre es castellana y se tuesta al sol. El padre habla en cataln con la hija, una nia que, disgustada por el tamao de sus muslos, se cubre de cintura para abajo con una toalla. El padre quiere que se la quite pero ella hace mohnes. Tres adolescentes, a mi derecha, comentan sus asuntos. Xabi es que es el ms guapo de la colla, dice una. Y multimillonario, matiza la segunda. Pero un plomo, cuando te llama por telfono te tiene una hora, yo miro la tele mientras habla, afirma la tercera. Las ha hundido. La primera suspira y se tumba en la toalla, voy a dormir un rato que ayer estuve hasta las cinco con, bueno, ni s cmo se llamaba, deja caer para compensar. La abuela, muy animada, nos anuncia un evento, ya sale, ya sale la rapacia.
Flix DE AZA Esplendor y nada, Lector

1. Resume el contenido del fragmento de Flix de Aza. 2. Qu modalidades oracionales encuentras en el texto
Agosto?

7. Seala la funcin sintctica que cumplen los pronombres


tonos subrayados en el pasaje.

8. Analiza sintcticamente las siguientes oraciones del texto


y descrbelas segn los criterios estudiados. a) La madre es castellana y se tuesta al sol. b) Tres adolescentes, a mi derecha, comentan sus asuntos. c) La primera suspira y se tumba en la toalla.

3. Escribe ejemplos (propios y originales) de las modalidades


oracionales que no aparezcan.

4. Por qu dice el autor que la tercera adolescente ha


hundido a sus amigas? Qu inferencias ha hecho?

5. Localiza en el discurso del narrador dos verbos intransitivos, dos verbos transitivos y todos los verbos que presenten atributo.

9. Qu usos del pronombre se no aparecen en el texto?


Pon un ejemplo de cada uno de ellos.

6. Identifica en esos mismos enunciados las oraciones


simples y las compuestas.

10. Contina el texto: imagina que ests en esa playa (o en


otra cualquiera) e informa de lo que ocurre; incluye reflexiones sobre lo que ves y sobre tus sensaciones. 115 7.
Sintaxis (II). Clases de oraciones. La coordinacin

ACTIVIDADES

AMPLIACIN

1. La vaguedad de las palabras


El siguiente texto trata sobre una caracterstica muy importante del lenguaje humano: la vaguedad de las palabras. Este rasgo de las lenguas plantea a veces dificultades de comunicacin a la hora de designar realidades concretas.

Palabras exactas y palabras inexactas


Decimos que un trmino es vago cuando sus lmites designativos son imprecisos. En tales circunstancias siempre existen realidades que pueden ser denotadas tanto por l como por signos vecinos en el sistema. No es infrecuente que una mujer de 25 aos sea designada con los nombres de nia, chica, muchacha, mujer e incluso seora. Ante un montn de tablas viejas unos hablantes aplicaban el trmino lea y otros la palabra madera. En un artculo de prensa se designaba una va de comunicacin urbana ya como calle ya como avenida. De mediana edad es un trmino vago en el sentido que acabamos de especificar. Uno no es de mediana edad ni a los cinco ni a los ochenta aos; s lo es a los cincuenta; pero, qu es a los 39, a los 41 o a los 60? A cada lado de los casos claros de edad parece haber una franja respecto a la cual no est claro qu es lo que debemos decir (Alston, 1974). La vaguedad es un fenmeno de designacin, no de significacin. Deriva de las fronteras borrosas que posee la clase designativa de los signos de las lenguas naturales. En el significado se establecen oposiciones, saltos cualitativos. Los significados son unidades discretas, pero en la designacin la transicin es imperceptible. Dnde est el lmite en la realidad entre claro y oscuro? [] Los signos de las lenguas naturales conocen un alto grado de vaguedad. El lenguaje cientfico y administrativo, que persigue la precisin y exactitud, aspira a evitarla en la medida de lo posible. Esta es la razn por la que en la ley el paso de menor a mayor de edad sea fijado minuciosamente (las cero horas del da en que se cumplen los 18 aos). Esta precisin es un ideal o desideratum que rara vez se consigue, pues las definiciones del lenguaje cientfico utilizan trminos del lenguaje natural (que a su vez son vagos). [] El reglamento del ftbol pretende delimitar con minuciosidad las circunstancias del penalti, pero cada jornada deportiva aparecen casos dudosos y discutibles.
Salvador GUTIRREZ ORDEZ Introduccin a la semntica funcional Sntesis

MATERIAL FOTOCOPIABLE / Oxford University Press Espaa, S. A.

1 Escribe un resumen en el que recojas las ideas principales expuestas en el texto anterior. 2 Cmo se organizan los contenidos que desarrolla este fragmento de Gutirrez Ordez? 3 En el caso de los nombres con que puede ser designada una mujer, qu factores contextuales o situacionales pueden intervenir en la eleccin de un trmino u otro?

4 Menciona otros ejemplos de trminos vagos cuyo empleo sea habitual en el lenguaje cotidiano. 5 Menciona voces del mbito escolar cuya aplicacin se halle delimitada y establecida y que, sin embargo, en ocasiones planteen distintas interpretaciones. 6 Qu procedimientos pueden usarse en el lenguaje cotidiano para restringir la vaguedad de algunas palabras?

8.

Semntica y lxico

131

ACTIVIDADES

REFUERZO

8
Los siguientes textos plantean la cuestin de los agrupamientos lxicos, en este caso, el campo semntico concreto de las edades del hombre, y el problema que surge en muchas ocasiones para determinar el contenido de los miembros de un campo. El primer fragmento pertenece a una novela humorstica de Eduardo Mendoza, Sin noticias de Gurb, cuyo protagonista es un extraterrestre que llega a la tierra y describe lo que observa desde su perspectiva. El segundo texto pertenece al Libro de estilo del diario El Pas. Este diario considera necesario establecer para sus redactores normas que determinen el trmino adecuado para referirse a las personas segn su edad.

2. Cuestiones semnticas

Texto 2
Hay que evitar expresion es tan desafortunadas (y frecuentes) como una jov en de 33 aos. La norm a es la siguiente: beb, menos de un ao; nia o nio, de 1a 12 aos; joven y adolesc ente, de 13 a 18 aos; hom bre o mujer, ms de 18 aos; anciana o anciano, ms de 65 aos. Este ltimo trmin o, sin embargo, solo deb e emplearse muy excepcion almente, y ms como exp onente de decrepitud fsica que como un estadio de edad. En tales casos, soslyese con expresiones como un hombre de 65 aos.
Libro de estilo El Pas

s mejanza de lo en cambio, a se , os , llo an m rro hu sa s Los sere apas de de r tres fases o et po a er an im es vi pr ra la at , insectos s que estn en lo permite. A lo segunda, si el tiempo se s; a los de la o ni a in s m no de lados. Los nio etapa se les la tercera, jubi de ro s pe lo , a n y bi currantes; currantes, tam les manda; los rciben hacen lo que se dos tambin pe bi r ello; los ju la po , pors da do na ui r ib ce tr re ha son no se les deja ro pe s, to sa en er las co s de unos emolum y suelen dejar ca e m fir los es no dico. [] Entre que su pulso stn y el peri ba a el un o lv n bi sa , m os ta las man sotros, se da no tre en o de m , co es la seres humanos retribuida, que o condicin, no . ar bl cuarta etapa ha que ms vale no Eduardo MENDOZA fiambre, y de la

Texto 1

rb Sin noticias de Gu Seix Barral

1 Cmo designa el narrador de Sin noticias de Gurb a los seres humanos que viven cada etapa evolutiva y qu rasgos semnticos usa para caracterizar estas fases? 2 Establece el campo semntico de las edades o etapas de desarrollo de las personas y compn un texto en el que las caracterices. 3 Escribe los sinnimos posibles para designar a las personas que viven cada una de las diferentes etapas. 4 Con qu dificultad nos podemos encontrar a la hora de aplicarle a las dems personas trminos como nio, adolescente, joven, anciano, seora, etc.? Qu ocurre con estas palabras en relacin con lo que designan?

5 Algunas de estas voces pueden usarse tambin con valores connotativos; pon ejemplos que ilustren estos casos. 6 Para aludir a una determinada etapa de la vida, se usa tambin la expresin tercera edad. Qu clasificacin supone esta denominacin? Justifica tu respuesta. 7 Lee atentamente las normas propuestas en el fragmento del Libro de estilo de El Pas. Responden al uso habitual que hacemos de esos trminos? 8 Menciona ejemplos de situaciones comunicativas en las que exista coincidencia o disidencia con lo que se afirma en este texto.

132 8.

Semntica y lxico

MATERIAL FOTOCOPIABLE / Oxford University Press Espaa, S. A.

ACTIVIDADES

REFUERZO

8
A lo largo del tiempo, las palabras adquieren nuevos significados. El siguiente texto aborda este fenmeno semntico. Lelo atentamente y responde a las cuestiones que se plantean a continuacin.

3. Nuevos significados

Perdonar
En el mbito futbolstico se ha desarrollado hace poco con virulencia agresiva una metfora que juzgo incurable. La omos a diario (esta temporada, literalmente a diario): el equipo se estira, el delantero le gana la espalda (?) a la defensa, est solo ante el portero en uno contra uno, el gol se ruge ya por la multitud, pero el crack chuta y manda la pelota a hacer gambetas al bandern. Y en ese instante, indefectiblemente, el comentarista-filsofo que suele acompaar en las retransmisiones al narrador, emite su solvente excogitacin: el Zaragoza (lo nombro porque lo quiero y porque es diestro en esa pifia) est perdonando mucho. Luego, el exegeta asevera grave: El equipo que perdona mucho acaba perdiendo. Y enseguida, sentenciar ms hondo an: El ftbol es as. Es probable que toda la comunidad hablante adopte pronto el verbo perdonar con ese inusitado significado intransitivo: En el ftbol, desperdiciar repetidamente un equipo las ocasiones de meter gol, antes se deca simplemente fallar. La nueva acepcin, por el momento, solo pertenece a la jerga balompdica, pero como el ftbol sale hasta por el tubo del dentfrico, el vocablo ser muy pronto de conocimiento general. Y de este modo, un tropo inventado como graciosa creacin personal por un ignoto artista de la crnica deportiva, ha cundido hasta rebosar por toda la extensin de las ondas y del papel. Ello constituye buena prueba de que el desenfado de muchos tales comentaristas puede convertir en triunfo el dislate. Porque perdonar significa en el habla comn alzar la pena, eximir o liberar de una obligacin a alguien. Y el arquero no tena obligacin de dejarse meter gol: no haba que eximirlo, al contrario. Por otra parte, quien perdona lo hace adrede y cobra fama de misericordioso, pero las gradas embravecidas suelen llamar imbcil al futbolista o al equipo que, queriendo arrasar al contrario todo menos perdonarlo!, marra el tanto tenindolo a huevo. Evidentemente, el idioma del estadio y de sus aledaos periodsticos es el ms desenfadado de todos, y en l se produce la mayor creatividad imaginable, en gran parte bastante estlida.
MATERIAL FOTOCOPIABLE / Oxford University Press Espaa, S. A.

Fernando LZARO CARRETER El dardo en la palabra Crculo de Lectores

1 En el texto, el autor reflexiona sobre un nuevo significado otorgado por los comentaristas deportivos a la palabra perdonar en su uso metafrico. Haz un resumen de las ideas expuestas en estas lneas. 2 Ests de acuerdo con la ltima afirmacin del profesor Lzaro Carreter? Por qu? 3 Cita otras metforas usuales en la jerga del ftbol. 4 En las crnicas de ftbol, a qu campo asociativo suelen pertenecer los trminos utilizados metafricamente? Por qu?

5 Escribe enunciados en los que las siguientes palabras tengan significados distintos a los del texto: equipo, defensa, portero, chutar, gol. 6 Redacta la definicin de crack, delantero y estadio. 7 Crees que en los comentarios de otras competiciones deportivas (ciclismo, baloncesto, tenis, etctera) se utilizan o crean metforas semejantes a la que se critica en el texto? En caso afirmativo, pon algn ejemplo.

8.

Semntica y lxico

133

ACTIVIDADES

REFUERZO

8
El lenguaje cientfico y tcnico incorpora muchos extranjerismos; he aqu una reflexin del lingista Rafael Lapesa acerca de este fenmeno:

4. Extranjerismos lingsticos
El equilibrio justo
S, estamos inmersos en el extranjerismo lingstico. Nos rodea, nos aturde, nos invade. Se interpone entre las cosas y nosotros, se infiltra en nuestra manera de verlas. Modifica paulatinamente la estructura de nuestra lengua y la configuracin de nuestro pensamiento. Ocurre as en gran parte por frivolidad o ignorancia; en otra gran parte por cuquera propagandstica: el marchamo forneo deja boquiabiertos a los papanatas, y estos abundan tanto que es prctico deslumbrarlos. Pero hay otra causa innegable y ms profunda, y es que desde hace tres siglos vamos a remolque del restante mundo occidental, tanto en las innovaciones con que la ciencia y la tcnica han cambiado las condiciones de la vida humana, cuanto en la exploracin de nuevos derroteros para las ideas. Primero fuimos a la zaga de Francia; despus, a la de Francia, Inglaterra y Alemania; ahora, adems, a la de Estados Unidos. Con las ideas y las cosas vienen inevitablemente las palabras. Ante este hecho, las lamentaciones son tan intiles como las protestas. La nica actitud positiva consistir en aprovecharnos del impulso ajeno tan hbilmente que en un futuro ms o menos prximo podamos seguir nuestro camino con autonoma; y por lo que se refiere al lenguaje, asimilar lo necesario para que nuestro idioma se mantenga a la altura de los tiempos, sin dejar de responder a lo que estos exigen; pero evitar en lo posible que tal puesta al da menosprecie su belleza y peculiaridad. El aluvin de extranjerismos no es problema que afecte solo al espaol. Por todas partes se oyen clamores parecidos, como ya dice Madariaga. Es que el mundo se nos queda chico. Tenemos noticia inmediata de lo que est pasando en Los ngeles, Ciudad del Cabo, Nueva Delhi o Gteborg, y con frecuencia lo vemos en directo. La comunicacin es tan rpida que no deja tiempo a traducciones reposadas. Las novedades tcnicas desde el ciclotrn al humilde rollito de cinta adherente se difunden a toda prisa con su nombre de origen, antes que se les busque sustituto, y muchas veces sin que quepa buscrselo por ser marca registrada. La frecuencia de viajes, el intercambio de turistas, estudiosos y emigrantes, todo conduce a la creciente uniformacin de horizontes y ambientes vitales, a la familiaridad con lo que antes era extrao, a su infiltracin en ideas, costumbres, maneras y lenguaje. En un mundo en que se internacionalizan sociedades masificadas el purismo lingstico est condenado al ms rotundo fracaso. Pero tampoco debemos cruzarnos de brazos y abandonar a la evolucin ciega un instrumento de tan rica tradicin y maravillosa capacidad como es nuestra lengua. [] En resumen, las circunstancias reclaman que en el uso de nuestra lengua tengamos una actitud de razonable abertura frente al neologismo: importar lo imprescindible y lo conveniente, sin fruncir demasiado el ceo ante alguna fruslera de contrabando. Los sagaces casuistas de antao saban que en ciertas ocasiones es necesario lo superfluo. El proteccionismo deseable debe consistir en sacar partido a los recursos naturales del idioma y a los de su patrimonio grecolatino; limitar el capricho con hbil flexibilidad; e imponer montaje fonolgico y gramatical espaol a lo que exhiba patente aliengena. As podremos seguir yendo a Calatayud y no a Kahlahtahyood; pero con nimo de no detenernos en la recia localidad aragonesa, con miras a horizontes ms ambiciosos que los divisables desde sus campanarios mudjares.
Rafael LAPESA El espaol moderno y contemporneo Crtica

1 Cules son las causas ms importantes de la presencia de extranjerismos en el lenguaje cientfico y tcnico? 2 Explica la postura de Rafael Lapesa a este respecto. Qu propuestas hace el autor en este texto? 3 Realiza un resumen de este fragmento del profesor Lapesa.

4 Crees que los medios de comunicacin de masas contribuyen de alguna manera a la difusin de los neologismos? Por qu? Ilustra tu respuesta con ejemplos. 5 Cita nombres comunes del lenguaje ordinario que provengan de marcas registradas. 6 Pon ejemplos de extranjerismos innecesarios y sustityelos por expresiones castellanas.

134 8.

Semntica y lxico

MATERIAL FOTOCOPIABLE / Oxford University Press Espaa, S. A.

ACTIVIDADES

REFUERZO

8
A continuacin, se reproducen titulares publicados en el diario El Pas y se proponen actividades relacionadas con los fenmenos lxico-semnticos estudiados en esta unidad.

5. Neologismos

Vinoterapia en pareja Y la economa salv el 11-S

Vuelve ABARTH, el tuning legendario


Tanger, un fin de semana bajo coste

Experiencia sensorial en el spa

Motera tenas que ser Trono antiestrs

La cuna del hip-hop se salva


Lorenzo se estrena con pole

Mam, quiero ser punk

El sablista estresado

Los policias antimafia , incomunicados en comisaria Cae una red que fabricaba al da 80 000 copias pirata
Actuaban a la carta y servan a los manteros de un da para otro El sndrome de la clase turista ya alarma a la OMS

Bolsos XXL para todos Marketing online en Espaa Para que funcione la globalizacin Detectives oenegeros

Graffitero de interior Un chip podria diagnosticar el mal celiaco De Eurovisin a Frikivisin


Comerciantes y vecinos abrazan la videovigilancia

La brecha digital en Espaa: carrete o memoria? La ciberdelincuencia se traslada a la web Uno de los servidores ha sido neutralizado Un western rural sobre incomunicacion y violencia Tres detenidos tras un robo por alunizaje en una perfumera

EE UU desliga la ayuda antisida de la castidad

Millones de navegantes comparten fotos, vdeos y artculos a travs de Internet Bruselas quiere castigar a los empresarios que empleen a sin papeles
MATERIAL FOTOCOPIABLE / Oxford University Press Espaa, S. A.

Castilla y Len ha dejado de aplicar el veneno antirroedores


El BBVA dota con 200 millones una fundacin para microcrditos El Congreso de la Lengua debate en su clausura sobre el ciberespacio

La OMS destaca la dificultad de conseguir medicamentos de ltima generacin baratos

Cientos de sin papeles esperan al alba en plazas de Madrid a ser reclutados para trabajar en urbanizaciones
Diez piratas controlan el 80% del correo basura que circula por Internet

Un ataque pirata convierte ms de 11 000 webs en focos de virus


Un temporal de troyanos convierte 400 000 ordenadores en zombies

1 Seala los neologismos presentes en estos textos, reconoce de qu tipo son y, si corresponde, di cmo se han formado. 2 Explica el significado de los neologismos identificados en la actividad 1.

3 Indica algunos de los casos en que ha habido cambios semnticos y menciona las figuras retricas que aparecen. 4 Seala algunos trminos pertenecientes a lenguajes especializados.

8.

Semntica y lxico

135

ACTIVIDADES

ORTOGRAFA

6. Uso de x y s
La letra x representa sonidos diferentes: Si se encuentra en final de palabra o en posicin intervoclica representa el grupo consonntico ks: taxi, relax. En posicin inicial de palabra, se pronuncia frecuentemente como s: xerografa. En posicin final de slaba, se pronuncia como s o ks: extremo. Adems de estas diferencias, normalmente se pronuncia de manera relajada, como gs, o incluso s, por lo cual son frecuentes los errores ortogrficos entre s y x. Existen solo unas pocas reglas para el uso de x: Las palabras que empiezan por los prefijos ex- y extra-: excntrico, extrajudicial. Las palabras que empiezan por las races clsicas xeno- (extranjero), xero- (seco, rido) y xilo- (madera): xenfobo, xerfila, xilfono. Las palabras que empiezan por la slaba ex- seguida del grupo -pr- y muchas que tengan la misma slaba inicial seguida del grupo -pl-: expresar, explicar. En el segundo caso, se excluyen esplendor y sus derivados, entre otras voces.

1 Coloca s o x, segn corresponda: e__acerbar e__ageracin e__cabroso e__cafandra e__altacin e__calofriante e__ange e__camotear e__caparate e__nime e__capulario e__asperado e__carcelar e__terminar e__cavacin e__caqueo e__cedente e__celente e__caramuza e__centricidad e__cepcin e__cardar e__cepto e__carmentar e__ceso e__carpado e__casez e__cesivo e__catimar e__citable e__clamar e__cayola e__clusin e__tincin e__clusivo e__cptico e__clarecer e__cluyente e__clavo e__comulgar e__clerosis e__coriacin e__crecin e__cocer e__tintor e__cremento e__culpar e__cursin e__coger e__cusa e__colapio e__cusar e__collo e__combro e__ecrable e__corbuto e__egesis e__tirpar e__encin e__foliacin e__coria e__equias e__foliante e__corpin e__halacin e__cote e__haustivo e__cotilla e__cozor e__hibicin e__hortar e__humacin e__cudriar e__igencia e__iguo e__torsionador e__crpulo e__iliado e__imente e__crutar e__culido e__istencial __ito e__cupir e__currir __odo e__traccin e__geno e__onerar e__orbitante e__fnter e__orcismo e__pantoso e__tico e__pandir e__parcir e__pectculo e__pansivo e__pecfico e__patriar e__tradicin e__pectacin e__pectro e__pesura e__pectativa e__trafino e__pectorar e__pedicin e__plndido e__pediente e__polear e__tralimitarse e__pedir e__peler e__pontneo e__pendedor e__pulgar e__traeza e__periencia e__putar e__perimental e__queje e__perto e__piacin e__planada e__playarse e__quilar e__plicable e__plicitar e__plorar e__quivo e__talactita e__plosin e__treo e__poliar e__trarradio e__ponente e__portar e__presar e__tereotipo e__tigma e__presionismo e__presivo e__trabismo e__trafalario e__propiar e__trangular e__travagancia e__puesto e__pugnacin e__pulsar e__purgar e__quisitez e__tremidad e__tratagema e__tender e__trategia e__tenuante e__trato e__uberante e__teriorizar e__croto

2 Construye oraciones con las siguientes palabras parnimas; si desconoces algn significado, bscalo en el diccionario: a) esotrico / exotrico c) esttico / exttico e) espiar / expiar g) espirar / expirar b) esclusa / exclusa d) contesto / contexto f) estirpe / extirpe h) laso / laxo

136 8.

Semntica y lxico

MATERIAL FOTOCOPIABLE / Oxford University Press Espaa, S. A.

Evaluacin 8
' ) ! # ,*
Estamos en un comedor estudiantil de una universidad alemana. Una alumna rubia e inequvocamente germana adquiere su bandeja con el men en el mostrador del autoservicio y luego se sienta a una mesa. Entonces advierte que ha olvidado los cubiertos y vuelve a levantarse a cogerlos. Al regresar, descubre con estupor que un chico negro, probablemente subsahariano por su aspecto, se ha sentado en su lugar y est comiendo de su bandeja. De entrada, la muchacha se siente desconcertada y agredida; pero enseguida corrige su pensamiento y supone que el africano no est acostumbrado al sentido de la propiedad privada y de la intimidad del europeo, o incluso que quiz no disponga de dinero suficiente para pagarse la comida, aun siendo esta barata para el elevado estndar de vida de nuestros ricos pases. De modo que la chica decide sentarse frente al tipo y sonrerle amistosamente. A lo cual, el africano contesta con otra blanca sonrisa. A continuacin, la alemana comienza a comer de la bandeja intentando aparentar la mayor normalidad y compartindola con exquisita generosidad y cortesa con el chico negro. Y as, l se toma la ensalada, ella apura la sopa, ambos pinchan paritariamente del mismo plato de estofado hasta acabarlo y uno da cuenta del yogur y la otra de la pieza de fruta. Todo ello trufado de mltiples sonrisas educadas, tmidas por parte del muchacho, suavemente alentadoras y comprensivas por parte de ella. Acabado el almuerzo, la alemana se levanta en busca de un caf. Y entonces descubre, en la mesa vecina detrs de ella, su propio abrigo colocado sobre el respaldo de una silla y una bandeja de comida intacta.

Das de verano, por Dale Kennington.

Dedico esta historia deliciosa, que adems es autntica, a todos los espaoles que, en el fondo, recelan de los inmigrantes y les consideran individuos inferiores. A todas esas personas que, aun bienintencionadas, les observan con condescendencia y paternalismo. Ser mejor que nos libremos de los prejuicios o corremos el riesgo de hacer el mismo ridculo que la pobre alemana, que crea ser el colmo de la civilizacin mientras el africano, l s inmensamente educado, la dejaba comer de su bandeja y tal vez pensaba: Pero qu chiflados estn los europeos.
Rosa MONTERO El Pas, 17 de mayo 2005

1. Resume la historia que se narra en el artculo El negro, de


la periodista Rosa Montero.

6. Escribe ejemplos de trminos sinnimos, antnimos,


homnimos y polismicos a partir de palabras tomadas del texto.

2. Explica los elementos de los que se compone el signo


lingstico, utilizando como ejemplo una palabra extrada del texto.

7. Menciona los sinnimos contextuales o referenciales


con los que se designa en el artculo a los protagonistas de la historia.

3. Autoservicio es una traduccin de la expresin inglesa


self-service. De qu tipo de neologismo se trata? Pon otros ejemplos de la misma clase.

8. Por qu se producen cambios de significado? Qu


formas retricas adoptan? Escribe algunos ejemplos de estos cambios semnticos.

4. Menciona otras clases de prstamos que se hayan incorporado recientemente y pon tres ejemplos de cada uno. Incluye los que aparecen en el texto en la clase que corresponda.

9. Seala las caractersticas de los tecnicismos y explica su


formacin segn su origen y en funcin de los procedimientos de creacin.

5. Localiza en el texto la palabra paritariamente y explica


cmo est formada. Cul es el significado de este trmino en el contexto en el que aparece? Enuncia los procedimientos que ofrece la lengua para crear nuevas palabras y cita ejemplos de cada uno.

10. Expresa tu opinin argumentada sobre la incesante introduccin de extranjerismos en la actualidad. Incluye reflexiones sobre quines los difunden, en qu mbitos de la vida cotidiana abundan especialmente y cul es su arraigo, comprensin y uso en la mayora de los hablantes. 131 8.
Semntica y lxico

ACTIVIDADES

REFUERZO

1. Coherencia, cohesin y adecuacin textuales


A continuacin, figuran tres fragmentos en los que puedes observar las propiedades textuales bsicas: coherencia, cohesin y adecuacin.

Texto 1
mento codificado de La lengua es ese instru a re las personas. Adquirid comunicacin verbal ent r po a nad cio fec per o, ent por herencia tras el nacimi que o alg es a gu len la io, la experiencia y el estud sona, como grficamente lleva puesto toda per se ha dicho. bre primitivo invenTanto si decimos que el hom imos que la invencin del t el lenguaje como si dec ate inteligente se convirlenguaje hizo que un prim s la misma cosa (B. Malmtiera en hombre, decimo , erios de la ciencia actual berg, 1969: 13). En los crit o com ein desde Wittgenst tanto para los filsofos omsky, el hombre es hoCh de des tas is para los ling er o sapiens (M. Bunge, mo loquens antes que fab ial la persona humasoc a 1983: 9). En su experienci a comunicativa de conna se sita en una prctic parte. Es decir, est instala texto comn que se com la en a ret erp int se que a da en el mundo de la vid mbros de una sociedad mie los de dad tivi bje rsu inte organizada.
El enu

Texto 2
Hubiera querido disimu lar. Le habra gustado hacer como en las pel culas, mirar fija e inexpresivamente a la chica, dejar correr el silencio algunos segundos, tambo rilear con delicadeza sus dedos sobre la mesa del bar, reposar la taza otra vez sobre su plato y decir escueto: No s de qu me est usted habla ndo. Pero, en cambio, la glotis equivoc su cam ino, se atragant con el caf con leche y se pus o colorado como un pimiento mientras tosa con estrpito. Comprender que de eso no puedo informarle. Es un secreto mil itar articul al fin a duras penas. Exactamente ese es el problema en este caso, mi amigo. Aqu tod o es un secreto, incluso el nombre de la chica. No me la presenta?
Juan Luis CEBRIN La isla del viento, Alfaguara

Vidal LAMQUIZ discurso, Ariel del ico st ling lisis An nciado textual.

Texto 3
Cuando hablamos de elementos del texto, podemos aludir tanto a elementos del significante, como a elementos del significado. Por supuesto que los segundos estn implcitos y aparecen como un desarrollo a partir de los primeros. Cualquier anlisis del texto debe por tanto tener su punto de partida en el hecho de que el texto est constituido por una sucesin de significantes grficos o monemas, formados por grupos de grafemas. Y es esta sucesin, invariable si se prescinde del desgaste producido por la tradicin, la que desarrolla despus, en el acto de la lectura, los significados.
Cesare SEGRE Principios de anlisis del texto literario, Crtica

MATERIAL FOTOCOPIABLE / Oxford University Press Espaa, S. A.

1 Explica por qu es coherente el texto 1 atendiendo al tema, a la relacin entre las proposiciones y a las presuposiciones. 2 Indica cmo se da la conexin lxica en el fragmento de Vidal Lamquiz. 3 Seala los elementos decticos del texto 2 y explica en cada caso de qu tipo de deixis se trata.

4 Indica en el fragmento de Juan Luis Cebrin la presencia de un conector y di de qu tipo es. 5 Busca en ese mismo texto ejemplos de anforas. 6 Analiza todos los elementos que aseguran la cohesin del texto 3. 7 Son adecuados los tres textos anteriores? Justifica tu respuesta.

9.

El texto

149

ACTIVIDADES

REFUERZO

2. Estructuradores de la informacin y otros marcadores


Los textos ensaysticos abundan en estructuradores de la informacin y otros tipos de marcadores. Corrobralo en estos fragmentos de la obra Las preguntas de la vida, de Fernando Savater.

Texto 1
Quiz parezca extrao que un libro que quiere iniciar en cuestiones filosficas se abra con un captulo dedicado a la muerte. No desanimar un tema tan lgubre a los nefitos? No sera mejor comenzar hablando de la libertad o del amor? Pero ya he indicado que me propongo invitar a la filosofa a partir de mi propia experiencia intelectual y en mi caso fue la revelacin de la muerte de mi muerte como certidumbre lo que me hizo ponerme a pensar. Y es que la evidencia de la muerte no solo le deja a uno pensativo, sino que le vuelve a uno pensador. Por un lado, la conciencia de la muerte nos hace madurar personalmente: todos los nios se creen inmortales (los muy pequeos incluso piensan que son omnipotentes y que el mundo gira a su alrededor; salvo en los pases o en las familias atroces donde los nios viven desde muy pronto amenazados por el exterminio y los ojos infantiles sorprenden por su fatiga mortal, por su anormal veterana), pero luego crecemos cuando la idea de la muerte crece dentro de nosotros. Por otro lado, la certidumbre personal de la muerte nos humaniza, es decir, nos convierte en verdaderos humanos, en mortales. Entre los griegos humano y mortal se deca con la misma palabra, como debe ser.

Texto 2
Inquietarse por los aos y los siglos en que ya no estaremos entre los vivos resulta tan caprichoso como preocuparse por los aos y los siglos en que an no habamos venido al mundo. Ni antes nos doli no estar ni es razonable suponer que luego nos doler nuestra definitiva ausencia. En el fondo, cuando la muerte nos hiere a travs de la imaginacin pobre de m, todos tan felices disfrutando del sol y del amor; todos menos yo, que ya nunca ms, nunca ms! es precisamente ahora que todava estamos vivos. Quiz deberamos reflexionar un poco ms sobre el asombro de haber nacido, que es tan grande como el espantoso asombro de la muerte. Si la muerte es no ser, ya la hemos vencido una vez: el da que nacimos. Es el propio Lucrecio quien habla en su poema filosfico de la mors aeterna, la muerte eterna de lo que nunca ha sido ni ser. Pues bien, nosotros seremos mortales pero de la muerte eterna ya nos hemos escapado. A esa muerte enorme le hemos robado un cierto tiempo los das, meses o aos que hemos vivido, cada instante que seguimos viviendo y ese tiempo pase lo que pase siempre ser nuestro, de los triunfalmente nacidos, y nunca suyo, pese a que tambin debamos luego irremediablemente morir.

Texto 3
MATERIAL FOTOCOPIABLE / Oxford University Press Espaa, S. A.

Parece evidente que el peso de los condicionamientos subjetivos vara grandemente segn el campo de verdad que en cada caso estemos considerando: si de lo que hablamos es de mitologa, de gastronoma o de expresin potica, el peso de nuestra cultura o nuestra idiosincrasia personal es mucho ms concluyente que cuando nos referimos a ciencias de la naturaleza o a los principios de la convivencia humana. En cualquier caso, tambin para determinar hasta qu punto nuestros conocimientos estn teidos de subjetivismo necesitamos un punto de vista objetivo desde el que compararlos unos con otros y todos con una cierta realidad ms all de ellos a la que se refieren! En fin, hasta para desconfiar de los criterios universales de razn y de verdad necesitamos algo as como una razn y una verdad que sirvan de criterio universal. Sin embargo, la aportacin ms valiosa del relativismo consiste en subrayar la imposibilidad de establecer una fuente ltima y absoluta de la que provenga todo conocimiento verdadero.
Fernando SAVATER Las preguntas de la vida, Ariel

1 Resume el contenido de los textos 1 y 2. Cul es el tema comn que los vincula? 2 Reconoce en ellos estructuradores de la informacin. 3 A qu subtipo pertenecen? Explica su valor.

4 Explica el contenido del tercer fragmento. 5 Qu tipo de marcadores aparecen en l? Seala su valor. 6 Qu formas tienen los marcadores presentes en los tres fragmentos?

150 9.

El texto

ACTIVIDADES

ORTOGRAFA

3. Coherencia y signos de puntuacin


Recuerda:

El punto separa oraciones dentro de un texto. La palabra que sigue a un punto y seguido o a un punto y aparte se escribe con mayscula inicial. La coma marca pausas internas dentro de una oracin. El punto y coma marca una pausa mayor que la de la coma y menor que la del punto y seguido. Despus de los dos puntos suele figurar una aclaracin o la consecuencia de lo dicho previamente.

Un mundo decodificable
Supongamos que el seor Sigma en el curso de un viaje a Paris empieza a sentir molestias en el vientre utilizo un termino generico porque el seor Sigma por el momento tiene una sensacion confusa se concentra e intenta definir la molestia ardor de estomago? espasmos? dolores viscerales? intenta dar nombre a unos estimulos imprecisos y al darles un nombre los culturaliza es decir encuadra lo que era un fenomeno natural en unas rubricas precisas y codificadas o sea intenta dar a una experiencia personal propia una calificacion que la haga similar a otras experiencias ya expresadas en los libros de medicina o en los articulos de los periodicos por fin descubre la palabra que le parece adecuada esta palabra vale por la molestia que siente y dado que quiere comunicar sus molestias a un medico sabe que podra utilizar la palabra (que el medico esta en condiciones de entender) en vez de la molestia (que el medico no siente y que quizas no ha sentido nunca en su vida) Todo el mundo estara dispuesto a reconocer que esta palabra que el seor Sigma ha individualizado es un signo pero nuestro problema es mas complejo el seor Sigma decide pedir hora a un medico consulta la guia telefonica de Paris unos signos graficos precisos le indican quienes son medicos y como llegar hasta ellos sale de casa busca con la mirada una seal particular que conoce muy bien entra en un bar si se tratara de un bar italiano intentaria localizar un angulo proximo a la caja donde podria estar un telefono de color metalico pero como sabe que se trata de un bar frances tiene a su disposicion otras reglas interpretativas del ambiente busca una escalera que descienda al sotano sabe que en todo bar parisino que se respete alli estan los lavabos y los telefonos es decir el ambiente se presenta como un sistema de signos orientadores que le indican desde donde podra hablar
Umberto ECO Signo, Labor

1 Vuelve a escribir el texto de Umberto Eco colocando punto, coma, punto y coma y dos puntos donde corresponda y explica en cada caso por qu lo haces. 2 Aade las tildes que faltan en el texto anterior. 3 Completa con coma, punto y coma y dos puntos las siguientes oraciones: a) Nuestra profesora adems de qumica es fsica. b) Todo en ella era muy hermoso sus ojos su boca su pelo. c) Caballero por favor decdase usted por alguno de estos perfumes cerramos dentro de dos minutos. d) Limpia la lentilla as qutatela despus de parpadear colcala en la palma frtala con limpiador diario y aclrala con solucin salina. e) El recuento para mi sorpresa arroj este resultado el viaje a Pars tres votos la excursin por Italia dos las siete noches de Ibiza ninguno y los siete das de senderismo por Sierra Nevada veintisiete.

4 Explica la diferencia de significado entre estos pares de oraciones:


a) Los vecinos, a quienes les toc la lotera,

se han separado. Los vecinos a quienes les toc la lotera se han separado.
b) Han decidido que no; se instalar

MATERIAL FOTOCOPIABLE / Oxford University Press Espaa, S. A.

un portero automtico nuevo. Han decidido que no se instalar un portero automtico nuevo.
c) Los perros, hambrientos, robaron

la comida. Los perros hambrientos robaron la comida.


d) Me sorprende esa respuesta; de tu hermana

no me fo. Me sorprende esa respuesta de tu hermana; no me fo.


e) Sus sobrinas sacaron gran beneficio de la

herencia; la mayor, parte de los inmuebles. Sus sobrinas sacaron gran beneficio de la herencia: la mayor parte de los inmuebles.

9.

El texto

151

Evaluacin 9
' ) < ( ! ,* ! 0: .i( - ( *, . ' ! - ! . , " i * , ! ! / ) 8 % ! ) # * -.*
Ayer, tras recibir el parte de incidencias, en los despachos de la DGT cunda el desnimo. Sobre todo porque los datos negativos, que han roto la tendencia positiva que se arrastraba desde que comenz el ao, han llegado justo cuando se haban intensificado las campaas de concienciacin. A los desplazamientos de fin de semana se suman los de salida o retorno por cambio de quincena, los viajes a poblaciones en fiestas y los de inmigrantes portugueses y magrebes que se dirigen a sus pases para pasar las vacaciones desde Espaa o Francia. An as, Trfico entiende que no hay razones para alarmarse. En primer lugar, porque no se pueden tomar como referencia los accidentes que han ocurrido en solo tres das, y menos comparndolos con los del mismo fin de semana del ao pasado, que fue especialmente bueno, segn advirti un portavoz de la DGT. Tampoco se pueden sacar conclusiones objetivas, de acuerdo con Trfico, a partir de las cifras de las dos primeras semanas de agosto. El ao pasado, adems, se acababa de estrenar el carn por puntos, que entr en vigor en julio de 200 . La DGT interpreta que la novedad estimul la prudencia de los conductores.

Retencin en un da de operacin salida.

En definitiva, los responsables de Trfico se consuelan observando que el balance anual de mortalidad sigue siendo positivo: hasta el pasado domingo, los accidentes de trfico haban causado 1 99 fallecidos. A estas alturas, el ao pasado ya haba 1 891 muertos, con lo cual el descenso se sita en el 10,1 %.

El ltimo fin de semana, por otro lado, ha vuelto a confirmar que corren malos tiempos para la seguridad de los motoristas. De los 37 fallecidos, 11 viajaban a bordo de motocicletas, el nico vehculo de todo el parque mvil en el que la siniestralidad aumenta ao tras ao.
El Pas, 14 de agosto de 2007

1. Cul es el tema del texto que acabas de leer? Explica el


contenido de cada uno de los prrafos en los que se estructura.

7. Analiza la cohesin temporal manifestada por el empleo


de las formas verbales en el texto.

2. Analiza la coherencia del fragmento atendiendo a los


siguientes aspectos. Presuposiciones. Conocimiento del mundo. Implicaciones. Marco.

8. En el fragmento se observan varios marcadores: reconcelos, clasifcalos y explica su funcin. Aade t ejemplos de la o las clases que no aparecen en el texto.
* )! / * -! . * -! . ! ' % ) " * -( %; * -! . /% 1 * . %* ) '! . )

. /-u /u ! " * -( u' + ! -

3. Reconoce en el texto un elemento anafrico y seala su


referente.

* -! . - # u( ! )/ * -! . * ) 1 ! -.

4. Analiza la deixis presente en esta noticia de El Pas. 5. En el texto hay dos elipsis: localzalas y di qu elementos
se han omitido.

9. Consideras el texto apropiado, adecuado y oportuno?


Justifica tu respuesta.

6. En el fragmento tambin puede observarse un caso de


cohesin lxica por hiperonimia. Selalo.

10. Elabora un esquema de todos los elementos que permiten asegurar la cohesin de un texto. Incluye ejemplos. 143 9.
El texto

ACTIVIDADES

REFUERZO

10

1. Textos y modalidades
En un mismo libro, pueden presentarse las distintas modalidades textuales bsicas: narracin, descripcin, exposicin y argumentacin. Es el caso de los textos 2, 3 y 4, que pertenecen a la misma obra, Las preguntas de la vida, de Fernando Savater. El texto 1 es un fragmento de la novela La hora violeta, de Montserrat Roig.

Texto 1
La mar embravecida golpea las rocas. La espuma vuela y lame furiosamente la arena. La mujer del pescador, que lleva una bata de cuadritos y red en la cabeza, mira con insistencia hacia el faro, hacia La Mola. El tiempo se ha revuelto y el marido no regresa. Oigo como la mujer del pescador explica a una extranjera que la barca es pequea, pero fuerte. La extranjera no sabe qu decirle, la consuela y veo que procura prepararla para la resignacin. Otros extranjeros, indiferentes, mojan pan en unos enormes tazones de leche. El cielo es de color gris y, por poniente, est moteado de manchas negras. Las olas insisten, como si alguna fuerza oscura las empujase. Una tras otra van muriendo, furiosas y cansadas, sobre los arrecifes. En el mar hay una escala, tenue, matizada, de colores: verde botella-azul celeste-azul cobalto.
Montserrat ROIG La hora violeta Alianza

Texto 2
Las leyendas ms antiguas no pretenden consolarnos de la muerte sino solo explicar su inevitabilidad. La primera gran epopeya que se conserva, la historia del hroe Gilgamesh, se compuso en Sumeria aproximadamente 2 700 a. C. Gilgamesh y su amigo Enkidu, dos valientes guerreros y cazadores, se enfrentan a la diosa Isthar, que da muerte a Enkidu. Entonces Gilgamesh emprende la bsqueda del remedio de la muerte, una hierba mgica que renueva la juventud para siempre, pero la pierde cuando est a punto de conseguirla. Despus aparece el espritu de Enkidu, que explica a su amigo los sombros secretos del reino de los muertos, al cual Gilgamesh se resigna a acudir cuando llegue su hora. Ese reino de los muertos no es ms que un siniestro reflejo de la vida que conocemos, un lugar profundamente triste. Lo mismo que el Hades de los antiguos griegos. En la Odisea de Homero, Ulises convoca los espritus de los muertos y entre ellos acude su antiguo compaero Aquiles. Aunque su sombra sigue siendo tan majestuosa entre los difuntos como lo fue entre los vivos, le confiesa a Ulises que preferira ser el ltimo porquerizo en el mundo de los vivos que rey en las orillas de la muerte. Nada deben envidiar los vivos a los muertos. En cambio, otras religiones posteriores, como la cristiana, prometen una existencia ms feliz y luminosa que la vida terrenal para quienes hayan cumplido los preceptos de la divinidad (por contrapartida, aseguran una eternidad de refinadas torturas a los que han sido desobedientes).

MATERIAL FOTOCOPIABLE / Oxford University Press Espaa, S. A.

10.

Las modalidades textuales

161

ACTIVIDADES

10

Texto 3
Las plantas y los animales no son mortales porque no saben que van a morir, no saben que tienen que morir: se mueren pero sin conocer nunca su vinculacin individual, la de cada uno de ellos, con la muerte. Las fieras presienten el peligro, se entristecen con la enfermedad o la vejez, pero ignoran (o parece que ignoran?) su abrazo esencial con la necesidad de la muerte. No es mortal quien muere, sino quien est seguro de que va a morir. Aunque tambin podramos decir que ni las plantas ni los animales estn por eso mismo vivos en el mismo sentido en que lo estamos nosotros. Los autnticos vivientes somos solo los mortales, porque sabemos que dejaremos de vivir y que en eso precisamente consiste la vida. Algunos dicen que los dioses inmortales existen y otros que no existen, pero nadie dice que estn vivos: solo a Cristo se le ha llamado Dios vivo y eso porque cuentan que encarn, se hizo hombre, vivi como nosotros y como nosotros tuvo que morir. [] Es precisamente la certeza de la muerte la que hace la vida mi vida, nica e irrepetible algo tan mortalmente importante para m. Todas las tareas y empeos de nuestra vida son formas de resistencia ante la muerte, que sabemos ineluctable. Es la conciencia de la muerte la que convierte la vida en un asunto muy serio para cada uno, algo que debe pensarse. Algo misterioso y tremendo, una especie de milagro precioso por el que debemos luchar, a favor del cual tenemos que esforzarnos y reflexionar. Si la muerte no existiera habra mucho que ver y mucho tiempo para verlo pero muy poco que hacer (casi todo lo hacemos para evitar morir) y nada en qu pensar.

1. Textos y modalidades

REFUERZO

Texto 4
Recuerdo muy bien la primera vez que comprend de veras que antes o despus tena que morirme. Deba andar por los diez aos, nueve quiz, eran casi las once de una noche cualquiera y estaba ya acostado. Mis dos hermanos, que dorman conmigo en el mismo cuarto, roncaban apaciblemente. En la habitacin contigua mis padres charlaban sin estridencias mientras se desvestan y mi madre haba puesto la radio que dejara sonar hasta tarde, para prevenir mis espantos nocturnos. De pronto me sent a oscuras en la cama: yo tambin iba a morirme!, era lo que me tocaba, lo que irremediablemente me corresponda!, no haba escapatoria! No solo tendra que soportar la muerte de mis dos abuelas y de mi querido abuelo, as como la de mis padres, sino que yo, yo mismo, no iba a tener ms remedio que morirme. Qu cosa tan rara y terrible, tan peligrosa, tan incomprensible, pero sobre todo qu cosa tan irremediablemente personal! [] A los diez aos cree uno que todas las cosas importantes solo les pueden pasar a los mayores: repentinamente se me revel la primera gran cosa importante de hecho, la ms importante de todas que sin duda ninguna me iba a pasar a m. Iba a morirme, naturalmente dentro de muchos, muchsimos aos, despus de que se hubieran muerto mis seres queridos (todos menos mis hermanos, ms pequeos que yo).
Fernando SAVATER Las preguntas de la vida Ariel

1 A qu modalidad textual corresponde cada uno de los fragmentos? Justifica tu respuesta atendiendo al contenido y a la finalidad. 2 En el texto 1 predomina una de las modalidades, pero incluye tambin otra; de qu manera lo hace? 3 El texto 2 pertenece a una de las modalidades textuales bsicas, pero incluye otra de ellas. Razona esta afirmacin.

4 Realiza un resumen del texto 2 y explica su estructura. 5 Enuncia el tema y la tesis del texto 3. 6 Determina los argumentos que sustentan dicha tesis. 7 Extrae la idea principal del texto 4 8 Analiza las caractersticas lingsticas y textuales de los textos 1 y 3.

162 10.

Las modalidades textuales

MATERIAL FOTOCOPIABLE / Oxford University Press Espaa, S. A.

ACTIVIDADES

REFUERZO

10

2. Narraciones literarias
Quim Monz es uno de los grandes narradores de la actualidad. Sus cuentos, que han sido traducidos a muchas lenguas, han merecido numerosos premios. A continuacin, reproducimos un cuento completo y un fragmento de otro, ambos publicados por la editorial Anagrama en el libro Ochenta y seis cuentos.

La bella durmiente
En medio de un claro, el caballero ve el cuerpo de la muchacha, que duerme sobre una litera hecha con ramas de roble y rodeada de flores de todos los colores. Desmonta rpidamente y se arrodilla a su lado. Le coge una mano. Est fra. Tiene el rostro blanco de una muerta. Y los labios finos y morados. Consciente de su papel en la historia, el caballero la besa con dulzura. De inmediato la muchacha abre los ojos grandes, almendrados y oscuros, y lo mira: con una mirada de sorpresa que enseguida (una vez ha cavilado quin es y dnde est y por qu est all y quin ser ese hombre que tiene al lado y que, supone, acaba de besarla) se tie de ternura. Los labios van perdiendo el tono morado y, una vez recobrado el rojo de la vida, se abren en una sonrisa. Tiene unos dientes bellsimos. El caballero no lamenta nada tener que casarse con ella, como estipula la tradicin. Es ms: ya se ve casado, siempre junto a ella, compartindolo todo, teniendo un primer hijo, luego una nena y por fin otro nio. Vivirn una vida feliz y envejecern juntos. Las mejillas de la muchacha han perdido la blancura de la muerte y ya son rosadas, sensuales, para morderlas. l se incorpora y le alarga las manos, las dos, para que se coja a ellas y pueda levantarse. Y entonces, mientras (sin dejar de mirarlo a los ojos, enamorada) la muchacha (dbil por todo el tiempo que ha pasado acostada) se incorpora gracias a la fuerza de los brazos masculinos, el caballero se da cuenta de que (unos veinte o treinta metros ms all, mucho antes de que el claro d paso al bosque) hay otra muchacha dormida, tan bella como la que acaba de despertar, igualmente acostada en una litera de ramas de roble y rodeada de flores de todos los colores.

El sapo
El prncipe inclina el cuerpo y adelanta la cara. El sapo est justo frente a l. La papada se le hincha y deshincha sin cesar. Ahora que lo ve tan cerca siente que lo invade el asco; pero no tarda en reponerse y acerca los labios al morro del anfibio. Mua. En menos de una milsima de segundo, con un ruido ensordecedor, el sapo se convierte en un prisma de cien mil colores, que multiplica infinitamente las caras, hasta que todas las caras y colores se convierten en una muchacha preciosa de cabellos de oro. Y una corona encima que demuestra la nobleza de su linaje. Por fin el prncipe ha encontrado a la mujer que siempre ha buscado, esa con la que compartir el trono y la vida. Por fin has llegado le dice ella. Si supieras cmo he esperado al prncipe que deba librarme del hechizo. Lo comprendo. Te he buscado siempre, desde que era nio. Y siempre he sabido que te encontrara. Se miran a los ojos, se cogen las manos. Es para siempre, y los dos son conscientes de ello. Era como si este momento no fuera a llegar nunca dice ella. Pues ya ha llegado. S. Qu bien, no? Ests contento? S. Y t? Yo tambin. El prncipe mira el reloj. Qu ms debe decirle? De qu deben hablar? Debe invitarla enseguida a su casa o se lo tomar a mal? En realidad no hay ninguna prisa. Tienen toda la vida por delante. En fin S. Ya ves Tanto esperar y de repente, plaf, ya est. S, ya est.

MATERIAL FOTOCOPIABLE / Oxford University Press Espaa, S. A.

1 Realiza un esquema de las acciones principales de cada uno de los textos. 2 Menciona los otros elementos propios de una narracin y explica cmo se manifiestan en estos textos. 3 Los dos relatos remiten a cuentos tradicionales. Qu partes y elementos estn modificados? En qu consiste el cambio y cul puede ser la finalidad?

4 Analiza en ambos textos el narrador, la perspectiva y seala con qu estilo se presenta el discurso de los personajes. 5 Reconoce los rasgos lingsticos propios de la narracin que aparecen en estos textos. 6 A partir de otro cuento tradicional que recuerdes, crea uno nuevo alterando el esquema narrativo o las caractersticas de los elementos principales del relato.

10.

Las modalidades textuales

163

ACTIVIDADES

REFUERZO

10

3. Descripciones literarias
La descripcin es una modalidad presente en el gnero narrativo y, generalmente, muestra las particularidades de la tendencia literaria a la que pertenecen las novelas en que se hallan incluidas y ciertos rasgos caractersticos del estilo del escritor.

Texto 1
La noche le pareci interminable: dio vueltas y ms vueltas; apagaron la luz elctrica, los tranvas cesaron de pasar, la plaza qued a oscuras! Entre la calle de la Montera y la de Alcal iban y venan delante de un caf, con las ventanas iluminadas, mujeres de trajes claros y pauelos de crespn, cantando, parando a los noctmbulos; unos cuantos chulos, agazapados tras de los faroles, las vigilaban y charlaban con ellas, dndoles rdenes Luego fueron desfilando busconas, chulos y celestinas. Todo el Madrid parsito, holgazn, alegre, abandonaba en aquellas horas las tabernas, los garitos, las casas de juego, las madrigueras y los refugios del vicio, y por en medio de la miseria que palpitaba en las calles, pasaban los trasnochadores con el cigarro encendido, hablando, riendo, bromeando con las busconas, indiferentes a las agonas de tanto miserable desharrapado, sin pan y sin techo, que se refugiaba temblando de fro en los quicios de las puertas. Quedaban algunas viejas busconas en las esquinas, envueltas en el mantn, fumando Tard mucho en aclarar el cielo; an de noche se armaron puestos de caf; los cocheros y los golfos se acercaron a tomar su vaso o su copa. Se apagaron los faroles de gas. Danzaban las claridades de las linternas de los serenos en el suelo gris, alumbrado vagamente por el plido claror del alba, y las siluetas negras de los traperos se detenan en los montones de basura, encorvndose para escarbar en ellos. Todava algn trasnochador plido, con el cuello del gabn levantado, se deslizaba siniestro como un bho ante la luz, y mientras tanto comenzaban a pasar obreros El Madrid trabajador, y honrado se preparaba para su ruda faena diaria.
Po BAROJA La busca, Caro Raggio

Texto 2
Lul era una muchacha graciosa, pero no bonita; tena los ojos verdes, oscuros, sombreados por ojeras negruzcas; unos ojos que a Andrs le parecieron muy humanos; la distancia de la nariz a la boca y de la boca a la barba era en ella demasiado grande, lo que le daba cierto aspecto simio; la frente pequea, la boca, de labios finos, con una sonrisa entre irnica y amarga; los dientes blancos, puntiagudos; la nariz un poco respingona, y la cara plida, de mal color. Lul demostr a Hurtado que tena gracia, picarda e ingenio de sobra; pero le faltaba el atractivo principal de una muchacha: la ingenuidad, la frescura, la candidez. Era un producto marchito por el trabajo, por la miseria y por la inteligencia. Sus dieciocho aos no parecan juventud.
Po BAROJA El rbol de la ciencia, Caro Raggio

164 10.

Las modalidades textuales

MATERIAL FOTOCOPIABLE / Oxford University Press Espaa, S. A.

Texto 3
Este es un casino amplio, nuevo, cmodo. Est rodeado de un jardn; el edificio consta de dos pisos, con balcones de piedra torneada. Primero aparece un vestbulo enladrillado de menuditos mosaicos pintorescos; los motantes de las puertas cierran con vidrieras de colores. Despus se pasa a un saln octgono; enfrente est el gabinete de lectura, con una agradable sillera gris y estantes llenos de esos libros grandes que se imprimen para ornamentacin de las bibliotecas en que no lee nadie. A la derecha hay un gran saln vaco (porque no hace falta tanto local), y a la izquierda otro gran saln igual al anterior, donde los socios se renen con preferencia. Mesas cuadradas y redondas de mrmol se hallan esparcidas ac y all, alternando con otras de tapete verde; junto a la pared, corre un ancho divn de peluche rojo; en un ngulo destaca un piano de cola, y verdes jazmines cuajados de florecillas blancas festonean las ventanas. Son los primeros das de otoo; los balcones estn cerrados; el viento mueve un leve murmullo en el jardn; poco a poco van llegando los socios a su recreo de la noche; brillan las lmparas elctricas.
AZORN Antonio Azorn Ctedra

REFUERZO

ACTIVIDADES

Texto 4
Yo recordaba nebulosamente aquel antiguo jardn donde los mirtos seculares dibujaban los cuatro escudos del fundador en torno de una fuente abandonada. El jardn y el Palacio tenan esa vejez seorial y melanclica de los lugares por donde en otro tiempo pas la vida amable de la galantera y del amor. Bajo la fronda de aquel laberinto, sobre las terrazas y en los salones, haban florecido las risas y los madrigales, cuando las manos blancas que en los viejos retratos sostienen apenas los paolitos de encaje, iban deshojando las margaritas que guardan el cndido secreto de los corazones. Hermosos y lejanos recuerdos! Yo tambin los evoqu un da lejano, cuando la maana otoal y dorada envolva el jardn hmedo y reverdecido por la constante lluvia de la noche. Bajo el cielo lmpido, de un azul herldico, los cipreses venerables parecan tener el ensueo de la vida monstica. La caricia de la luz temblaba sobre las flores como un pjaro de oro, y la brisa trazaba en el terciopelo de la yerba, huellas ideales y quimricas como si danzasen increbles hadas.
Ramn M. DEL VALLE-INCLN Sonata de otoo

MATERIAL FOTOCOPIABLE / Oxford University Press Espaa, S. A.

1 Indica qu se describe en cada uno de los textos y qu aspectos o elementos se tratan. 2 Qu tipo de descripciones son segn la perspectiva del emisor? Menciona las caractersticas lingsticas que demuestran la pertenencia a una clase u otra.

3 Analiza los rasgos lingsticos generales propios de la modalidad descriptiva y ejemplifcalos con enunciados de los textos. 4 Reconoce los recursos expresivos que aparezcan en los fragmentos.

10.

Las modalidades textuales

165

3. Descripciones literarias

10

ACTIVIDADES

REFUERZO

10

4. Textos explicativoargumentativos
El desarrollo de las nuevas tecnologas ha producido cambios enormes, no solo en el campo del conocimiento y de las comunicaciones, sino tambin en la vida cotidiana de las personas. El siguiente texto de Joan Maj ofrece unas reflexiones sobre las posibilidades de la llamada revolucin digital para crear un mundo mejor.

La sociedad del conocimiento: un mundo mejor?


Hemos aprendido a hacer lo que queramos con la informacin. A generar toda la que deseemos, a almacenarla, a transmitirla, sin ningn lmite desde el punto de vista del espacio, del tiempo o de la velocidad. Como consecuencia de ello hemos pasado de un mundo en el que la informacin era un recurso escaso, a otro en el que es un recurso abundante, incluso a menudo excesivo. Tambin hemos pasado de un mundo en el que la informacin era un recurso estable, a otro en el que es un recurso enormemente cambiante. Puesto que la informacin es la materia prima para que nuestro cerebro elabore conocimientos, cada vez estamos en condiciones de adquirir ms (siempre que no nos dejemos ahogar por el exceso de informacin y siempre que aprendamos a actualizar constantemente los conocimientos que van quedando anticuados!). Si esto es lo que ha ocurrido, vamos a ver qu cambios produce y producir en nuestra sociedad. Llamamos a esta sociedad futura la sociedad del conocimiento porque este va a ser un elemento muy abundante, pero sobre todo porque va a ser el factor clave de nuestro bienestar. Su gran cantidad, disponibilidad y capacidad innovadora han convertido el conocimiento en un recurso econmico fundamental. El acceso al conocimiento se ha convertido en un elemento clave de la estrategia de las empresas y en un componente fundamental de nuestro bienestar personal. Las empresas sustituyen constantemente la utilizacin de recursos materiales por la aplicacin de nuevos conocimientos y nosotros vamos aumentando ms y ms la parte de nuestro presupuesto personal que dedicamos a acceder a elementos inmateriales, al tiempo que disminuimos nuestro consumo de elementos materiales. Pensemos en la diferencia entre comprar un disco (almacn de material de msica) o escuchar msica a la carta a travs de Internet o de un canal especializado. O bien en la diferencia existente entre comprar un reloj (una mquina de medir el tiempo) para saber la hora, o tener acceso a la hora a travs de la red. Si este proceso de desmaterializacin de nuestra vida tiene lugar, va a suponer la posibilidad de cambios muy positivos en la organizacin social, porque una economa basada en gran medida en el conocimiento tiene unas reglas de funcionamiento distintas de la economa industrial. [] A partir del momento en el que una parte importante de nuestro bienestar (nunca todo, evidentemente) comienza a depender del acceso a bienes inmateriales, el problema de la escasez y el del reparto adquieren otra dimensin. Deja de ser un problema la escasez, porque el conocimiento es reproducible, y cambia el problema del reparto porque no hay que renunciar a algo para que otro pueda tenerlo. Esto abre unas perspectivas sociales muy esperanzadoras.
Joan MAJ Clo, abril de 2002, n. 6

1 Qu cambios, segn Joan Maj, se han producido en la informacin debido a las nuevas tecnologas? 2 En qu se funda la denominacin de la sociedad del conocimiento? 3 Qu cambios econmicos y sociales produce el acceso a la informacin mediante las nuevas tecnologas? 4 Realiza el resumen del texto.

5 Determina la estructura del fragmento y reconoce las caractersticas lingsticas propias de este tipo de textos. 6 Identifica los argumentos sobre los que se sostiene la conclusin del texto. 7 Escribe un texto reflexivo donde se presenten argumentos referidos a la dificultad o el impedimento de lo expresado en la conclusin (difusin de la red, posibilidades de acceso y regulacin del mismo, instalacin de lneas).

166 10.

Las modalidades textuales

MATERIAL FOTOCOPIABLE / Oxford University Press Espaa, S. A.

ACTIVIDADES

REFUERZO

10

5. Argumentaciones
La argumentacin constituye una modalidad imprescindible en un ensayo, ya que es el tipo de texto en que se exponen y defienden unas ideas determinadas sobre un tema. A continuacin se citan fragmentos en los que se utilizan distintos tipos de argumentos.

Texto 1
Resulta sorprendente que los especialistas en miedos hayan estudiado tan poco las tcticas de amedrentamiento, los mecanismos de utilizacin del temor, porque son la contrafigura del miedo. Hay terrores no intencionados y terrores provocados voluntariamente, estratgicos. La erupcin de un volcn, el feroz encrespamiento de un tsunami o una epidemia de peste no pretenden suscitar miedo. Lo causan, sin ms. Pero la arbitrariedad de un Estado policiaco, los atentados terroristas, los chantajes de cualquier tipo, las crueldades domsticas, los ataques preventivos, las operaciones de escarmiento se sirven del miedo para alcanzar unos objetivos.

Texto 3
El ser humano siente miedo y responde psicolgicamente al miedo con mecanismos muy prximos a los que usan los animales: huida, ataque, inmovilidad y sumisin. Biolgicamente, el miedo no plantea ningn problema. Qu otra cosa va a hacer el ciervo sino huir del leopardo? Qu otra cosa va a hacer el escarabajo sino hacerse el muerto cuando lo toco? Son respuestas adaptativas eficaces para todos los animales. Pero el hombre no se encuentra cmodo en esas rutinas tan contrastadas. El ser humano quiere vivir por encima del miedo. Sabe que no puede eliminarlo, sin caer en la locura o en la insensibilidad, como ya deca Aristteles, pero quiere actuar a pesar de l. Aqu se revela nuestra naturaleza paradjica: no podemos vivir sin que nuestros sentimientos nos orienten, pero no queremos vivir a merced de nuestros sentimientos.

Texto 2
En el mundo animal, los cazadores intentan aislar a su presa o buscar presas solitarias. El grupo es la gran defensa. En los seres humanos sucede lo mismo, por ello es fcil comprender que todas las dictaduras anulen el ejercicio del derecho de reunin. En su teora relacional del valor, Judith Jordan sostiene que la valenta no es un rasgo interno, sino que surge en la conexin con los dems: As human beings we encoura one another. Nos animamos unos a otros. [] Los estudios sobre la violencia de gnero demuestran que una de las tcticas del violento, del que pretende la sumisin de la otra persona, es aislarla, obligarla a romper los vnculos con su familia, anular su posibilidad de comunicarse con los dems, obligarla a prescindir de todos los sistemas de apoyo social que podran ayudarla. El proceso suele comenzar con crticas a la familia de la vctima, prohibicin de visitas, reclusin en casa, celos, tcnicas de aislamiento, en una palabra.

Texto 4
Nuestra bsqueda de la felicidad es con frecuencia desgarradora, porque estamos movidos por dos deseos contradictorios: el bienestar y la superacin. Necesitamos estar cmodos y necesitamos crear algo de lo que nos sintamos orgullosos y por lo que nos sintamos reconocidos. Una actividad que d sentido a nuestra existencia, por muy ilusorio que sea ese sentido. Tenemos, pues, que armonizar anhelos contradictorios. Necesitamos construir la casa y descansar en ella. Necesitamos estar refugiados en el puerto y navegando.
Jos Antonio MARINA Anatoma del miedo, Crculo de Lectores

MATERIAL FOTOCOPIABLE / Oxford University Press Espaa, S. A.

1 En el texto 1 el autor diferencia dos tipos de miedo, cules son y cmo justifica esa distincin? 2 En el texto 2 se afirma que el aislamiento acrecienta el miedo. Menciona los argumentos (y sus tipos) con que desarrolla esta idea. 3 A qu tema se refiere el segundo prrafo del texto 3?Qu relacin tiene con lo explicado anteriormente?

4 Con qu afirmacin se inicia el texto 3? Qu comparacin se realiza? Seala qu tipos de interrogaciones son las que se incluyen a continuacin. Qu contraste se establece posteriormente? 5 Qu tipo de recursos expresivos cierran el texto 4? Cul es su sentido y cmo se relacionan con las ideas anteriores?

10.

Las modalidades textuales

167

ACTIVIDADES

ORTOGRAFA

10

6. Uso de c y z
Palabras con cc y c La letra c representa dos fonemas distintos: el oclusivo velar sordo ante -a, -o y -u (caro, coral, cuyo), ante consonante y en posicin final de slaba o de palabra (acto, vivac); y el fricativo interdental sordo ante -e, -i (cereza, ciento). En las palabras con cc, la primera representa el sonido oclusivo velar y la segunda el fricativo interdental y adems pertenecen a slabas distintas: correc-cin. En muchos casos, si en una familia de palabras existe un morfema lxico con ct, las palabras derivadas terminadas en -cin se escriben con cc: atractivo, atraccin; ficticio, ficcin; redactar, redaccin. Palabras con z La letra z representa el fonema fricativo interdental sordo ante -a, -o, -u (zapato, zorro, zumo) y en posicin final de slaba o de palabra (pez). En este ltimo caso, el plural se escribe con c (peces). La regla general presenta bastantes excepciones: zjel, zigzag, zigzagueo, enzima (la), nazi, nazismo, zepeln, zipizape, zen Se escriben con z las formas irregulares de la 1. persona del singular del presente de indicativo y todo el presente de subjuntivo de los verbos terminados en -acer, -ecer, -ocer y -ucir (merezco, merezcas). Algunas palabras pueden escribirse con c o con z, aunque la RAE prefiere la variante escrita con c: cebra / zebra, cenit / zenit, eccema / eczema, cigoto / zigoto, zinc / cinc, cedilla / zedilla. En las zonas donde existe el seseo, c (ante e, i) y z representan el sonido de s.

1 Escribe el plural de las siguientes palabras: nuez juez vez veloz raz tenaz infeliz capaz 2 Escribe el singular de paredes, voluntades, rapaces, veraces, disfraces. 3 Escribe las familias de palabras de objeto y abstraccin. 4 Escribe c, cc o z, segn corresponda:
a) diale to b) infe in c) cara terstica d) inspe in e) rela iones f) conjun in g) afli in h) a eso i) ju gar j) afe in k) co in l) defla in m) a in n) fi in ) t ita o) precau in p) fra in q) interje in r) discre in s) obje in t) produ in u) indu in v) convi in w) sele in

6 Escribe los trminos derivados formados con el morfema -cin que pertenezcan a las familias de las palabras que figuran a continuacin: a) sujetar j) inhalar r) inducir b) conducir k) deducir s) actuar c) cocer l) afectar t) asociar d) contraer m) situar u) reducir e) corregir n) exhalar v) construir f) sustraer ) redactar w) destruir g) inventar o) satisfacer x) dirigir h) distraer p) instruir y) inyectar i) concretar q) proyectar z) relacionar 7 Escribe la 1. y la 3. persona del singular y del plural del presente de indicativo y del subjuntivo de los siguientes verbos: reconocer, pertenecer, reducir, crecer, conducir, convencer. 8 Conjuga en la 1. persona singular del presente de subjuntivo y del pretrito perfecto simple de indicativo de utilizar, empezar, realizar y generalizar. 9 Forma verbos parasintticos a partir de las palabras flaco, grande, loco, noble, dulce. A continuacin, conjgalos en la 1. persona del singular del presente de indicativo y de subjuntivo. 10 Escribe verbos acabados en -ocer y -ucir y conjgalos en las mismas personas de la actividad 9.
MATERIAL FOTOCOPIABLE / Oxford University Press Espaa, S. A.

5 Crea oraciones con cada una de estas tres parejas de palabras: a) Adicin / adiccin. b) Inflacin / infraccin c) aficin - afeccin

168 10.

Las modalidades textuales

Evaluacin 10

xito de taquilla, por Dale Kennington.

El valor de las opiniones


En vez de ser consideradas propuestas imprecisas, limitadas por la insuficiencia de conocimientos o el apresuramiento, las opiniones se convierten en expresin irrebatible de la personalidad del sujeto: esta es mi opinin, eso ser su opinin, como si lo relevante en ellas fuese a quin pertenecen en lugar de en qu se fundan. La antigua y poco elegante frase que suelen decir los tipos duros de algunas pelculas yanquis l as opiniones son como los culos: cada cual tiene la suya cobra vigencia, porque ni de las opiniones ni de los traseros cabe por lo visto discusin alguna ni nadie puede desprenderse ni de unas ni de otro aunque lo quisiera. A ello se une la obligacin beatfica de respetar las opiniones ajenas, que si de verdad se pusiera en prctica paralizara cualquier desarrollo intelectual o social de la humanidad. Por no hablar del derecho a tener su opinin propia, que no es el de pensar por s mismo y someter a confrontacin razonada lo pensado sino el de mantener la propia creencia sin que nadie interfiera con molestas objeciones. Este subjetivismo irracional cala muy pronto en nios y adolescentes, que se acostumbran a suponer que todas las opiniones es decir, la del maestro que sabe de lo que est hablando y la suya, que parte de la ignorancia valen igual y que es seal de personalidad autnoma no dar el brazo a torcer y ejemplo de tirana tratar de convencer al otro de su error con argumentos e informacin adecuada. La tendencia a convertir las opiniones en parte simblica de nuestro organismo y a considerar cuanto las desmiente como una agresin fsica (ha herido mis convicciones!) no solo es una dificultad para la educacin humanstica sino tambin para la convivencia democrtica. Vivir en una sociedad plural impone asumir que lo absolutamente respetable son las personas, no sus opiniones, y que el derecho a la propia opinin consiste en que esta sea escuchada y discutida, no en que se la vea pasar sin tocarla como si de una vaca sagrada se tratase.
Fernando SAVATER El valor de educar, Ariel

1. Redacta un resumen del contenido del texto de Fernando


Savater.

6. Enuncia las finalidades de los textos expositivos y argumentativos y cita mensajes escritos con tales modalidades.

2. Determina el tema (o los temas) que se desarrollan en el


fragmento.

7. En qu tipos de textos orales y escritos pueden aparecer la narracin y la descripcin? Con qu objetivo?

3. A qu modalidad textual corresponde el texto? Justifica


tu respuesta.

8. Realiza el anlisis morfolgico de las siguientes palabras: imprecisas, absolutamente, acostumbran.

4. Establece la estructura y, segn corresponda, los procedimientos que se utilizan para desarrollar la informacin o la tesis y los argumentos que sustentan las afirmaciones del autor.

9. Realiza el anlisis sintctico de las siguientes oraciones.


a) Las opiniones se convierten en expresiones irreba tibles de la personalidad del sujeto. b) A ello se une la obligacin beatfica del respeto a las opiniones ajenas.

5. Reconoce en el texto las caractersticas propias de la


modalidad empleada y las funciones del lenguaje que predominan.

10. Escribe una reflexin crtica sobre el tema del fragmento.


Formula una tesis y desarrolla los argumentos. 159 10.
Las modalidades textuales

ACTIVIDADES

REFUERZO

11

1. El gnero lrico
A continuacin puedes leer un poema de Blas de Otero (1916-1979) y otro de Gloria Fuertes (1918-1998). Ambos autores pertenecen a la poesa social, que se desarroll en Espaa a partir de los aos cincuenta.

Djame
Me haces dao, Seor. Quita tu mano de encima. Djame con mi vaco, djame. Para abismo, con el mo tengo bastante. Oh Dios, si eres humano, compadcete ya, quita esa mano de encima. No me sirve. Me da fro y miedo. Si eres Dios, yo soy tan mo como t. Y a soberbio, yo te gano. Djame. Si pudiese yo matarte, como haces t, como haces t! Nos coges con las dos manos, nos ahogas. Matas no se sabe por qu. Quiero cortarte las manos. Esas manos que son trojes del hambre, y de los hombres que arrebatas.
Blas DE OTERO Poesa espaola contempornea (1939-1980), Alhambra

10

El planeta tierra
El planeta tierra debera llamarse planeta agua. En la tierra hay ms agua que cuerpo, en el cuerpo hay ms cuerpo que alma, en la tierra hay ms peces que aves, en las aves ms plumas que alas. En el verso hay ms sangre que tinta, en la tinta ms sombra que nada, en la nada hay ms algo que alga y ese algo se mueve y reluce y nace la palabra.

MATERIAL FOTOCOPIABLE / Oxford University Press Espaa, S. A.

10

Gloria FUERTES Poesa espaola contempornea (1939-1980), Alhambra

1 Justifica la adscripcin de estos dos textos al gnero lrico. 2 Localiza en el poema de Blas de Otero al emisor y al receptor y explica el contenido del mensaje. 3 Explica la mtrica de ambos textos: la rima, la medida de los versos, el agrupamiento estrfico y el tipo de poema.

4 Qu recurso expresivo abunda en el poema Djame? Localiza, adems, un apstrofe y un caso de similicadencia. 5 Cul es el mensaje del poema de Gloria Fuertes? Qu crees que significa: En el verso hay ms sangre que tinta? 6 Qu recurso expresivo de repeticin predomina en el segundo poema?

11.

La literatura y los textos literarios

181

ACTIVIDADES

REFUERZO

11

2. El gnero narrativo
El siguiente relato pertenece al escritor argentino Jorge Luis Borges (1899-1986), quien ha destacado tanto por sus cuentos como por sus poemas. Entre sus obras narrativas se encuentran El jardn de los senderos que se bifurcan, El Sur, El libro de arena y El Aleph, al que pertenece este cuento.

Los dos reyes y los dos laberintos


Cuentan los hombres dignos de fe (pero Al sabe ms) que en los primeros das hubo un rey de las islas de Babilonia que congreg a sus arquitectos y magos y les mand construir un laberinto tan perplejo y sutil que los varones ms prudentes no se aventuraban a entrar, y los que entraban se perdan. Esa obra era un escndalo, porque la confusin y la maravilla son operaciones propias de Dios y no de los hombres. Con el andar del tiempo vino a su corte un rey de los rabes, y el rey de Babilonia (para hacer burla de la simplicidad de su husped) lo hizo entrar en el laberinto, donde vag afrentado y confundido hasta la declinacin de la tarde. Entonces implor socorro divino y dio con la puerta. Sus labios no profirieron queja ninguna, pero le dijo al rey de Babilonia que l en Arabia tena otro laberinto y que, si Dios era servido, se lo dara a conocer algn da. Luego regres a Arabia, junt a sus capitanes y sus alcaides y estrag los reinos de Babilonia con tan venturosa fortuna que derrib sus castillos, rompi sus gentes e hizo cautivo al mismo rey. Lo amarr encima de un caballo veloz y lo llev al desierto. Cabalgaron tres das, y le dijo: Oh, rey del tiempo y sustancia y cifra del siglo!, en Babilonia me quisiste perder en un laberinto de bronce con muchas escaleras, puertas y muros; ahora el Poderoso ha tenido a bien que te muestre el mo, donde no hay escaleras que subir, ni puertas que forzar, ni fatigosas galeras que recorrer, ni muros que te veden el paso. Luego le desat las ligaduras y lo abandon en mitad del desierto, donde muri de hambre y de sed. La gloria sea con Aquel que no muere.
Jorge Luis BORGES Obras completas, Emec
MATERIAL FOTOCOPIABLE / Oxford University Press Espaa, S. A.

1 Justifica por qu podemos afirmar que estamos en presencia de un texto narrativo. 2 A qu subgnero narrativo pertenece este texto de Jorge Luis Borges? Argumenta tu respuesta. 3 Resume el contenido de la historia narrada por Borges. 4 Explica los conceptos de tiempo de la historia y tiempo del discurso a partir de este texto.

5 Qu movimientos narrativos reconoces en el texto? 6 Caracteriza al narrador y su perspectiva. 7 Seala en el cuento ejemplos de discurso referencial, valorativo, universal, directo e indirecto. 8 En el discurso directo presente en el fragmento tambin puede observarse discurso potico; explica por medio de qu recurso expresivo se materializa.

182 11.

La literatura y los textos literarios

ACTIVIDADES

REFUERZO

11

3. El gnero dramtico
El siguiente fragmento pertenece a uno de los principales dramaturgos espaoles del siglo XX, Antonio Buero Vallejo. Entre sus principales obras se encuentran Historia de una escalera (1949), que abri un nuevo panorama en el drama de posguerra, En la ardiente oscuridad (1950), Casi un cuento de hadas (1953), Irene o el tesoro (1954), El concierto de San Ovidio (1962), El tragaluz (1967) y La fundacin (1974).

La confusin
ALBERTO.[] Se oye un bastn. JUANA.Tambin yo lo oigo. (Todos atienden. Pausa. Por la derecha, tanteando el suelo con su bastn y con una expresin de vago susto, aparece IGNACIO. Es un muchacho delgaducho, serio y reconcentrado, con cierto desalio en su persona: el cuello de la camisa desabrochado, la corbata floja, el cabello peinado con ligereza. Viste de negro intemporalmente, durante toda la obra. Avanza unos pasos indeciso y se detiene.) LOLITA.Qu raro! (IGNACIO se estremece y retrocede un paso.) MIGUEL.Quin eres? (Temeroso, IGNACIO se vuelve para salir por donde entr. Despus, cambia de idea y sigue hacia la izquierda, rpido.) ANDRS.No contestas? (IGNACIO tropieza con el silln de JUANA. Tiende el brazo y ella coge su mano.) MIGUEL.(Levantndose.) Espera, hombre! No te marches. (Se acerca a palparle, mientras JUANA dice, inquieta:) JUANA.Me ha cogido la mano No le conozco. (IGNACIO la suelta y MIGUELN le sujeta por un brazo.) MIGUEL.Ni yo.
MATERIAL FOTOCOPIABLE / Oxford University Press Espaa, S. A.

LOLITA.(Riendo.) Te ha salido un competidor, Migueln. ESPERANZA.Un competidor? Un maestro! ALBERTO.Debe de ser algn gracioso del primer curso. MIGUEL.Dejdmelo a m. Qu has dicho que eres? IGNACIO.(Asustado.) Un ciego. MIGUEL.Oh, pobrecito, pobrecito! Quiere que le pase a la otra acera? (Los dems se desternillan.) Largo, idiota! Vete a rer de los de tu curso. ANDRS.Realmente la broma es de muy mal gusto. Anda, mrchate. (Lo empujan. IGNACIO retrocede hacia el proscenio.) IGNACIO.(Violento, quiz al borde del llanto.) Os digo que soy ciego! MIGUEL.Qu bien te has aprendido la palabrita! Largo! (Avanzan hacia l, amenazadores. ALBERTO se levanta tambin.) IGNACIO.Pero, es que no lo veis? MIGUEL.Cmo? (JUANA y CARLOS, que comentaban en voz baja el incidente, intervienen.) CARLOS.Creo que estamos cometiendo un error muy grande, amigos. l dice la verdad. Sentaos otra vez. MIGUEL.Atiza! CARLOS.(Acercndose con JUANA a IGNACIO.) Nosotros tambin somos ciegos, como t dices.
Antonio BUERO VALLEJO En la ardiente oscuridad, Magisterio Espaol

(ANDRS se levanta y se acerca tambin para cogerle por el otro brazo.) IGNACIO. (Con temor.) Djame. ANDRS.Qu buscas aqu? IGNACIO.Nada. Dejadme. Yo soy un pobre ciego.

1 Explica lo que ocurre en este fragmento de la obra En la ardiente oscuridad, de Antonio Buero Vallejo. 2 Justifica la adscripcin de esta obra al gnero dramtico. 3 En qu subgnero lo incluiras? Argumenta tu respuesta.

4 Diferencia en el fragmento el texto principal y el texto secundario. 5 A qu hacen referencia las acotaciones que aparecen? 6 Transforma el fragmento La confusin en un texto narrativo. Recuerda que puedes incluir dilogo en l.

11.

La literatura y los textos literarios

183

Evaluacin

11

Los espritus de la tormenta, por Evelyn de Morgan.

Rima LII
Olas gigantes que os rompis bramando en las playas desiertas y remotas, envuelto entre la sbana de espumas, llevadme con vosotras! Rfagas de huracn que arrebatis del alto bosque las marchitas hojas, arrastrado en el ciego torbellino, llevadme con vosotras! Nubes de tempestad que rompe el rayo y en fuego encienden las sangrientas orlas, arrebatado entre la niebla oscura, llevadme con vosotras! Llevadme por piedad a donde el vrtigo con la razn me arranque la memoria. Por piedad! tengo miedo de quedarme con mi dolor a solas! 10

15

Gustavo Adolfo BCQUER Libro de los gorriones, Planeta

1. Justifica por qu la Rima LII es un texto literario y por


qu pertenece al gnero lrico.

7. Consulta el anexo de las pginas 352-353 de este libro


y seala los recursos de repeticin que se emplean en el texto de Bcquer.

2. Identifica al emisor y a los receptores del proceso comunicativo interno del poema y comenta sus caractersticas.

8. Reconoce en esa misma composicin otros recursos


expresivos.

3. Qu actitud ofrece la voz potica en esta composicin?


Explica por qu.

9. Qu rasgos debera presentar esta composicin de


Gustavo Adolfo Bcquer para pertenecer a cada uno de los otros dos gneros literarios?

4. Explica el contenido del mensaje en esta rima. 5. Qu tema desarrolla? Indica qu motivos aparecen. 6. Analiza la mtrica. Atiende a la medida de los versos, al
esquema y el tipo de rima y al agrupamiento en estrofas. 177 11.

10. Redacta el comentario de la Rima LII del Libro de los


gorriones integrando todos los aspectos que has analizado en las respuestas anteriores.
La literatura y los textos literarios

ACTIVIDADES

AMPLIACIN

12

1. Lrica culta gallego-portuguesa. Las Cantigas de Santa Mara


Aunque se admite que el rey Alfonso X el Sabio es el autor de algunas cantigas, se supone que la mayora fueron creadas por un reducido grupo de escritores cortesanos. De las 420 Cantigas de Santa Mara, solo 64 son de carcter lrico; la mayor parte son narrativas y relatan milagros de la Virgen. Todas van acompaadas de melodas y de preciosas miniaturas que permiten conocer aspectos de la vida cotidiana de la poca. Adems de la Introduccin y los dos prlogos, el resto de las cantigas lricas son de loor o alabanza a la Virgen. Entre ellas predominan las de tipo hmnico, que celebran a Mara como auxiliadora o se refieren a festividades marianas y cristolgicas. En las Cantigas se registran cerca de 300 combinaciones mtricas distintas, de las cuales las ms irregulares corresponden a las de carcter lrico. Los versos varan entre las dos y las veinticuatro slabas, y entre ellos destaca el alejandrino. En cuanto a las estrofas empleadas, predomina el zjel.

Cantiga 10
ESTA DE LOOR DE SANTA MARA, COM FREMOSA E BOA E GRAN PODER Rosas das rosas e Fror das frores, Dona das donas, Sennor das sennores. Rosa de beldad e de parecer e Fror d alegria e de prazer, Dona en mui piadosa seer, Sennor en toller coitas e doores. Rosa das rosas e Fror das frores A tal Sennor dev ome muit amar, que de todo mal o pode guardar; e pode-ll os peccados perd oar, que faz no mundo per maos sabores. Rosa das rosas e Fror das frores Devemo-la muit amar e servir, ca punna de nos guardar de falir; des i dos erros nos faz repentir, que nos fazemos come pecadores. Rosa das rosas e Fror das frores
MATERIAL FOTOCOPIABLE / Oxford University Press Espaa, S. A.

Rosa de las rosas y Flor de las flores, Seora de las seoras, Seor de los seores. Rosa de beldad y jucio y Flor de alegra y placer, Seora muy piadosa en su ser, Seor en quitar penas y dolores. Rosa de las rosas A tal Seor debe el hombre mucho amar, que de todo mal lo puede guardar, y puede perdonarle los pecados, que acta en el mundo contra los sinsabores. Rosa de las rosas La debemos amar mucho y servir, porque lucha por guardarnos de caer; de los errores nos hace arrepentir, que nosotros hacemos como pecadores. Rosa de las rosas Esta seora que tengo por Seor y de quien quiero ser trovador, si yo por fortuna puedo tener su amor, doy al demonio otros amores. Rosa de las rosas

10

10

15

15

Esta dona que tenno por Sennor e de que quero seer trobador, se eu per ren poss aver seu amor, dou ao demo os outros amores. Rosa das rosas e Fror das frores

20

20

Alfonso X EL SABIO Cantigas de Santa Mara, Castalia

Traduccin de los autores

1 Por qu pertenece a la lrica la Cantiga 10? Para contestar, consulta la UNIDAD 11 y justifica tu respuesta. 2 Explica cmo se presenta la figura de la Virgen en este poema mariano.

3 Analiza la mtrica y los recursos expresivos empleados en la cantiga. 4 Qu composiciones de la lrica culta pueden haber influido en las Cantigas? Para responder, apyate en ejemplos extrados de la Cantiga 10.

12.

La lrica medieval

193

ACTIVIDADES

REFUERZO

12

2. La lrica primitiva popular


En la Pennsula, la lrica primitiva popular se manifiesta en tres tipos de composiciones: las jarchas, las cantigas de amigo y los villancicos. Repasa las caractersticas formales y temticas de estas producciones literarias y despus lee detenidamente los siguientes textos.

Texto 1
Non dormireyo, mamma, a rayo de maana. Bon Abu-l-Qasim, la facha de matrana. No dormir, madre, al rayar la maana. El buen Abu-l-Qasim, la cara de la aurora.

Texto 5
Mia irmana fremosa, treydes comigo a la igreja de Vig, u o mar salido: E miraremos las ondas! Mia irmana fremosa, treydes de grado a la igresa de Vig, u o mar levado: E miraremos las ondas! A la igresa de Vig, u o mar salido, e verr i, mia madre, o meu amigo: E miraremos las ondas! A la igresa de Vig, u o mar levado, e verr y, mia madre, o meu amado: E miraremos las ondas! 10 5

Texto 2
Quiero dormir y no puedo, que el amor me quita el sueo.

Texto 3
Qu far, mamma? Meu al-habib est ad yana. Qu har, madre? Mi amigo est en la puerta.

(Invita a su hermana a visitar la iglesia de Vigo durante la marea alta, y dice a su madre que all ir su amigo.)
Antologa de la poesa gallego-portuguesa Alhambra

Texto 4
Con amores, mi madre, con amores me adorm.
Lrica espaola de tipo popular Ctedra

1 Identifica entre estos cinco poemas cules son jarchas. En qu aspecto o aspectos te has basado para reconocerlas? 2 Caracteriza el emisor, receptor y contenido del mensaje en cada una de ellas. 3 Analiza el nmero de versos y la rima empleada en estas composiciones. 4 En una de las jarchas figura una metfora; localzala y explica su valor. 5 Entre los poemas se incluyen tambin villancicos castellanos. Selalos y explica en qu te has basado para reconocerlos.

6 Establece el proceso de la comunicacin en esos poemas: emisor, receptor y contenido del mensaje. 7 Qu tipo de poema configura la cantiga de amigo? Para responder, consulta, si es necesario, el apndice (pginas 358 y 359) y la pgina 184 de tu libro de texto. 8 Establece las semejanzas de contenido entre el poema del texto 5, las jarchas y los villancicos; seala, igualmente, las semejanzas y diferencias formales entre estos tres tipos de composiciones de la lrica primitiva popular.

194 12.

La lrica medieval

MATERIAL FOTOCOPIABLE / Oxford University Press Espaa, S. A.

ACTIVIDADES

REFUERZO

12

3. La lrica del siglo XV


El siglo XV es una poca de eclosin de la poesa. El Romancero constituye un hito en nuestra tradicin literaria y crea una tipologa que permanece hasta la actualidad. La poesa culta de los cancioneros ser la fuente de la que bebern los poetas renacentistas.

Texto 1
Y pues vemos lo presente cmo en un punto se es ido y acabado, si juzgamos sabiamente, daremos lo non venido por pasado. No se engae nadie, no, pensando que a de durar lo que espera ms que dur lo que vio, porque todo ha de pasar por tal manera.

Texto 2
Tus casos falaces, Fortuna, cantamos, estados de gentes que giras y trocas, tus grandes discordias, tus firmezas pocas, y los que en tu rueda quejosos hallamos, hasta que al tiempo de ahora vengamos; de hechos pasados codicia mi pluma, y de los presentes, hacer breve suma; d fin Apolo, pues nos comenzamos.

10

Jorge MANRIQUE Poesa, Crtica

Juan DE MENA El laberinto de la Fortuna, Espasa Calpe

Texto 3
Ven, muerte, tan escondida que no te sienta conmigo, porque el gozo de contigo no me torne a dar la vida. Ven como rayo que hiere: que hasta que ha herido no se siente su ruido, por mejor herir do quiere. As sea tu venida; si no desde aqu me obligo que el gozo que habr contigo me dar de nuevo vida. 5

Texto 4
Vos me matis de tal suerte, con pena tan glorosa, que no s ms dulce cosa que los trances de mi muerte. Y de ella soy tan ufano, tan penado y tan contento, que no trocar un tormento por mil bienes de otra mano. Y pues que quiso mi suerte darme pena gloriosa, no quiero ms dulce cosa que los trances de mi muerte. 5

Texto 5
Soy contento, vos servida, ser penado de tal suerte, que por vos quiero la muerte mas que no, sin vos, la vida. Quiero ms por vos tristura, siendo vuestro sin mudanza, que placer sin esperanza de enamorada ventura. No tengis la fe perdida pues la tengo yo tan fuerte, que por vos quiero la muerte mas que no, sin vos, la vida. 5

10

10

10

Poesa medieval castellana Taurus

Poesa medieval castellana Taurus

Poesa medieval castellana Taurus

MATERIAL FOTOCOPIABLE / Oxford University Press Espaa, S. A.

1 Reconoce en el texto 1 la mtrica empleada por su autor, Jorge Manrique, en las Coplas por la muerte de su padre y seala en la estrofa otras caractersticas de esta obra. 2 Quin es el receptor del texto 2? Qu le dice la voz potica? Explica qu intencin manifiesta en los tres ltimos versos. 3 Reconoce la mtrica empleada por Juan de Mena en la obra a la que pertenece la estrofa del texto 2 y seala en ella otras caractersticas de esta obra.

4 A qu tipo de composiciones corresponden los otros tres textos? Explica su mtrica y su estructura. Texto 1 Texto 2 Texto 3 5 Explica brevemente el contenido de cada uno de los poemas reproducidos en esta pgina. 6 Seala en los textos 3, 4 y 5 los recursos expresivos caractersticos de este tipo de composiciones.

12.

La lrica medieval

195

ACTIVIDADES

LITERATURA UNIVERSAL

12

4. El arte de los trovadores


El arte de los trovadores provenzales fue producto de un largo proceso de aprendizaje de potica, retrica y msica. Ya desde la primera mitad del siglo XII se diferencian claramente dos grupos de poetas segn su modo de expresin: los partidarios de una poesa sencilla el trobar leu y los que cultivan un arte ms complejo el trobar clus y el trobar ric.

El trobar leu
El trobar leu o arte de la versificacin sencilla fue practicado, entre otros, por el poeta Bernart de Ventadorn, que ha sido considerado el primero de los trovadores amorosos de la literatura provenzal. A l pertenece este fragmento prosificado.

Texto 1
Tengo mi corazn tan lleno de alegra que todo me lo transfigura: el fro me parece flor blanca, roja y amarilla, pues con el viento y con la lluvia me crece la ventura; por lo que mi mrito aumenta y sube y mi canto mejora. Tanto amor tengo en el corazn, tanta alegra y dulzura, que el hielo me parece flor y la nieve verdor.
Bernart de VENTADORN en Historia de la literatura universal, Planeta

El trobar clus y el trobar ric


Entre los poetas partidarios del arte de la poesa difcil, algunos practicaron el denominado trobar clus o cerrado, que utilizaba un lenguaje sumamente enigmtico, recargado de conceptos, aunque a veces incorporaba tambin un registro popular y bajo. Otro grupo de trovadores, por el contrario, se volcaron hacia el trobar ric, un arte ms complejo en el que se atenda esencialmente a la belleza formal, a la sonoridad, a la sugerencia del sonido y el ritmo, y a la seleccin de un vocabulario inusual. El trobar clus surgi entre poetas realistas y de condicin humilde, que rechazaban el amor corts, como Marcabr. El trobar ric, en cambio, posea un carcter aristocrtico, aceptaba los postulados de ese cdigo amoroso y no era en absoluto moralizante. Estos rasgos se observan, por ejemplo, en la obra de Arnaut Daniel.

Texto 2
Homicidas y traidores, simonacos1, saludadores, lujuriosos y usureros que viven de ruines menesteres, y los que hacen hechiceras y las repugnantes hechiceras estarn por igual en el fuego ardiente. Borrachos y cornudos, falsos curas y falsos abades, falsas reclusas y falsos reclusos, all penarn, dice Marcabr.
MARCABR

Texto 3
El aura amarga hace aclarar los bosquecillos ramosos, que la dulce espes con hojas, y mantiene balbucientes y mudos los alegres picos de los pjaros de las ramas, aparejados y no aparejados. Por qu yo me esfuerzo en hacer y decir cosas agradables a muchos? Por aquella que me ha vuelto de arriba abajo, de lo que temo morir si no me da fin a los afanes.
Arnaut DANIEL en Historia de la literatura universal Planeta

simonaco: el que compra o vende bienes espirituales.


1

1 Qu sentimiento embarga al yo potico del texto 1? Qu lo causa y qu efecto produce? 2 A quines ataca especialmente el poeta Marcabr en el fragmento seleccionado?

3 Qu anhela el emisor de la composicin de Arnaut Daniel? 4 Explica qu provoca en el emisor de texto 3 el estado en el que se encuentra.

196 12.

La lrica medieval

MATERIAL FOTOCOPIABLE / Oxford University Press Espaa, S. A.

Evaluacin 12
Texto 1
Plega Dios que alguno quieras como yo, mi bien, te quiero, porque penes, porque mueras, porque sepas que de veras por tu sola causa muero. Amada ni ms ni menos seas como yo de ti, porque defectos ajenos te hagan mencin de m; y penes porque debieras quererme como te quiero; y por me cobrar te mueras y no puedas aunque quieras siendo yo muerto primero.

10

Pedro LVAREZ OSORIO CONDE DE TRASTMARA y MARQUS DE ASTORGA Poesa espaola 2. Edad Media: Lrica y cancioneros Crtica

Ilustracin de la Divina comedia, de Dante Alighieri.

Texto 2
Ojal alcanzara la unin mi corazn, que est en continua angustia! Todo corazn que arde en la pasin del amor est inquieto por la agitacin que sufre y que deshace al que en ansias se acongoja.
Ibn RUHAYM en F. CORRIENTE, Poesa dialectal rabe y romance en al-ndalus Gredos

Texto 4
A quin contar yo mis quejas, mi lindo amor? A quin contar yo mis quejas si a vos no?
Lrica espaola de tipo popular, Ctedra

Texto 5
Qu se hizo la moneda que guard para mis daos tantos tiempos, tantos aos, plata, joyas, oro o seda? Ca de todo no me queda sino este cadahalso. Mundo malo, mundo falso, no es quin contigo pueda.

Texto 3
Garid vos, ay yermaniellas!, cm contenir el mio male? Sin el habib non vivreyo: ad ob li rey demandare?
Lrica espaola de tipo popular Ctedra

(TRADUCCIN: Decidme, ay hermanitas!, / cmo contener mi mal? / Sin el amado no vivir: / adnde ir a buscarlo?)

Marqus DE SANTILLANA Poesa cancioneril castellana, Akal

1. El texto 1 corresponde a una cantiga de la poesa cancioneril del siglo & V. Reconoce en ella las caractersticas formales y temticas de este tipo de composiciones.

6. Los textos 3 y 4 brindan muestras de dos tipos de poesa


primitiva popular. Identifcalas y explica sus rasgos.

2. El texto 2 es la traduccin prosificada de una estrofa de


una moaxaja rabe. Seala los rasgos propios de estos poemas y explica el contenido de la estrofa reproducida.

7. Qu tipo de poesa popular no aparece representada?


Seala semejanzas y diferencias con los anteriores.

8. El texto 5 es una estrofa de una obra del marqus de


Santillana. Explica su contenido y analiza su mtrica. De qu tipo de poesa cancioneril es muestra?

3. Qu otra especie potica culta fue cultivada en alndalus? Explica en qu se diferencia de la moaxaja.

9. Cul es la gran creacin potica de Jorge Manrique?


Explica los siguientes aspectos de esa obra: Contenido de cada parte. Temas. Mtrica. Estilo.

4. Qu poesa se cultiv en el este de la Pennsula en los


siglos &II y & III? Explica su vertiente amorosa.

5. En qu manifestacin culta de la poesa del oeste


peninsular influye la lrica aludida en la actividad 4? Comenta sus rasgos principales.

10. Redacta una exposicin sobre la poesa medieval culta


y popular en gallego-portugus. 193 12.
La lrica medieval

ACTIVIDADES

REFUERZO

13

1. El Cantar de Mio Cid


La pica castellana tiene su mejor exponente en el Cantar de Mio Cid, que exalta la figura del infanzn Rodrigo Daz de Vivar, el Cid Campeador, tanto en su dimensin histrica como en las circunstancias personales de la vida del hroe.

MATERIAL FOTOCOPIABLE / Oxford University Press Espaa, S. A.

Comienzan ya a preparar la gran sala de palacio. Los suelos con mucha alfombra, todo bien encortinado. Cunta seda y cunta prpura y cunto pao preciado! Gusto os dara vivir y comer en el palacio! Los caballeros del Cid aprisa all se juntaron, y entonces en aquel punto por los Infantes mandaron. Ya cabalgan los Infantes, camino van del palacio, con muy ricas vestiduras, galanamente ataviados. A pie y con muy buena cara, Dios qu discretos entraron! Recibiolos nuestro Cid; con l todos sus vasallos. Ante el Cid y su mujer los Infantes se inclinaron. A sentar ellos se fueron en un muy precioso escao. Los de la casa del Cid, siempre en todo mesurados, estn atentos mirando al que naci afortunado. All el Cid Campeador ved que en pie se ha levantado: Puesto que hacerlo tenemos, por qu lo vamos tardando? Venid ac, mi Alvar Fez, el que tanto quiero y amo. Aqu tenis mis dos hijas; yo las pongo en vuestras manos. Sabis que al Rey esto mismo se lo tengo as rogado. No quiero faltar en nada de lo que fue concertado. A los dos Infantes, vos ddselas con vuestras manos, que tomen las bendiciones, y vayamos acabando. Entonces dijo Minaya: Esto har yo de buen grado. De pie las dos se levantan, y l las tom de las manos. Y a los Infantes, Minaya esto mismo les va hablando: Aqu estis ante el Minaya, vosotros, los dos hermanos. De manos del Rey Alfonso, pues a m me lo ha mandado, estas dueas yo os entrego, que ambas son hijasdalgo; que las tomis por mujeres, dadles honras y cuidados. Los de Carrin las reciben con amor y de buen grado. Al Cid y a doa Jimena, van a besarles las manos. As que esto hubieron hecho, se salieron del palacio. Aprisa, a Santa Mara, hacia all van caminando. Muy pronto se revisti don Jernimo, el prelado, y a la puerta de la iglesia estbalos esperando. Las bendiciones les dio; despus la misa ha cantado. Al salirse de la iglesia cabalgaron a buen paso. Afuera de la ciudad, en un arenal cercano, Dios, y qu buen juego de armas hizo el Cid a sus vasallos!

10

15

20

25

30

35

Poema del Cid Castalia

1 Qu episodio del Cantar narra este fragmento? Indica a qu parte del poema corresponde. 2 Seala las apelaciones al auditorio presentes en estos versos. Qu funciones del lenguaje predominan en ellos? 3 Reconoce el empleo de eptetos picos y seala a quin se califica por medio de este recurso. 4 Mide los versos y analiza la rima de esta tirada del Cantar de Mio Cid.

5 Diferencia la presencia del discurso del narrador del de los personajes en este fragmento. 6 Para relatar los sucesos, el narrador emplea dos tiempos verbales; de cules se trata? 7 Qu efecto se produce con el cambio de formas verbales? 8 Explica cmo se caracterizan los personajes mencionados en este pasaje.

13.

La narrativa medieval

207

ACTIVIDADES

REFUERZO

13

2. Un milagro, de Gonzalo de Berceo


A continuacin, tienes uno de los Milagros de Nuestra Seora, de Gonzalo de Berceo; lelo con atencin y observa su estructura, pues esta se repite en los restantes milagros que componen la coleccin.

El labrador avaro
rase en una tierra un hombre labrador que usaba de la reja ms que de otra labor; ms amaba la tierra que no a su Criador, y de muchas maneras era revolvedor. Haca una enemiga bien sucia de verdad: cambiaba los mojones por ganar heredad; haca en todas formas tuertos y falsedad, tena mal testimonio entre su vecindad. Aunque malo, quera bien a Santa Mara, oa sus milagros, muy bien los acoga; saludbala siempre, decale cada da: Ave grata plena que pariste al Mesas. Fin el arrastrapajas de tierra bien cargado, de los diablos fue luego en soga cautivado; lo arrastraban con cuerdas, de coces bien sobado, le pechaban al doble el pan que dio mudado. Dolironse los ngeles de esta alma mezquina por cuanto la llevaban los diablos en rapina; quisieron socorrerla, ganarla por vecina, mas para hacer tal pasta mengubales harina. Si les decan los ngeles de bien una razn, ciento decan los otros malas, que buenas non; los malos a los buenos tenan en un rincn, la alma por sus pecados no sala de prisin. 5 Levantndose, un ngel dijo: Yo soy testigo, verdad, es no mentira, esto que ahora os digo: el cuerpo que traa esta alma consigo fue de Santa Mara buen vasallo y amigo. Siempre la mencionaba al yantar y a la cena, decale tres palabras: Ave, grata plena . Boca por que sala tan santa cantilena no mereca yacer en tan mala cadena. Luego que este nombre de la Santa Rena oyeron los demonios, salieron tan ana, derramronse todos como una neblina, desampararon todos a esa alma mezquina. Los ngeles la vieron quedar desamparada, de manos y de pies con sogas bien atada, estaba como oveja cuando yace enzarzada: fueron y la llevaron junto con su majada. Nombre tan adornado, lleno de virtud tanta, y que a los enemigos los seguda y espanta, no nos debe doler ni lengua ni garganta que no digamos todos: Salve, Regina sancta.
Gonzalo DE BERCEO Milagros de Nuestra Seora Castalia

25

30

10

35

15

40

20

1 Caracteriza al protagonista de este milagro. Qu le sucede? 2 Resume por escrito el contenido de El labrador avaro. 3 Caracteriza al narrador de esta historia de Gonzalo de Berceo. 4 Diferencia en el milagro que acabas de leer las partes narrativas de las intervenciones de los personajes. 5 Cmo se introducen estas ltimas? 6 Seala las partes en que se divide la narracin de Berceo.

7 Reconoce los movimientos narrativos y los tipos de discurso que aparecen en el texto. 8 Cul es la finalidad de este milagro? Relacinala con la del resto de la obra a la que pertenece. 9 Se cumple en este milagro en concreto lo que en tu libro de texto se plantea en general para todas estas narraciones? Justifica tu respuesta. 10 Escribe en prosa la historia narrada por Berceo en El labrador avaro. No olvides cuidar la ortografa y la presentacin de tu redaccin.

208 13.

La narrativa medieval

MATERIAL FOTOCOPIABLE / Oxford University Press Espaa, S. A.

ACTIVIDADES

REFUERZO

13

3. Fragmentos del Libro de buen amor


Los dos fragmentos siguientes corresponden al Libro de buen amor. El primero, de carcter narrativo, pertenece a la serie de aventuras que el protagonista vive en la sierra, donde se enfrenta a mujeres de caractersticas muy especiales; el segundo, lrico, se inserta en el conjunto de poemas intercalados en la historia narrada. Estos dos textos, adems, presentan diferencias en cuanto a la mtrica empleada.

La serrana Aldara
Sus miembros y su talle no son para callar, me podis creer, era gran yegua caballar; quien con ella luchase mal se habra de hallar, si ella no quiere, nunca la podrn derribar. [] Tena la cabeza mucho grande y sin guisa cabellos cortos, negros, como corneja lisa, ojos hundidos, rojos; ve poco y mal divisa; mayor es que de osa su huella, cuando pisa. Las orejas, mayores que las de aal borrico, el su pescuezo, negro, ancho, velludo, chico; las narices, muy gordas, largas, de zarapico, sorbera bien pronto un caudal de hombre rico! Su boca era de alano, grandes labios muy gordos, dientes anchos y largos, caballunos, moxmordos; sus cejas eran anchas y ms negras que tordos. Los que quieran casarse, procuren no estar sordos! Mayores que las mas tiene sus negras barbas; yo no vi ms en ella, pero si ms escarbas, hallars, segn creo, lugar de bromas largas, aunque ms te valdr trillar en las tus parvas. [] Ms anchas que mi mano tiene la su mueca, velluda, pelos grandes y que nunca est seca; voz profunda y gangosa que al hombre da jaqueca, tarda, enronquecida, muy destemplada y hueca. Es su dedo meique mayor que mi pulgar, son los dedos mayores que puedes encontrar, que, si algn da, ella te quisiere espulgar, daarn tu cabeza cual vigas de lagar. Tena en el justillo las sus tetas colgadas, dbanle en la cintura porque estaban dobladas, que, de no estar sujetas, diranle en las ijadas; de la ctara al son bailan, aun no enseadas. 25 5

Gozos de Santa Mara


Madre de Dios, gloriosa Virgen Santa Mara hija y leal esposa de tu hijo Mesa, t, Seora, dame ahora la tu gracia a toda hora; que te sirva en toda va. Porque honrarte codicio, pecador yo!, por tanto ofrezco en tu servicio los gozos que te canto. El primero en que tercero a ti el ngel mensajero fue del Espritu Santo. Concebiste a tu Padre; fue tu gozo segundo, cuando pariste, Madre, sin dolor sali al mundo. 20 Cual naciste (t pariste), intacta permaneciste Oh Virgen del santo mundo! El tercero, la estrella, luciente meteoro; los Reyes, con luz de ella, trajeron su tesoro y alabaron y adoraron y a tu Hijo presentaron el incienso, mirra y oro. 25 5

10

10

15

15

20

MATERIAL FOTOCOPIABLE / Oxford University Press Espaa, S. A.

30

30

Arcipreste DE HITA Libro de buen amor Castalia

Arcipreste DE HITA Libro de buen amor Castalia

1 Atiende a la descripcin de la serrana Aldara y explica cmo aparece caracterizada. 2 Seala las referencias al receptor. Con qu finalidad aparecen? 3 Recuerda los rasgos de la pastorela y justifica por qu esta serrana es un texto pardico. 4 Explica los recursos que emplea el narrador para degradar la figura de la serrana.

5 Analiza la mtrica y la rima de los Gozos. 6 Seala la diferencia mtrica existente entre estos versos de los Gozos y la utilizada, en general, en el Libro de buen amor. 7 Con qu recursos se exalta en la segunda composicin la figura de la Virgen Mara? 8 Justifica por qu los Gozos de Santa Mara constituyen un texto de carcter lrico.

13.

La narrativa medieval

209

ACTIVIDADES

AMPLIACIN

13

4. Novela de caballeras y novela sentimental


Los siguientes fragmentos corresponden a dos de las principales orientaciones de la narrativa del siglo XV: la novela de caballeras y la novela sentimental.

Texto 1
Amads viva en una gran tristeza y soledad. Un da cabalgaba por la orilla del mar, y al subir a unas peas vio que se acercaba una nave. En la nave vena Durn, hermano de la doncella de Dinamarca. Gandaln lo abraz y lo condujo hasta donde estaba Amads. Cuando estuvieron cerca, vieron una especie de gigante envuelto en pieles que esgrima un venablo para arrojarlo contra Amads. Gandaln y Durn empezaron a dar grandes voces, y el venablo pas por encima de la cabeza de Amads. Amads se levant de un salto y pudo salvarse de un segundo venablo. Ech mano a la espada, pero aquella figura extraa sali huyendo, subi en el caballo de Amads y se alej, diciendo a grandes voces: Yo soy Andandona, la giganta de la Isla Triste, tu enemiga, y si ahora no acab lo que deseaba, otra vez ser. Amads dej que Gandaln la persiguiese y abraz a Durn y le pregunt por el objeto de su venida. Durn le dio una carta de Oriana y le dijo que ella haba tenido un hijo que criaba la abadesa del monasterio de Miraflores, y que le rogaba que no partiese de Gaula hasta tener su mando. Volvi Gandaln, con el caballo de Amads y la cabeza de Andandona atada al pretal por los cabellos largos y canos. Amads envi la cabeza a Bruneo de Bonamar y dijo a Durn: Vuelve y dile a tu seora que le beso las manos por la carta que me enva y por lo que t me has dicho de su parte, y que le ruego que tenga piedad de mi honra y no me deje holgar mucho tiempo, porque lo atribuiran a poquedad de corazn y perdera la gloria que por ella he ganado.
Amads de Gaula Castalia

Texto 2
Lucenda: es tan grande merced esta que hacerme quisiste, que si yo de ms valer o ella de menos sustancia no fuese, servicio ninguno satisfacerla podra, si pena en cuenta de servir recibida no fuese; la cual si recibir no quisieres, si merecida te la tengo en m lo puedes bien ver, porque en mi desfiguramiento a ella y a tus obras pintadas vers; y no solamente mis lgrimas a acrecentar mis dolores me han bastado, mas viendo mi parecer, a muchos enamorados de amar he hecho temerosos; y puesto que as es, te suplico que, arrepentida de lo pasado, lo por venir enmiendes. No seas en el daar siempre una; pon con tus obras mis guerras en paz; no s por qu pudiendo has dejado de servida ser. Todas lo quieren y t lo rehyes; bien parece que yo tengo mayor necesidad de tus mercedes, que t voluntad de mis servicios tenas. Oh qu combates de mi mucha fe y de tu poca esperanza he recibido!, los cuales, como ves, la fuerza de mi salud han enflaquecido. Cuanto t menos de mi dolor te dolas, ms mi dolor me dola. Si pudiese en la boca poner lo que en el alma he sentido, cunta culpa por mi pena te daras! Nunca nadie menos bien tuvo; nunca nadie ms mal sufri; nunca de tanta memoria tanto olvido se tuvo; mi afeccin y tu menosprecio destruyen mi salud. Todo esto, Lucenda, te digo, porque ms y en ms mi querer tengas, y porque en mi sufrimiento mi firmeza conozcas; que ni por todo mi mal jams en mudanza pens, ni de sufrir cansado me vi; antes ganado, porque t me perdas, siempre me hall, pero no por pequea causa, que no tena yo menos razn para penar que t hermosura para penarme.
MATERIAL FOTOCOPIABLE / Oxford University Press Espaa, S. A.

Diego DE SAN PEDRO Tratado de amores de Arnalte y Lucenda. Sermn Castalia

1 Describe cmo aparecen caracterizados los personajes del primer texto y justifica por qu son prototpicos del tipo de novela al que pertenecen. 2 En el primer pasaje se plantean dos situaciones; de cules se trata? A qu caractersticas de la novela de caballeras remite esta circunstancia? 3 Teniendo en cuenta lo que sabes acerca de la novela a la que pertenece el primer fragmento, explica la frase con la que concluye (perdera la gloria que por ella he ganado).

4 A qu clase de texto corresponde el fragmento de Diego de San Pedro? Explica de qu modelo narrativo del siglo XV es propio. 5 Justifica por qu su contenido tambin corresponde a ese tipo de novela. 6 Analiza en el fragmento de Diego de San Pedro recursos estilsticos de este tipo de textos. 7 Qu funciones del lenguaje predominan en cada uno de los fragmentos? 8 Compara los dos pasajes y seala las diferencias ms relevantes.

210 13.

La narrativa medieval

Evaluacin 13
Planto de Pleberio
PLEBERIO.Oh Fortuna variable, ministra y mayordoma de los temporales bienes! Por qu no ejecutaste tu cruel ira, tus mudables ondas, en aquello que a ti es sujeto? Por qu no destruiste mi patrimonio? Por qu no quemaste mi morada? Por qu no asolaste mis grandes heredamientos? Dejrasme aquella florida planta, en quien t poder no tenas. Dirasme, fortuna fluctuosa, triste la mocedad con vejez alegre; no pervirtieras el orden. Mejor sufriera persecuciones de tus engaos en la recia y robusta edad, que no en la flaca postrimera. Oh vida de congojas llena, de miserias acompaada! Oh mundo, mundo! Muchos mucho de ti dijeron, muchos en tus cualidades metieron la mano; a diversas cosas por odas te compararon. Yo por triste experiencia lo contar, como a quien las ventas y compras de tu engaosa feria no prsperamente sucedieron, como aquel que mucho ha hasta ahora callado tus falsas propiedades por no encender con odio tu ira, porque no me secases sin tiempo esta flor que este da echaste de tu poder. Pues ahora, sin temor, como quien no tiene qu perder, como aquel a quien tu compaa es ya enojosa, como caminante pobre que sin temor de los crueles salteadores va cantando en alta voz. Yo pensaba en mi ms tierna edad que eras y eran tus hechos regidos por algn orden; ahora, visto el pro y la contra de tus bienandanzas, me pareces un laberinto de errores, un desierto espantable, una morada de fieras, juego de hombres que andan en corro, laguna llena de cieno, regin llena de espinas, monte alto, campo pedregoso, prado

El suicidio de Melibea, grabado de la edicin de Valencia de 1514.

lleno de serpientes, huerto florido y sin fruto, fuente de cuidados, ro de lgrimas, mal de miserias, trabajo sin provecho, dulce ponzoa, vana esperanza, falsa alegra, verdadero dolor. Cbasnos, mundo falso, con el manjar de tus deleites; al mejor sabor nos descubres el anzuelo: no lo podemos huir, que nos tiene ya cazadas las voluntades. Prometes mucho, nada cumples. chasnos de ti, porque no te podamos pedir que mantengas vanos prometimientos. Corremos por los prados de tus viciosos vicios muy descuidados, a rienda suelta; descbresnos la celada cuando ya no hay lugar de volver.
Fernando DE ROJAS La Celestina, Crtica

1. Indica a qu parte de la historia de La Celestina corresponde este pasaje y reconoce en l recursos expresivos. Qu modalidad del discurso dramtico presenta este fragmento? Explica las caractersticas de las otras dos modalidades.

6. Compara el Cantar de Mio Cid con los poemas del mester


de clereca, considerando los aspectos indicados.

CANTAR
! ( . 9 / -% %)' % % " u. % ; )

MESTER DE CLERECA

2. Qu relacin guarda la intervencin de Pleberio con el


tratamiento del amor en la obra? Comenta otros aspectos del amor considerados en la unidad.

3. Expn cuanto sepas acerca del gnero y los personajes


principales de La Celestina.

4. En cierta forma, la obra de Fernando de Rojas parodia la


novela sentimental. Explica por qu y compara este tipo de ficcin narrativa del siglo & V con la de caballeras, atendiendo a los aspectos que se indican en la tabla.

7. Menciona obras del mester de clereca que estn escritas


en cuaderna va.

8. Qu son los enxiemplos? Indica cmo se insertan en


El conde Lucanor.

9. Explica la estructura y el sentido del Libro de buen amor. DE CABALLERAS


2 / ! ).%; %* )! . ! . )

SENTIMENTAL

Qu personaje incorpora esta obra medieval a la tradicin literaria?

10. Redacta una exposicin acerca de los romances que


contemple los aspectos sealados a continuacin. Caractersticas de los romances viejos. Forma de transmisin de dichas composiciones. Temas, estructura narrativa y recursos expresivos caractersticos. 215 13.
La narrativa medieval

! -.* )&

5. Explica los dos asuntos que determinan la estructura del


Cantar de Mio Cid.

ACTIVIDADES

REFUERZO

14

1. Obra potica de Garcilaso de la Vega


En la obra potica de Garcilaso, el amor es el tema ms importante, con el que se relacionan los de la naturaleza y la mitologa. Estos temas estn presentes en sus sonetos y en las glogas.

Soneto XXIX
Pasando el mar Leandro el animoso, en amoroso fuego todo ardiendo, esforz el viento1 y fuese embraveciendo el agua con un mpetu furioso. Vencido del trabajo presuroso, contrastar2 a las ondas no pudiendo, y ms del bien que all perda muriendo que de su propia vida congojoso3, como pudo, sforz su voz cansada y a las ondas habl desta manera, mas nunca fue su voz dellas oda: Ondas, pues no se escusa que yo muera, dejadme all llegar, y a la tornada4 vuestro furor esecut5 en mi vida. 5

gloga II
ALBANIO: En aquel prado all nos reclinamos, y, del Cfiro13 fresco recogiendo el agradable espirtu14, respiramos; las flores, a los ojos ofreciendo diversidad estraa de pintura15, diversamente as estaban oliendo; y en medio aquesta fuente clara y pura, que como de cristal resplandeca, mostrando abiertamente su hondura, el arena, que doro pareca, de blancas pedrezuelas varada, por do manaba el agua, se bulla. 10 5

10

esforz el viento: se desat el vendaval. contrastar: no pudiendo resistir las olas. congojoso: preocupado. tornada: vuelta. esecut: ejecutad. d: dnde agora: ahora. doquier: donde quiera vencimiento: derrota. columna: el cuello. dorado techo: la rubia cabellera. escura: oscura. Cfiro: viento suave y apacible. espirtu: aire. estraa de pintura: diversidad extraordinaria de colores.

gloga I
NEMOROSO: D6 estn agora7 aquellos claros ojos que llevaban tras s, como colgada, mi alma, doquier8 que ellos se volvan? Do est la blanca mano delicada, llena de vencimientos9 y despojos, que de m mis sentidos l`ofrecan? Los cabellos que van con gran desprecio al oro como a menor tesoro adnde estn, adnde el blanco pecho? D la columna10 quel dorado techo11 con proporcin graciosa sostena? Aquesto todo agora ya sencierra, por desventura ma, en la escura12, desierta y dura tierra.

2 3 4 5 6

7 8 9

10

MATERIAL FOTOCOPIABLE / Oxford University Press Espaa, S. A.

10

11 12 13 14

15

15

1 Lee el Soneto XXIX y explica qu le ocurre a Leandro. Qu sentimientos le impulsan a esforzarse? 2 A quines se dirige en los tercetos y qu les pide? 3 Qu descripcin de la amada realiza Nemoroso en la Egloga I ? 4 A qu imagen corresponden los rasgos que has sealado en la pregunta anterior?

5 Qu tpico recrean las interrogaciones que estructuran el fragmento de la gloga I? 6 Qu rasgos tiene el paisaje descrito en la gloga II? A qu tpico responde? 7 Analiza la mtrica y los principales recursos expresivos de las tres composiciones. 8 Relaciona los temas y tpicos de los textos con los de la poesa renacentista.

14.

La lrica renacentista

237

ACTIVIDADES

REFUERZO

14

2. Fray Luis de Len


Fray Luis sufri un proceso inquisitorial que lo llev a la crcel. El agustino se sinti acosado y crey que la envidia de otros, sobre todo dominicos, haba sido determinante; a esto se refiere en el primer poema reproducido. La segunda composicin forma parte de sus poemas religiosos.

A la salida de la crcel
Aqu la envidia y mentira me tuvieron encerrado. Dichoso el humilde estado del sabio que se retira de aqueste mundo malvado, y con pobre mesa y casa en el campo deleitoso con solo Dios se compasa y a solas su vida pasa, ni envidiado ni envidioso.

En la Ascensin
Y dejas, Pastor santo, tu grey1 en este valle hondo, escuro, con soledad y llanto; y t, rompiendo el puro aire, te vas al inmortal seguro2? Lo antes bienhadados y los agora tristes y afligidos, a tus pechos criados, de ti desposedos, a d convertirn3 ya sus sentidos? Qu mirarn los ojos, que vieron de tu rostro la hermosura, que no les sea enojos? Quien oy tu dulzura, qu no tendr por sordo4 y desventura? Aqueste mar turbado quin le pondr ya freno?, quin concierto al viento fiero, airado?; estando t encubierto, qu norte guiar la nave al puerto? Ay nube envidiosa!: aun deste breve gozo, qu te aquejas? D vuelas presurosa? Cun rica t te alejas! Cun pobres y cun ciegos, ay, nos dejas!
1 2 3

20

10

25

grey: conjunto de fieles cristianos. convertirn: volvern, dirigirn. sordo: sonido desprovisto de armona.
MATERIAL FOTOCOPIABLE / Oxford University Press Espaa, S. A.

seguro: lugar desprovisto de todo peligro.

15

1 Adems del tema de la envidia, qu otros temas desarrolla el primer poema? 2 Qu tpico aparece? Relaciona este texto con la Oda a la vida retirada (pgina 227 de tu libro de texto). 3 Qu suceso religioso se canta en el segundo poema? 4 Explica a quin se dirige en la primera estrofa y qu le reprocha.

5 Cules sern los sentimientos de los apstoles, despus de la partida de Jess? 6 A qu pueden referirse las metforas del mar, el viento fiero, la nave y el puerto de la siguiente estrofa? 7 La nube envidiosa tapa la subida de Cristo, cmo se quedan los apstoles? 8 Realiza el anlisis mtrico del primer poema. Cul es la estrofa utilizada en el segundo?

238 14.

La lrica renacentista

ACTIVIDADES

REFUERZO

14

3. San Juan de la Cruz


El Cntico espiritual es una de las obras maestras de la poesa mstica. En l, san Juan utiliza el lenguaje simblico para referirse a la bsqueda que realiza la Esposa (el alma) de su amado (Dios), junto con la valoracin de la naturaleza y numerosos recursos expresivos.

Cntico espiritual
ESPOSA: Adnde te escondiste, Amado, y me dejaste con gemido? Como el ciervo huiste, habindome herido; sal tras ti clamando, y eras ido. Pastores, los que fuerdes all por las majadas al otero2, si por ventura vierdes aquel que yo ms quiero, decildle que adolezco3, peno y muero. Buscando mis amores ir por esos montes y riberas; ni coger las flores, ni temer las fieras, y pasar los fuertes y fronteras. (PREGUNTA A LAS CRIATURAS) Oh, bosques y espesuras, plantadas por la mano del Amado! Oh, prado de verduras, de flores esmaltado!, decid si por vosotros ha pasado RESPUESTA DE LAS CRIATURAS: Mil gracias derramando, pas por estos sotos con presura4, e, yndolos mirando, con sola su figura vestidos los dej de hermosura. ESPOSA: Ay, quin podr sanarme! Acaba de entregarte ya de vero5; no quieras enviarme de hoy ms ya mensajero, que no saben decirme lo que quiero. []
1fuerdes 1

ESPOSA: Gocmonos, Amado, y vmonos a ver en tu hermosura al monte o al collado, do mana el agua pura; entremos ms adentro en la espesura. Y luego a las subidas cavernas de la piedra nos iremos, que estn bien escondidas, y all nos entraremos y el mosto de granadas gustaremos. All me mostraras aquello que mi alma pretenda, y luego me daras all, t, vida ma, aquello que me diste el otro da: el aspirar del aire, el canto de la dulce filomena7, el soto y su donaire en la noche serena, con llama que consume y no da pena. 20
6

35

10

40

15

45

50

San Juan DE LA CRUZ Cntico espiritual, Crtica

/ vierdes: formas arcaicas de fueres y vieres.

25
3 4 5 6

otero: cerro aislado que domina un llano. adolecer: enfermar. presura: prisa, prontitud, ligereza. de vero: de verdad. aspirar: el soplo. filomena: el ruiseor.

MATERIAL FOTOCOPIABLE / Oxford University Press Espaa, S. A.

30

1 De qu se queja la amada? Cmo se siente? 2 Qu les pide a los pastores? Qu decide hacer, segn la tercera estrofa y a qu est dispuesta? 3 La amada se dirige a la naturaleza, cmo se presenta sta? Qu le responde? 4 Qu le reprocha al amado en la sexta estrofa? Qu rechaza?

5 En los versos 31-35 ya se ha producido la unin, explica cul es el ruego de la esposa, a dnde le sugiere que vayan? 6 Qu desea la esposa en la ltima estrofa? 7 Analiza la mtrica del poema. 8 Localiza y explica los principales recursos expresivos empleados por san Juan en estos versos.

14.

La lrica renacentista

239

ACTIVIDADES

AMPLIACIN

14

4. Lrica renacentista
En esta pgina se incluyen poemas de otros autores renacentistas, que siguieron el modelo de Garcilaso de la Vega: Gutierre de Cetina (1510-1554), Fernando de Herrera (1534-1597), Francisco de Aldana (1537-1578). Del bellsimo ltimo texto, annimo, no se sabe exactamente su fecha.

Texto 1
Entre armas, guerra, fuego, ira y furores, que al soberbio francs tienen opreso1, cuando el aire es ms turbio y ms espeso, all me aprieta el fiero ardor de amores. Miro el cielo, los rboles, las flores, y en ellos hallo mi dolor expreso; que en el tiempo ms fro y ms avieso2 nacen y reverdecen mis temores. Digo llorando: Oh dulce primavera!, cundo ser que a mi esperanza vea, verde, prestar al alma algn sosiego? Mas temo que mi fin mi suerte fiera tan lejos de mi bien quiere que sea entre guerra y furor, ira, armas, fuego.
Gutierre DE CETINA
1

Texto 3
De sus hermosos ojos dulcemente un tierno llanto Filis despeda que por el rostro amado pareca claro y precioso aljfar5 transparente; 5 en brazos de Damn, con baja frente, triste, rendida, muerta, helada y fra, estas palabras breves le deca, creciendo a su llorar nueva corriente: Oh pecho duro, oh alma dura y llena de mil durezas! Dnde vas huyendo? D vas con ala tan ligera y presta? Y l, soltando de llanto amarga vena, della las dulces lgrimas bebiendo, besola, y solo un ay! fue su respuesta.
Francisco DE ALDANA
5

10

10

opreso: oprimido.

avieso: malo, torcido.

aljfar: perla pequea de forma irregular.

Texto 2
Serena Luz, en quien presente espira divino amor, que enciende y junto enfrena4 el noble pecho, que en mortal cadena al alto Olimpo levantarse aspira; ricos cercos dorados, do se mira tesoro celestial de eterna vena; , armona de anglica sirena, que entre las perlas y el coral respira, cul nueva maravilla, cul ejemplo de la inmortal grandeza nos descubre aquesta sombra del hermoso velo? Que yo en esa belleza que contemplo (aunque a mi flaca vista ofende y cubre), la inmensa busco, y voy siguiendo al cielo.
Fernando DE HERRERA
3

Texto 4 Soneto a Cristo crucificado


No me mueve, mi Dios, para quererte el cielo que me tienes prometido; ni me mueve el infierno tan temido para dejar por eso de ofenderte. T me mueves, seor; muveme el verte clavado en una cruz y escarnecido; muveme ver tu cuerpo tan herido; muvenme tus afrentas y tu muerte. 10 Muveme, en fin, tu amor, y en tal manera que aunque no hubiera cielo, yo te amara, y aunque no hubiera infierno, te temiera. No tienes que me dar porque te quiera, pues aunque cuanto espero no esperara, lo mismo que te quiero te quisiera.
ANNIMO Poesa lrica del Siglo de Oro, Ctedra

10
MATERIAL FOTOCOPIABLE / Oxford University Press Espaa, S. A.

espira: sopla.

enfrena: frena.

1 En qu situacin se halla el yo potico en el texto 1? Cules son sus sentimientos? 2 Cules son sus deseos y sus temores, segn los tercetos? Qu temas desarrolla este texto? 3 Qu descripcin se hace de la amada en el texto 2? 4 Qu visin de ella nos ofrece la voz potica? Relaciona esta imagen con las ideas del neoplatonismo.

5 En el texto 3, hay una escena de amor correspondido; sin embargo muestra una situacin dolorosa, cul es? 6 Cul es la reaccin y la respuesta del amante? 7 Explica las razones del amor a Cristo del yo potico en el ltimo texto. 8 Relaciona los temas de estos poemas con los de la lrica renacentista.

240 14.

La lrica renacentista

Evaluacin 14

Concierto campestre, por Tiziano.

El lamento de Albanio
En medio del invierno est templada el agua dulce desta clara fuente, y en el verano ms que nieve helada. Oh claras ondas, cmo veo presente, en vindoos, la memoria da quel da de que el alma temblar y arder se siente! En vuestra claridad vi mi alegra oscurecerse toda y enturbiarse; cuando os cobr, perd mi compaa. A quin pudiera igual tormento darse, que con lo que descansa otro afligido venga mi corazn a atormentarse? El dulce murmurar deste rido, el mover de los rboles al viento, el suave olor del prado florecido podrian tornar d enfermo y descontento cualquier pastor del mundo alegre y sano: yo solo en tanto bien morir me siento. Oh hermosura sobre l ser humano, oh claros ojos, oh cabellos d oro, oh cuello de marfil, oh blanca mano! Cmo puede ora ser que n triste lloro se convirtiese tan alegre vida y en tal pobreza todo mi tesoro? Quiero mudar lugar y a la partida quiz me dejar parte del dao que tiene el alma casi consumida.

1. Explica el contenido de este fragmento de la gloga II,


de Garcilaso de la Vega.

2. Menciona el tema del poema y reconoce los tpicos que


aparecen. 5

3. Justifica el carcter literario y el gnero de este texto. 4. Seala los principales recursos expresivos y realiza el
anlisis mtrico de la composicin.

5. Atendiendo a tus respuestas, relaciona el tema y los


10 tpicos, los recursos expresivos y la mtrica de estos versos con los principales rasgos de la lrica renacentista.

6. El fragmento citado pertenece a la gloga II. Explica qu es


una gloga y comenta el contenido de las tres que escribi Garcilaso de la Vega. Menciona tambin los temas principales de su produccin potica.

15

7. Qu innovaciones de la lrica renacentista proceden de


Italia? Por qu se dio esta influencia?

8. Qu tipos de poesa religiosa se cultivaron en la segunda


20 mitad del siglo & VI? Cita los ttulos de los poemas mayores de san Juan de la Cruz, explica el asunto principal abordado en su poesa y el recurso bsico que utiliza.

9. Cules son los temas y tpicos ms importantes de la


25 Oda a la vida retirada, de fray Luis de Len? Explica qu mtrica utiliza en la mayora de sus poemas.

10. Redacta un texto con las etapas y las tendencias poticas


de la poesa del siglo & 233 14.
La lrica renacentista
VI.

Cita autores y obras.

ACTIVIDADES

AMPLIACIN

15

1. Novela bizantina y novela pastoril


Los amores de Clareo y Florisea y los trabajos de la sin ventura Isea, de Alonso Nez de Reinoso, y La Diana, de Jorge de Montemayor, son obras paradigmticas de la novela bizantina y de la novela pastoril, respectivamente, en la pennsula.

En la fuente de los alisos


Despus que Sireno puso fin a su canto, vio como hacia l vena la hermosa Selvagia, y el pastor Sylvano, de que no recibi pequeo contentamiento, y despus de haberse recibido, determinaron ir a la fuente de los alisos, donde el da antes haban estado. Y primero que all llegasen, dijo Sylvano: Escucha, Selvagia, no oyes cantar? S oigo dijo Selvagia, y aun parece ms de una voz. A dnde ser? dijo Sireno. Parceme respondi Selvagia que es en el prado de los laureles, por donde pasa el arroyo que corre de esta clara fuente. Bien ser que nos lleguemos all, y de manera que no nos sientan los que cantan, porque no interrumpamos la msica. Vamos dijo Selvagia. Y as su paso a paso se fueron hacia aquella parte donde las voces se oan, y escondindose entre unos rboles que estaban junto al arroyo, vieron sobre las doradas flores asentadas tres ninfas, tan hermosas que pareca haber en ellas dado la naturaleza muy clara muestra de lo que puede. Venan vestidas de unas ropas blancas, labradas por encima de follajes de oro, sus cabellos que los rayos del sol oscurecan, revueltos a la cabeza, y tomados con sendos hilos de orientales perlas, con que encima de la cristalina frente se haca una lazada, y en medio della estaba un guila de oro, que entre las uas tena un muy hermoso diamante.
Jorge DE MONTEMAYOR La Diana, Ctedra

La nsula de la crueldad
Pasadas todas estas cosas, yo determin de partirme en Efeso; y mandando a fletar una nao y despidindome de Ibrina y de su marido, nos embarcamos y comenzamos de navegar la va de la ciudad de Efeso. Y habiendo algunos das que con buen tiempo hacamos nuestro camino, vimos de lejos una isla, la cual pareca tan oscura que la noche no lo es tanto. Pareca que unos humos negros de azufre salan della; las casas y arboledas eran todas negras y de negra color, las aguas que por ella corran eran todas de color de sangre; oanse grandes y dolorosos gritos y grandes alaridos que ponan espanto a los que los oan. Y ans, nosotros quedamos espantados y con deseo de saber qu aventura fuese aquella. Y porque ya era de noche, mandamos a los marineros que detuviesen la nao hasta la maana porque queramos saber qu tierra fuese aquella. Ellos lo hiceron ans y dijeron que era mejor tomar tierra porque aquella, aunque tan triste pareciese, era segura, a la cual se llamaba la nsula de la Crueldad, porque en ella estaban sepultados todos aquellos que cruelmente haban sido muertos, y que ans era el uso de aquellas tierras que estaban cercanas y de ms lejos, que era traer all a sepultallos.
MATERIAL FOTOCOPIABLE / Oxford University Press Espaa, S. A.

Yo quisiera que pasramos adelante por no quedar all en tan triste tierra, pero Clareo no lo consinti, antes mand tomar tierra. Y ans lo hicimos; y saliendo al puerto nos quedamos all la noche, la cual fue tan larga que pensamos que nunca amaneciese, y era la causa porque el sol no pareca en aquella isla, antes hua della. Pero habiendo entrado el da, aunque oscuro, bien conocimos que era ya de da, y comenzando a mirar la tierra vimos muchas sepulturas de muchas personas que cruelmente haban sido muertas.
Alonso NEZ DE REINOSO Los amores de Clareo y Florisea y los trabajos de la sin ventura Isea, SPUE

1 Resume el contenido de ambos fragmentos. 2 Explica por qu constituyen textos narrativos. 3 Reconoce en los dos pasajes: Narrador. Movimientos narrativos. Tipos de discurso.

4 Seala los recursos expresivos con los que se materializa en cada uno de los fragmentos la funcin potica. 5 Indica los rasgos por los que los textos reproducidos en esta pgina se adscriben a la novela pastoril, el primero, y a la bizantina, el segundo.

15.

La narrativa renacentista

251

ACTIVIDADES

AMPLIACIN

15

2. Novela celestinesca y novela morisca


A continuacin presentamos un par de textos representativos de dos de los modelos narrativos cultivados en el siglo XVI: la novela celestinesca y la novela morisca. Se trata, respectivamente, de Segunda Celestina, de Feliciano de Silva, y de El Abencerraje, que apareci incorporado en una edicin de La Diana.

El regreso de Celestina
muero de miedo! ARESA.Ay Jess, que me , que mi madre Celestina ELICIA.Ay, hermana ma gen Mara, y no sea algn parece! Ay, vlame la Vir matar! fantasma que nos quiera ! hayis miedo, que yo soy CELESTINA.Ay bobas, y no ar, raz ab e idm ven s, ore am Las mis hijas y los mis que ac tornar me dej! y dad las gracias a Dios ar, que no tendr sufriAs juntas os quiero abraz r. miento para ms espera lame Dios! Ay madre, des V ELICIA.Ay Jess, Jess! que nso pie de miedo, que vate all que me muero a! eres muert tos, bita!; y de qu das gri CELESTINA.Ay boba, bo mis las y hija amor y mi loca? No hayas miedo, mi ermu no y soy a viv , que entraas, que yo soy tu ta as, ad Os s? liza da an esc te ta; abrzame, loca, qu ja vie y jer zo como soy mu que si fuera hombre y mo r. aza abr me me ver ni de que no te espantaras de n seas venida! Y para bie y ELICIA.Ay ta, seora, l hora y en mal punto? qu dices malicias en ma ivenir a quien yo ms qu Y qu hombre pudiera siera que a ti? que nio, que a osadas, hija, CELESTINA.Aquel Sempro vi, lo yo y da, que llo segn no estuviera bien abraza en los infiernos abrasado. o DE SILVA

La victoria de Narvez
Rodrigo de Narvez, barruntando la necesidad en que sus compaeros estaban, atraves el camino, y como traa mejor caballo se adelant; y viendo la valenta del moro, qued espantado, porque de los cinco escuderos tena los cuatro en el suelo, y el otro, casi al mismo punto. l le dijo: Moro, vente a m, y si t me vences, yo te aseguro de lo dems. Y comenzaron a trabar brava escaramuza, mas como el alcaide vena de refresco, y el moro y su caballo estaban heridos, dbale tanta priesa que no poda mantenerse; mas viendo que en sola esta batalla le iba la vida y contentamiento, dio una lanzada a Rodrigo de Narvez que, a no tomar el golpe en su darga, le hubiera muerto. l, en recibiendo el golpe, arremeti a l y diole una herida en el brazo derecho, y cerrando luego con l, le trab a brazos y, sacndole de la silla, dio con l en el suelo. Y yendo sobre l le dijo: Caballero, date por vencido; si no, matarte he. Matarme bien podrs dijo el moro que en tu poder me tienes, mas no podr vencerme sino quien una vez me venci. El alcaide no par en el misterio con que se decan estas palabras, y usando en aquel punto de su acostumbrada virtud, le ayud a levantar, porque de la herida que le dio el escudero en el muslo y de la del brazo, aunque no eran grandes, y del gran cansancio y cada, qued quebrantado; y tomando de los escuderos aparejo, le lig las heridas.
El Abencerraje (novela y romancero) Ctedra

Felician Segunda Celestina, Ctedra

1 Resume el contenido de ambos fragmentos. 2 Explica la peculiaridad del discurso del primer texto. 3 Reconoce en el segundo texto los siguientes aspectos: Narrador. Movimiento narrativo. Tipos de discurso.

4 Seala los principales recursos expresivos con los que se materializa en los fragmentos de Segunda Celestina y del Abencerraje la funcin potica. 5 Indica los rasgos por los que estos textos pertenecen a la novela celestinesca, el primero, y a la morisca, el segundo. No olvides argumentar tu respuesta.

252 15.

La narrativa renacentista

MATERIAL FOTOCOPIABLE / Oxford University Press Espaa, S. A.

ACTIVIDADES

REFUERZO

15

3. Lazarillo de Tormes
El siguiente fragmento corresponde al Tratado III, que pertenece al primer mdulo de la obra, donde se cuenta la infancia de Lzaro de Tormes.

Al servicio del escudero


La maana venida, levantmonos, y comienza a limpiar y sacudir sus calzas1 y jubn2 y sayo3 y capa; y yo que le serva de pelillo4. Y vsteseme muy a su placer, despacio. Echele aguamanos5, peinose y psose su espada en el talabarte6 y, al tiempo que la pona, djome: Oh, si supieses, mozo, qu pieza es esta! No hay marco de oro en el mundo por que yo la diese. Mas ans ninguna de cuantas Antonio hizo no acert a ponelle7 los aceros tan prestos como esta los tiene. Y sacola de la vaina y tentola con los dedos, diciendo: Vesla aqu? Yo me obligo con ella cercenar un copo de lana. Y yo dije entre m: Y yo con mis dientes, aunque no son de acero, un pan de cuatro libras. Tornola a meter y cisela, y un sartal8 de cuentas gruesas del talabarte. Y con un paso sosegado y el cuerpo derecho, haciendo con l y con la cabeza muy gentiles meneos, echando el cabo de la capa sobre el hombro y a veces so el brazo, y poniendo la mano derecha en el costado, sali por la puerta, diciendo: Lzaro, mira por la casa en tanto que yo voy a or misa, y haz la cama y ve por la vasija de agua al ro, que aqu bajo est, y cierra la puerta con llave, no nos hurten algo, y ponla aqu al quicio, porque si yo viniere en tanto pueda entrar. Y sbese por la calle arriba con tal gentil semblante y continente, que quien no lo conociera pensara ser muy cercano pariente al conde de Arcos, o al menos camarero que le daba de vestir. Bendito seis Vos, Seor qued yo diciendo, que dais la enfermedad y ponis el remedio! Quin encontrar a aquel mi seor que no piense, segn l contento de s lleva, haber anoche bien cenado y bien dormido en buena cama, y, aun agora es de maana, no le cuenten por muy bien almorzado? Grandes secretos son, Seor, los que Vos hacis y las gentes ignoran! A quin no empear aquella buena disposicin y razonable capa y sayo? Y quin pensar que aquel gentil hombre se pas todo el da sin comer, con aquel mendrugo de pan que su criado Lzaro trujo9 un da y una noche en el arca de su seno, do no se le poda pegar mucha limpieza, y hoy, lavndose las manos y cara, a falta de pao de manos se haca servir de la halda10 del sayo? Nadie, por cierto, lo sospechar. Oh, Seor, y cuntos de aquestos debis Vos tener por el mundo derramados, que padecen por la negra que llaman honra lo que por Vos no sufriran!
Lazarillo de Tormes Ctedra

MATERIAL FOTOCOPIABLE / Oxford University Press Espaa, S. A.

calza: prenda que cubra, cindolos, el muslo y la pierna. 2jubn: vestidura que cubra desde los hombros hasta la cintura, ceida y ajustada al cuerpo. 3sayo: prenda de vestir holgada y sin botones que cubra el cuerpo hasta las rodillas. 4servir de pelillo: hacer servicios de poca importancia.
1

aguamanos: aguamanil; agua que sirve para lavar las manos. 6talabarte: cinturn de cuero del que cuelga la espada. 7ponelle: ponerle. 8sartal: sarta de cosas metidas en un hilo o cuerda. 9trujo: trajo (de traer). 10halda: falda.
5

1 Resume el contenido de este fragmento, correspondiente al Tratado III del Lazarillo y caracteriza el comportamiento del escudero. 2 En el primer prrafo se observa un cambio en el uso de los tiempos verbales. Qu efecto produce? Este recurso se reitera ms adelante; localzalo.

3 Qu movimiento narrativo se aprecia en la primera parte del fragmento? Por qu tipo de discurso es interrumpido? 4 Indica a qu tipo de discurso pertenece la exclamacin final. 5 Seala y explica el empleo del recurso de la irona en las palabras del escudero.

15.

La narrativa renacentista

253

ACTIVIDADES

REFUERZO

15

4. Don Quijote de la Mancha


El siguiente texto pertenece a la segunda parte del Quijote. Los protagonistas han llegado al palacio de los duques, quienes, habiendo ledo la primera parte del libro, deciden invitarlos para divertirse a su costa. La duquesa le pide al caballero que describa a Dulcinea, y este comienza su respuesta aludiendo al encantamiento de que ha sido objeto (recuerda el engao de Sancho).

El encantamiento de Dulcinea
S hiciera, por cierto respondi don Quijote, si no me la hubiera borrado de la idea la desgracia que poco ha que le sucedi, que es tal, que ms estoy para llorarla que para describirla, porque habrn de saber vuestras grandezas que yendo los das pasados a besarle las manos, y a recebir su bendicin, beneplcito y licencia para esta tercera salida, hall otra de la que buscaba: hallela encantada y convertida de princesa en labradora, de hermosa en fea, de ngel en diablo, de olorosa en pestfera, de bien hablada en rstica, de reposada en brincadora, de luz en tinieblas, y, finalmente, de Dulcinea del Toboso en una villana de Sayago. Vlame Dios! dando una gran voz, dijo a este instante el duque. Quin ha sido el que tanto mal ha hecho al mundo? Quin ha quitado dl la belleza que le alegraba, el donaire que le entretena y la honestidad que le acreditaba? Quin? respondi don Quijote. Quin puede ser sino algn maligno encantador de los muchos invidiosos que me persiguen? Esta raza maldita, nacida en el mundo para escurecer y aniquilar las hazaas de los buenos, y para dar luz y levantar los fechos de los malos. Perseguido me han encantadores, encantadores me persiguen, y encantadores me perseguirn hasta dar conmigo y con mil altas caballeras en el profundo abismo del olvido, y en aquella parte que me daan y hieren donde ven que ms lo siento, porque quitarle a un caballero andante su dama es quitarle los ojos con que mira, y el sol con que se alumbra, y el sustento con que se mantiene. Otras muchas veces lo he dicho, y ahora lo vuelvo a decir; que el caballero andante sin dama es como el rbol sin hojas, el edificio sin cimiento, y la sombra sin cuerpo de quien se cause. No hay ms que decir dijo la duquesa; pero si, con todo eso, hemos de dar crdito a la historia que del seor don Quijote de pocos das a esta parte ha salido a la luz del mundo, con aplauso de las gentes, della se colige, si mal no me acuerdo, que nunca vuesa merced ha visto a la seora Dulcinea.

Y que esta tal seora no es en el mundo, sino que es dama fantstica, que vuesa merced la engendr y pari en su entendimiento, y la pint con todas aquellas gracias y perfecciones que quiso. En eso hay mucho que decir respondi don Quijote. Dios sabe si hay Dulcinea o no en el mundo, o si es fantstica, o no es fantstica; y estas no son de las cosas cuya averiguacin se ha de llevar hasta el cabo. Ni yo engendr ni par a mi seora, puesto que la contemplo como conviene que sea una dama que contenga en s las partes que puedan hacerla famosa en todas las del mundo, como son: hermosa sin tacha, grave sin soberbia, amorosa con honestidad, agradecida por corts, corts por bien criada y, finalmente, alta por linaje, a causa que sobre la buena sangre resplandece y campea la hermosura con ms grados de perfeccin que en las hermosas humildemente nacidas.
Miguel DE CERVANTES Don Quijote de la Mancha Planeta

1 Comenta los temas que se desarrollan en el dilogo entre don Quijote y los duques y relacinalos con las caractersticas del personaje protagonista y con lo que conoces del argumento de la novela. 2 Repasa las principales caractersticas de las novelas de caballeras y del concepto del amor corts y relacinalas con lo que expresa el caballero.

3 Qu responde el protagonista respecto al comentario de que Dulcinea es una dama fantstica? 4 Explica las caractersticas de la locura de don Quijote. 5 Analiza los recursos expresivos utilizados en la primera y la ltima intervencin de don Quijote en el fragmento que acabas de leer.

254 15.

La narrativa renacentista

MATERIAL FOTOCOPIABLE / Oxford University Press Espaa, S. A.

ACTIVIDADES

AMPLIACIN

15

5. Una imagen de mujer en el Quijote


A continuacin se presentan dos textos del Quijote. El primero corresponde al discurso que pronuncia el caballero sobre la Edad de Oro. El segundo fragmento pertenece a la historia pastoril intercalada de Marcela y Grisstomo. Ambos jvenes hermosos y ricos campesinosdeciden hacerse pastores: Marcela porque prefiere este tipo de vida; Grisstomo porque pretende conquistarla. Marcela lo rechaza, pues no desea casarse, y Grisstomo se quita la vida. En este texto, la pastora hace su alegato.

Texto 1
Dichosa edad y siglos dichosos aquellos a quien los antiguos pusieron nombre de dorados, y no porque en ellos el oro, que en esta nuestra edad de hierro tanto se estima, se alcanzase en aquella venturosa sin fatiga alguna, sino porque entonces los que en ella vivan ignoraban estas dos palabras de tuyo y mo []. Las doncellas y la honestidad andaban, como tengo dicho, por dondequiera, sola y seora, sin temor que la ajena desenvoltura y lascivo intento la menoscabasen, y su perdicin naca de su gusto y propia voluntad.
Miguel DE CERVANTES Don Quijote de la Mancha Planeta

Texto 2
Yo nac libre, y para poder vivir libre escog la soledad de los campos. Los rboles de estas montaas son mi compaa, las claras aguas de estos arroyos mis espejos; con los rboles y con las aguas comunico mis pensamientos y hermosura. Fuego soy apartado y espada puesta lejos. A los que he enamorado con la vista he desengaado con las palabras. Y si los deseos se sustentan con esperanzas, no habiendo yo dado alguna a Grisstomo ni a otro alguno, en fin de ninguno de ellos, bien se puede decir que antes le mat su porfa que mi crueldad. Y si se me hace cargo que eran honestos sus pensamientos, y que por esto estaba obligada a corresponder a ellos, digo que cuando en este mismo lugar donde ahora se cava su sepultura me descubri la bondad de su intencin, le dije yo que la ma era vivir en perpetua soledad, y que sola la tierra gozase el fruto de mi recogimiento y los despojos de mi hermosura; y si l, con todo este desengao, quiso porfiar contra la esperanza y navegar contra el viento, qu mucho que se anegase en la mitad del golfo de su desatino? [] Si yo conservo mi limpieza con la compaa de los rboles, por qu ha de querer que la pierda el que quiere que la tenga con los hombres? Yo, como sabis, tengo riquezas propias y no codicio las ajenas; tengo libre condicin y no gusto de sujetarme; ni quiero ni aborrezco a nadie. No engao a este, ni solicito a aquel; ni burlo con uno, ni me entretengo con el otro. La conversacin honesta de las zagalas destas aldeas y el cuidado de mis cabras me entretiene. Tienen mis deseos por trmino estas montaas, y si de aqu salen, es a contemplar la hermosura del cielo, pasos con que camina el alma a su morada primera.
Miguel DE CERVANTES Don Quijote de la Mancha Planeta

MATERIAL FOTOCOPIABLE / Oxford University Press Espaa, S. A.

1 Redactad en grupos un informe en el que se relacionen los siguientes aspectos. Los pensamientos de don Quijote sobre la Edad de Oro. El alegato de Marcela. Las ideas de Gelasia plasmadas en el soneto de la pgina 244 del Libro del alumno.

2 Escribe un texto en el que reflexiones acerca de la vinculacin de la imagen de la mujer independiente planteada en estos textos con las concepciones de la mujer vigentes en la sociedad actual. 3 Analiza los principales recursos expresivos utilizados en el texto 2.

15.

La narrativa renacentista

255

Evaluacin 15
Encuentro con el escudero
T, mozo, has comido? No, seor dije yo , que an no eran dadas las ocho cuando con Vuestra Merced encontr. Pues, aunque de maana, yo haba almorzado, y cuando ans como algo, hgote saber que hasta la noche me estoy ans. Por eso, psate como pudieres, que despus cenaremos. Vuestra Merced crea, cuando esto o, que estuve en poco de caer de mi estado, no tanto de hambre como por conocer de todo en todo la fortuna serme adversa. All se me presentaron de nuevo mis fatigas y torn a llorar de mis trabajos. All se me vino a la memoria la consideracin que haca cuando me pensaba ir del clrigo, diciendo que, aunque aquel era desventurado y msero, por ventura topara con otro peor. Finalmente, all llor mi trabajosa vida pasada y mi muerte venidera. Y con todo ello, disimulando lo mejor que pude: Seor, mozo soy, que no me fatigo mucho por comer, bendito Dios. De eso me podr yo alabar entre todos mis iguales, por de mejor garganta, y ans fui yo loado de ella hasta hoy da de los amos que yo he tenido. Virtud es esa dijo l , y por eso te querr yo ms. Porque el hartar es de los puercos y el comer regaladamente es de los hombres de bien. [ ] Pseme a un cabo del portal y saqu unos pedazos de pan del seno, que me haban quedado de los de por Dios. l que vio esto, djome: Ven ac, mozo. Qu comes? Yo llegueme a l y mostrele el pan. Tomome l un pedazo de tres que eran, el mejor y el ms grande. Y djome: Por mi vida, que parece este buen pan. Y cmo, agora dije yo, seor, es bueno?

Retrato doble de dos jvenes, por Giorgione.

S, a fe dijo l . Adnde lo hubiste? Si es amasado de manos limpias? N o s yo eso l e dije, mas a m no me pone asco el sabor de ello. As plega a Dios dijo el pobre de mi amo. Y llevndolo a la boca, comenz a dar en l tan fieros bocados como yo en lo otro.
Lazarillo de Tormes Ctedra

1. Resume el contenido de este fragmento del Lazarillo


de Tormes.

6. Qu estilos propios de la narrativa anterior estn


presentes en el Quijote? Argumenta por qu razn se la considera la primera novela moderna.

2. Caracteriza los elementos del nivel de la historia presentes en el fragmento. Personajes. Tiempo. Acciones. Espacio.

7. Explica la estructura narrativa de esta clebre obra


cervantina, caracteriza a sus protagonistas y enuncia sus temas principales.

3. Explica qu descubre Lzaro en este episodio. De qu


mdulo o mdulos de la estructura de la obra es una constante ese descubrimiento?

8. Menciona las caractersticas de las Novelas ejemplares de


Miguel de Cervantes atendiendo a los aspectos que se indican a continuacin. Caractersticas temticas Caractersticas estructurales

4. Identifica el movimiento narrativo presente en el fragmento, extrae ejemplos de discurso valorativo y universal, y seala el recurso de la irona.

9. Qu otras obras narrativas escribi Cervantes? Indica a


qu tipo de novela renacentista pertenecen.

5. Caracteriza al protagonista del Lazarillo de Tormes y


compralo con los protagonistas de las novelas de caballeras, sentimental, morisca y bizantina.

10. Redacta una exposicin sobre el tratamiento del amor


en la narrativa renacentista. 255 15.
La narrativa renacentista

ACTIVIDADES

AMPLIACIN

16

1. La apariencia barroca
La ideologa del barroco, condicionada por el contexto histrico, social y religioso, presenta una serie de rasgos que la diferencian de la de otros perodos y que se manifiestan en todas las producciones culturales. En el siguiente texto se trata uno de sus componentes esenciales.

La apariencia
No hace falta recordar que la imaginacin fue el gran componente del desarrollo cultural de la pennsula ibrica en el siglo XVII. Una sociedad de cuyo acervo cultural forman parte los milagros tiene que ser, a la fuerza, una sociedad imaginativa; incluso la cantidad de plagios, copias, versiones, variaciones, amontonamiento de citas y rplicas no hacen ms que atestiguar una actividad incesante, una reflexin minimalista, detallista, barroca, que descubre y respira en el cmulo de las infinitas posibilidades de lo que puede ser y lo que es. La gran pasin del siglo es la apariencia: la vida es sueo, desconfiemos de los sentidos, he aqu la reflexin inicial del pensamiento barroco. Esta desconfianza de los sentidos no tiene tanto que ver con la desconfianza en la percepcin, sino, al contrario, con una fe en el poder de seduccin y de comunicacin de la imagen, que imposibilita discernir entre la apariencia y la realidad. Mis ojos no me engaan respecto de lo que veo, sino respecto de la naturaleza de lo que veo. Por lo tanto el engao lo es del valor de las cosas: el oro resulta finalmente ser oropel. Precisamente por ser esta una poca transida de smbolos, el engao y el equvoco fluctan siempre en torno a un error, provocado o fortuito, en la interpretacin. El drama Del Rey abajo ninguno se construye en torno a la confusin creada por una banda que debera llevar al rey y lleva a otra persona; los consejos del hidalgo de El Buscn giran en torno a cmo engaar en la apariencia, es decir, a cmo sostener la diferencia entre lo que el espectador ve y lo que la capa sabe []. Encontramos este mismo inters en la poltica, mientras que en el mbito terico y cientfico esta inquietud por los reflejos de una realidad evasiva, se manifestar de dos formas distintas. La primera, acumulando maravillas, todo aquello que al ojo pueda, por distintos motivos, sorprender, agradar o enfadar; dicho de otro modo, haciendo recuento y acopio de todos los casos, de todos los objetos, de todas las formas que presentan una combinacin original, una excrecencia, una composicin excelsa. La segunda, intentando establecer un orden en esa multitud de apariencias, intentando crear modos de medir, de fijar y comprender las relaciones pticas con la realidad: desde la naturaleza de la luz y el funcionamiento del ojo, a la medicin de las distancias y la estimacin de las alturas. Refraccin, reflexin, sombra, proyeccin, movimientos aparentes, puntos fijos, perspectiva, gnomnica son solo una parte de los intereses de este siglo.
MATERIAL FOTOCOPIABLE / Oxford University Press Espaa, S. A.

Nuria VALVERDE El prodigio en su medida, en Madrid, ciencia y Corte, Consejera de Educacin y Cultura

1 Cul es la gran pasin del siglo XVII? En qu creencia se basa? 2 Relaciona la afirmacin anterior con el poema de Argensola de la pgina 260 del Libro del alumno. 3 Cmo se manifiesta este inters por la apariencia en el mbito tcnico y cientfico? Cita algunos ejemplos.

4 La confianza en el poder de persuasin y fascinacin de las imgenes se manifiesta en la proliferacin de otros tipos de actividades. De cules se trata? 5 Tomando en cuenta las respuestas anteriores, realiza un resumen del texto. 6 Cita algunos poemas ledos en esta unidad en los que se trate el tema del engao.

16.

La poesa y la prosa barrocas

275

ACTIVIDADES

REFUERZO

16

2. Sonetos de Luis de Gngora


Gngora escribi sonetos de diversos temas: amorosos, de influencia petrarquista; morales, satricos y burlescos. Tambin escribi los llamados heroicos, destinados a la alabanza de amigos, grandes o reyes.

A Crdoba
Oh excelso muro, oh torres coronadas de honor, de majestad, de gallarda! Oh gran ro1, gran rey de Andaluca, de arenas nobles, ya que no doradas2! Oh frtil llano, oh sierras levantadas, que privilegia el cielo y dora el da! Oh siempre glorosa patria ma, tanto por plumas cuanto por espadas3! Si entre aquellas rinas y despojos que enriquece Genil y Dauro4 baa tu memoria no fue alimento mo, 5

Grandes ms que elefantes y que abadas


Grandes6, ms que elefantes y que abadas7, ttulos8 liberales9 como rocas, gentiles hombres, solo de sus bocas, ilustri cavaglier10, llaves doradas11; hbitos12, capas digo remendadas, damas de haz y envs, viudas sin tocas13, carrozas14 de ocho bestias, y aun son pocas con las que tiran y que son tiradas; catarriberas15, nimas en pena, con Brtulos y Abades16 la milicia, y los derechos con espada y daga; casas17 y pechos18, todo a la malicia; lodos con perejil19 y yerbabuena: esto es la Corte. Buena pro les haga! 5

10

10

nunca merezcan mis ausentes ojos ver tu muro, tus torres y tu ro, tu llano y sierra, oh patria, oh flor de Espaa!

Suspiros tristes, lgrimas cansadas


Suspiros tristes, lgrimas cansadas, que lanza el corazn, los ojos llueven, los troncos baan y las ramas mueven de estas plantas5 a Alcides consagradas; mas del viento las fuerzas conjuradas los suspiros desatan y remueven, y los troncos las lgrimas se beben, mal ellos y peor ellas derramadas. Hasta en mi tierno rostro aquel tributo que dan mis ojos, invisible mano de sombra o de aire me le deja enjuto, porque aquel ngel fieramente humano no crea mi dolor, y as es mi fruto llorar sin premio y suspirar en vano. 5
4 5 6 7 8 9

por plumas espadas: se refiere a los cordobeses ilustres: Sneca, Juan de Mena y el Gran Capitn. Genil y Dauro: se refiere a los dos ros de Granada. plantas: lamos. grandes: ttulo nobiliario. abada: rinoceronte. ttulos: nobles. liberales: prdigos, generosos. ilustri cavaglier: en italiano, ilustres caballeros. llaves doradas: las de los del servicio de la cmara real. hbito: se refiere a los caballeros de las rdenes militares. damas tocas: infieles y viudas alegres. carrozas: las mejores solan tener cuatro caballos. catarriberas: abogados, alcaldes y corregidores. Brtulos y Abades: jurisconsultos del derecho civil y cannico. casas malicia: las construidas para librarse de la obligacin de alojar a los servidores del rey. pechos: voluntades. perejil: excrementos, inmundicias que se arrojaban a la calle.
MATERIAL FOTOCOPIABLE / Oxford University Press Espaa, S. A.

10 11 12

10

13 14 15 16 17

gran ro: traduccin literal de Guadalquivir. 2doradas: las distingue del Dauro, que llevaba oro.
1

18 19

1 Qu elementos de su ciudad natal, alaba Gngora en los cuartetos de A Crdoba? 2 Explica cmo expresa la aoranza y el amor a su tierra en los tercetos. 3 Qu recurso aparece en el ltimo terceto? 4 Segn los cuartetos del segundo texto, cmo se siente la voz potica? 5 Qu relacin se establece entre los suspiros y las lgrimas y los elementos de la naturaleza? Indica las correlaciones que hay en cada verso.

6 Quin es el ngel fieramente humano? Explica cul es la conclusin expresada en los tercetos. 7 Menciona el tema del soneto Suspiros tristes, lgrimas cansadas. 8 Qu se describe en el tercer texto? 9 Haz una relacin de los tipos humanos que componen la enumeracin de los cuartetos. 10 Qu tpico desarrolla el primer terceto? 11 Cul es la imagen final? En qu tipo de sonetos incluiras este poema?

276 16.

La poesa y la prosa barrocas

ACTIVIDADES

REFUERZO

16

3. Sonetos de Francisco de Quevedo


Se ha hecho una clasificacin temtica de la poesa de Quevedo, que incluye los poemas amorosos, los metafsicos y morales, los religiosos y los satricos y burlescos. Aqu se reproducen tres sonetos que tratan los temas ms importantes.

Vivir es caminar breve jornada


Vivir es caminar breve jornada y muerte viva es, Lico, nuestra vida, ayer al frgil cuerpo amanecida, cada instante en el cuerpo sepultada. Nada, que siendo, es poco y ser nada en poco tiempo, que ambiciosa olvida, pues de la vanidad mal persuadida anhela duracin tierra animada. Llevada de engaoso pensamiento y de esperanza burladora y ciega, tropezar en el mismo monumento, como el que divertido el mar navega y sin moverse vuela con el viento, y antes que piense en acercarse llega. 5

A Apolo siguiendo a Dafne


Bermejazo3 platero de las cumbres, a cuya luz se espulga4 la canalla, la ninfa5 Dafne, que se afufa6 y calla, si la quieres gozar, paga y no alumbres. Si quieres ahorrar de pesadumbres, ojo del cielo, trata de compralla: en confites gast Marte7 la malla8, y la espada en pasteles y en azumbres9. Volvise en bolsa10 Jpiter severo; levantse las faldas la doncella por recogerle en lluvia de dinero. Astucia fue de alguna duea11 estrella12, que de estrella sin duea no lo infiero: Febo13, pues eres sol, srvete de ella. 5

10

10

hidrpica: sedienta. reino del espanto: el Infierno. bermejazo: rojizo, pelirrojo. espulgar: limpiar de pulgas. ninfa: diosa, y en germana (jerga de ladrones y malhechores), prostituta. afufar: huir, escapar. Voz de germana. Marte: dios romano de la guerra. Quevedo da una versin burlesca de los amores de Marte y Venus. malla: parte de la armadura parecida a una red. azumbre: medida de capacidad. bolsa: odre, y tambin, bolsa para el dinero. Jpiter se convirti en lluvia de oro para seducir a Dnae. duea: alcahueta; mujer que no es doncella. estrella: embustera y astro. Febo: nombre romano del dios Apolo.

En los claustros de lalma la herida


En los claustros de lalma la herida yace callada, mas consume hambrienta la vida, que en mis venas alimenta llama por las medulas extendida. Bebe el ardor hidrpica1 mi vida que ya, ceniza amante y macilenta, cadver del incendio hermoso ostenta su luz en humo y noche fallecida. La gente esquivo y me es horror el da; dilato en largas noches negro llanto, que a sordo mar mi ardiente pena enva. A los suspiros di la voz de el canto; la confusin inunda lalma ma; mi corazn es reino del espanto2. 5

2 3 4 5

6 7

8 9

10

10

11 12 13

MATERIAL FOTOCOPIABLE / Oxford University Press Espaa, S. A.

1 Cmo define la vida el yo potico en el primer cuarteto de Vivir es caminar breve jornada? Qu tpico literario aparece? 2 Segn el segundo cuarteto, qu desea, sin embargo, el ser humano? 3 Qu idea desarrollan los tercetos? 4 Menciona los temas del texto y reconoce los principales recursos expresivos. 5 A qu herida se refiere la voz potica en el segundo texto? Qu efectos tiene y en qu convierte su vida? 6 Cmo se comporta el yo potico? Cules son sus sentimientos?

7 Qu motivos se desarrollan en el segundo soneto? Reconoce las metforas referidas al amor.

8 Cul es el consejo que le da la voz potica


a Apolo en el tercer texto? 9 Qu ejemplos de historias mitolgicas utiliza como argumentos? Cul es la conclusin? 10 Qu visin del amor aparece en el tercer poema? 11 Analiza los principales recursos expresivos utilizados en A Apolo siguiendo a Dafne. 12 Adscribe los tres sonetos en la clasificacin temtica de la poesa de Francisco Quevedo.

16.

La poesa y la prosa barrocas

277

ACTIVIDADES

AMPLIACIN

16

4. Lope y otros poetas barrocos


Adems de la obra potica de Lope de Vega, destacan en este perodo la de otros autores como Francisco de Rioja (Sevilla, h. 1583-1639) y Gabriel Bocngel (Madrid, 1603-1658), en cuyas composiciones aparecen tratados los temas ms importantes de la lrica del barroco.

Definicin de amor
Desmayarse, atreverse, estar furioso, spero, tierno, liberal, esquivo, alentado, mortal, difunto, vivo, leal, traidor, cobarde y animoso; no hallar fuera del bien centro y reposo, mostrarse alegre, triste, humilde, altivo, enojado, valiente, fugitivo, satisfecho, ofendido, receloso; huir el rostro al claro desengao, beber veneno por licor save, olvidar el provecho, amar el dao; creer que un cielo en un infierno cabe, dar la vida y el alma a un desengao: esto es amor: quien lo prob lo sabe.
Lope DE VEGA Poesa completa, Castalia

Huye del sol el sol, y se deshace


Huye del sol el sol, y se deshace la vida a manos de la propia vida; del tiempo que, a sus partos homicida, en mies de siglos las edades pace, 5 nace la vida, y con la vida nace del cadver la fbrica temida. Qu teme, pues, el hombre en la partida, si vivo estriba en lo que muerto yace? Lo que pas ya falta; lo futuro an no se vive; lo que est presente no est, porque es su esencia el movimiento. Lo que se ignora es solo lo seguro; este mundo, repblica de viento que tiene por monarca un accidente.
Gabriel BOCNGEL Poesa lrica del Siglo de Oro, Ctedra

10

10

Pura, encendida rosa


Pura, encendida rosa, mula de la llama que sale con el da, cmo naces tan llena de alegra si sabes que la edad que te da el cielo es apenas un breve y veloz vuelo? Y ni valdrn las puntas de tu rama, ni prpura hermosa a detener un punto la ejecucin del hado presurosa. El mismo cerco alado que estoy viendo riente, ya temo, amortiguado, presto despojo de la llama ardiente. Para las hojas de tu crespo seno te dio Amor de sus alas blandas plumas, y oro de su cabello dio a tu frente.
1

10

15

Francisco DE RIOJA Poesa lrica del Siglo de Oro Ctedra

de la deidad: Venus, que naci de la espuma del mar, se pinch con una espina y, al teir las rosas blancas con su sangre se volvieron rojas.

1 Qu concepcin del amor desarrolla el soneto de Lope? 2 Qu recursos expresivos utiliza para ello? 3 Segn el segundo poema, a qu est unida la vida desde su nacimiento? 4 Cul es la concepcin del tiempo y de la vida que se expone en los tercetos?

5 Qu descripcin de la rosa aparece en el poema Pura, encendida rosa, de Francisco de Rioja? 6 De qu tema importante del barroco se convirti en un smbolo la rosa? 7 Relaciona los temas de estos tres poemas con los de la lrica barroca.

278 16.

La poesa y la prosa barrocas

MATERIAL FOTOCOPIABLE / Oxford University Press Espaa, S. A.

Oh fiel imagen suya peregrina! Bate en su color sangre divina de la deidad 1 que dieron las espumas. Y esto, purprea flor, esto no pudo hacer menos violento el rayo agudo? Rbate en una hora, rbate licencioso su ardimiento el color y el aliento: tiendes an no las alas abrasadas y ya vuelan al suelo desmayadas: tan cerca, tan unida est al morir tu vida, que dudo si en sus lgrimas la aurora mustia, tu nacimiento o muerte llora.

20

25

30

ACTIVIDADES

AMPLIACIN

16

5. La fuerza del amor, de Mara de Zayas


Mara de Zayas y Sotomayor es una de las principales representantes de la evolucin de la novela corta, dentro de la variedad de la novela cortesana, a la que aport la originalidad de un punto de vista femenino. En 1637 public sus Novelas amorosas y ejemplares, y en 1647, la Parte segunda del sarao y entretenimiento honesto, conocida tambin con el nombre de Desengaos. Las Novelas constan de un marco narrativo (las reuniones en casa de Lisis) en el que, a su vez, se desarrolla una intriga amorosa, y de una serie de relatos narrados por los personajes del marco. Los temas principales de estas historias son el amor apasionado y firme, la deshonra y la venganza que esta desencadena, los celos, el sometimiento de la mujer y la violencia que se ejerce sobre ella, as como la bsqueda de independencia y libertad por parte de las mujeres. La autora rechaza la superioridad del hombre y denuncia la marginacin y subordinacin de la mujer, a quien se le vedaba la educacin y la cultura. Cuestiona, adems, el sistema patriarcal, que impona el matrimonio como nico destino de la mujer en general y como solucin en caso de violacin. La fuerza del amor es una de las narraciones de las Novelas amorosas y ejemplares. En este relato, Laura, la protagonista, se casa con Diego, un galn muy enamorado al principio, pero que, poco despus de la boda, reanuda sus relaciones con una novia anterior y empieza a despreciar a su esposa, llegando hasta el maltrato fsico. En el siguiente fragmento la protagonista habla consigo misma.

La fuerza del amor


Desdichada de ti, Laura, y cmo fueras ms venturosa si como le cost tu nacimiento la vida a tu madre, fuera tambin la tuya sacrificio de la muerte! Oh amor, enemigo mortal de las gentes! Y qu de males han venido por ti al mundo, y ms a las mujeres que como en todo somos las ms perdidosas y las ms fciles de engaar, parece que solo contra ellas tienes el poder, o por mejor decir, el enojo. No s para qu el cielo me cre hermosa, noble y rica, si todo haba de tener tan poco valor contra la desdicha, sin que tantos dotes de naturaleza y fortuna me quitasen la mala estrella en que nac. O, ya que lo soy, para qu me guarda la vida?, pues tenerla un desdichado ms es agravio que ventura. A quin contar mis penas que me las remedie? Quin oir mis quejas que se enternezca? Y quin ver mis lgrimas que me las enjugue? Nadie, por cierto, pues mi padre y hermanos, por no orlas me han desamparado, y hasta el cielo, consuelo de los afligidos, se hace sordo por no drmele. Ay don Diego, y quin lo pensara! Mas s debiera pensar, si mirara que eres hombre, cuyos engaos quitan el poder a los mismos demonios y hacen ellos lo que los ministros de maldades dejan de hacer. Dnde se hallar un hombre verdadero? En cul dura la voluntad un da, y ms si se ven queridos?, que parece que al paso que conocen el amor, crece su libertad y aborrecimiento. Malhaya la mujer que en ellos cree, pues al cabo hallar el pago de su amor, como yo le hallo! Quin es la necia que desea casarse, viendo tantos y tan lastimosos ejemplos?, pues la que ms piensa que acierta, ms yerra. Cmo es mi nimo tan poco, mi valor tan afeminado y mi cobarda tanta que no quito la vida, no solo a la enemiga de mi sosiego, sino al ingrato que me trata con tanto rigor? Mas, ay, que tengo amor! Y en lo uno temo perderle, y en lo otro enojarle. Por qu, vanos legisladores del mundo, atis nuestras manos para las venganzas, imposibilitando nuestras fuerzas con vuestras falsas opiniones, pues nos negis letras y armas? El alma no es la misma que la de los hombres? Pues si ella es la que da valor al cuerpo, quin obliga a los nuestros a tanta cobarda? Yo os aseguro que si entendierais que tambin haba en nosotros valor y fortaleza, no os burlarais como os burlis. Y as, por tenernos sujetas desde que nacemos, vais enflaqueciendo nuestras fuerzas con los temores de la honra, y el entendimiento con el recato de la vergenza, dndonos por espadas, ruecas, y por libros, almohadillas. Mas triste de m! De qu me sirven estos pensamientos, pues ya no sirven para remediar cosas tan sin remedio?
Mara DE ZAYAS Y SOTOMAYOR Novelas amorosas y ejemplares Ctedra

MATERIAL FOTOCOPIABLE / Oxford University Press Espaa, S. A.

1 A quin increpa Laura al principio del texto? 2 Explica qu reproche le hace la joven a Diego, su marido, en el segundo prrafo y, por extensin, a todos los hombres. 3 A quines se dirige Laura a continuacin? Explica de qu los acusa.

4 Indica de qu recurso expresivo se vale la autora para formular los reproches de Laura. 5 Distingue en el fragmento las partes que corresponden al discurso universal y al discurso valorativo. 6 Explica la denuncia de la situacin de la mujer que se expresa al final del texto.

16.

La poesa y la prosa barrocas

279

ACTIVIDADES

REFUERZO

16

6. Guzmn de Alfarache, de Mateo Alemn


El clebre fragmento que reproducimos a continuacin corresponde al principio del libro, cuando Guzmanillo, solo y todava muy inocente, deja su casa y sale a recorrer el mundo. Ha andado mucho, est cansado y hambriento, llega a una venta y pide de comer; el carcter repulsivo del episodio ilustra claramente la existencia del engao, del que es vctima por primera vez.

De lo que le sucedi a Guzmanillo en una venta


Djele [a la ventera] que iba a la corte, que me diese de comer. Hzome sentar en un banquillo cojo y encima de un poyo me puso un barredero de horno, con un salero hecho de un suelo de cntaro, un tiesto de gallinas lleno de agua y una media hogaza ms negra que los manteles. Luego me sac en un plato una tortilla de huevos, que pudiera llamarse mejor emplasto de huevos. Ellos, el pan, jarro, agua, salero, sal, manteles y la huspeda, todo era de lo mismo. Halleme bozal, el estmago apurado, las tripas de posta, que se daban unas con otras de vacas. Com, como el puerco la bellota, todo a hecho; aunque verdaderamente senta crujir entre los dientes los tiernecitos huesos de los sin ventura pollos, que era como hacerme cosquillas en las encas. Bien es verdad que se me hizo novedad y aun en el gusto, que no era como el de los otros huevos que sola comer en casa de mi madre; mas dej pasar aquel pensamiento con la hambre y el cansancio, parecindome que la distancia de la tierra lo causaba y que no eran todos de un sabor ni calidad. Yo estaba de manera que aquello tuve por buena suerte. Tan propio es al hambriento no reparar en salsas, como al necesitado salir a cualquier partido. Era poco, paselo presto con las buenas ganas. En el pan me detuve algo ms. Comilo a pausas, porque siendo muy malo, fue forzoso llevarlo de espacio, dando lugar unos bocados a otros, que bajasen al estmago por su orden. Comencelo por las cortezas y acabelo en el migajn, que estaba hecho engrudo; mas tal cual, no le perdon letra ni les hice a las hormigas migaja de cortesa ms que si fuera poco y bueno. As acontece si se juntan buenos comedores en un plato de fruta, que picando primero en la ms madura, se comen despus la verde, sin dejar memoria de lo que all estuvo. [] As prosegu mi camino, y no con poco cuidado de saber qu pudiera ser aquel taerme castaetas los huevos en la boca. Fui dando y tomando en esta imaginacin, que, cuanto ms la segua, ms gnero de desventuras me representaba y el estmago se me alteraba; porque nunca sospech cosa menos que asquerosa, vindolos tan mal guisados, el aceite negro, que pareca de suelos de candiles, la sartn negra y la ventera legaosa. Entre unas y otras imaginaciones, encontr con la verdad y, teniendo andada otra legua, con solo aquel pensamiento, fue imposible resistirme. Porque, como a mujer preada, me iban y venan eruptaciones del estmago a la boca, hasta que de todo punto no me qued cosa en el cuerpo. Y aun el da de hoy me parece que siento los pobreticos pollos pindome ac dentro. As estaba sentado en la falda del vallado de unas vias, considerando mis infortunios, harto arrepentido de mi mal considerada partida, que siempre se despean los mozos tras el gusto presente, sin respetar ni mirar el dao venidero.
Mateo ALEMN Guzmn de Alfarache, Planeta

1 Qu condiciones presenta la mesa que le sirven en la venta al protagonista? 2 Qu plato le sirve la ventera? Explica qu come en primer lugar y qu piensa el protagonista del estado del ingrediente principal.

3 El protagonista sigue su camino: Qu efectos le causan los alimentos? Qu verdad descubre? 4 Explica la reflexin final del narrador. 5 Seala en este fragmento enunciados de discurso valorativo y de discurso universal.

280 16.

La poesa y la prosa barrocas

MATERIAL FOTOCOPIABLE / Oxford University Press Espaa, S. A.

ACTIVIDADES

REFUERZO

16

7. Una descripcin mordaz


El siguiente texto corresponde a la famosa descripcin del licenciado Cabra, en la que Quevedo muestra su ingenio y su capacidad para la agudeza. Pablos, el protagonista, marcha a Segovia como criado de don Diego Coronel, a quien su padre ha puesto bajo la tutela del licenciado.

El licenciado Cabra
Entramos, primero domingo despus de Cuaresma, en poder de la hambre viva, porque tal laceria1 no admite encarecimiento. l era un clrigo cerbatana, largo2 solo en el talle; una cabeza pequea; los ojos, avecindados en el cogote, que pareca que miraba por cuvanos3, tan hundidos y escuros, que era buen sitio el suyo para tiendas de mercaderes; la nariz, de cuerpo de santo, comido el pico, entre Roma y Francia4, porque se le haba comido de unas bas5 de resfriado, que aun no fueron de vicio porque cuestan dinero; las barbas, descoloridas de miedo de la boca vecina, que, de pura hambre, pareca que amenazaba a comrselas; los dientes, le faltaban no s cuntos, y pienso que por holgazanes y vagamundos se los haban desterrado; el gaznate, largo como de avestruz, con una nuez tan salida, que pareca se iba a buscar de comer forzada de la necesidad; los brazos, secos; las manos, como un manojo de sarmientos cada una; mirado de medio abajo, pareca tenedor o comps, con dos piernas largas y flacas; su andar, muy espacioso: si se descompona algo, le sonaban los gesos como tablillas de San Lzaro6; la habla, tica7; la barba, grande, que nunca se la cortaba por no gastar, y l deca que era tanto el asco que le daba ver la mano del barbero por su cara, que antes se dejara matar que tal permitiese: cortbale los cabellos un muchacho de nosotros. Traa un bonete los das de sol, ratonado con mil gateras, y guarniciones de grasa; era de cosa que fue pao, con los fondos en caspa. La sotana, segn decan algunos, era milagrosa, porque no se saba de qu color era. Unos, vindola tan sin pelo, la tenan por de cuero de rana; otros decan que era ilusin: desde cerca pareca negra, y desde lejos entreazul. Llevbala sin ceidor; no traa cuello ni puos. Pareca, con esto y los cabellos largos y la sotana y el bonetn, teatino lanudo8. Cada zapato poda ser tumba de un filisteo9. Pues su aposento? Aun araas no haba en l. Conjuraba los ratones de miedo que no le royesen algunos mendrugos que guardaba. La cama tena en el suelo, y dorma siempre de un lado por no gastar las sbanas. Al fin, l era archipobre y protomiseria.
Francisco DE QUEVEDO La vida del Buscn, Crtica

1 2 3

laceria: miseria. largo: alto y, tambin, generoso (diloga). cuvanos: cestos de mimbre usados en la vendimia.

tablillas de san Lzaro: las utilizaban las personas enfermas de lepra para avisar de su presencia y pedir limosna. tica: un tipo de fiebre, que afectaba al habla. teatino lanudo: religioso de una orden que sola acompaar a los condenados a muerte. filisteo: hombre de gran estatura y corpulencia.

7 8

MATERIAL FOTOCOPIABLE / Oxford University Press Espaa, S. A.

entre Roma y Francia: alude a que la nariz era roma (pequea y poco puntiaguda) y deforme, como atacada por la sfilis (llamada el mal francs). bas: bubas, tumores blandos.

1 Divide el texto en partes segn los aspectos que abarca la descripcin: partes del cuerpo, vestimenta, habitacin y hbitos. 2 Que rasgo principal se destaca de la conducta del licenciado Cabra? 3 Qu personaje de una novela picaresca anterior tiene caractersticas similares a las del licenciado?

4 Analiza los recursos expresivos y determina cules predominan. 5 Realiza una parfrasis del texto. 6 Relaciona el modo en que se describe al personaje con los rasgos temticos y formales de la obra. 7 Reconoce algunas de las caractersticas propias de un texto descriptivo.

16.

La poesa y la prosa barrocas

281

ACTIVIDADES

AMPLIACIN

16

8. Los Sueos, de Quevedo


En El alguacil endemoniado el narrador habla con un demonio que se ha apoderado del alma de un alguacil, personaje que pertenece a un sector social habitualmente criticado por Quevedo; el diablo advierte que estos golpes que le doy y lo que le aporreo, no es sino que yo y l reimos ac sobre quin ha de estar en mejor lugar y andamos a ms diablo es l. El narrador le pregunta sobre quines estn en el Infierno y sobre sus penas.

Del sitio que a cada cual le corresponde en el Infierno


Y en el Infierno estn todos aposentados as. Que un artillero que baj all el otro da, queriendo que le pusiesen entre la gente de guerra, como al preguntarle del oficio que haba tenido dijese que hacer tiros1 en el mundo, fue remitido al cuartel de los escribanos, pues son los que hacen tiros en el mundo. Un sastre, porque dijo que haba vivido de cortar de vestir2, fue aposentado con los maldicientes. Un ciego, que quiso encajarse con los poetas, fue llevado a los enamorados, por serlo todos. Los que venan por el camino de los locos, ponemos con los astrlogos, y a los por mentecatos, con los alquimistas. Uno vino por unas muertes, y est con los mdicos. Los mercaderes que se condenan por vender, estn con Judas. Los malos ministros, por lo que han tomado, alojan con el mal ladrn. Los necios estn con los verdugos. Y un aguador, que dijo haba vendido agua fra, fue llevado con los taberneros. Lleg un mohatrero3 tres das ha, y dijo que l se condenaba por haber vendido gato por liebre, y pusmoslo de pies con los venteros, que dan lo mismo. Al fin, el Infierno est repartido en estas partes. Ote decir antes de los enamorados, y por ser cosa que a m me toca, gustara saber si hay muchos. Mancha es la de los enamorados respondi que lo toma todo, porque todos lo son de s mismos: algunos, de sus dineros; otros de sus palabras; otros, de sus obras, y algunos, de las mujeres. Y destos postreros hay menos que de todos en el Infierno, porque las mujeres son tales, que, con ruindades, con malos tratos y peores correspondencias les dan ocasiones de arrepentimiento cada da a los hombres. [] Algunos hay que en celos y esperanzas amortajados y en deseos, se van por la posta4 al Infierno, sin saber cmo ni cundo ni de qu manera.
MATERIAL FOTOCOPIABLE / Oxford University Press Espaa, S. A.

Francisco DE QUEVEDO Sueos, Espasa Calpe


1 2 3 4

hacer tiro: engaar. cortar de vestir: murmurar, hablar mal de alguien. mohatrero: persona que engaa, que comete fraudes. por la posta: de prisa, velozmente.

1 Menciona todos los tipos humanos y sociales que se nombran en el primer prrafo. 2 Cmo se ubica en el Infierno a los que van llegando? Explica alguna de las distribuciones que se realizan. 3 Menciona cul es la finalidad de Quevedo al determinar la agrupacin de los condenados. Reconoce los recursos expresivos que le sirven para tal objetivo.

4 Seala los tipos de enamorados. Qu significa la frase todos lo son de s mismos? 5 Por qu hay pocos enamorados de las mujeres en el Infierno? 6 Relaciona el contenido del texto con los temas de los Sueos. 7 En qu otras obras de Quevedo aparecen temas semejantes?

282 16.

La poesa y la prosa barrocas

ACTIVIDADES

AMPLIACIN

16

9. Orculo manual y arte de prudencia, de Baltasar Gracin


Este libro, compuesto por trescientos aforismos, ofrece un arte de prudencia, unas normas de vida constituidas por afirmaciones caracterizadas por su laconismo. A continuacin reproducimos algunos de ellos.

105. No cansar. Suele ser pesado el hombre de un negocio1 y el de un verbo. La brevedad es lisonjera, y ms negociante: gana por lo corts lo que pierde por lo corto. Lo bueno, si breve, dos veces bueno; y aun lo malo, si poco, no tan malo. Ms obran quintas esencias que frragos; y es verdad comn que hombre largo raras veces es entendido, no tanto en lo material de la disposicin, cuanto en lo formal del discurso. Hay hombres que sirven ms de embarazo que de adorno del universo, alhajas perdidas que todos las desvan. Excuse el discreto el embarazar, y mucho menos a grandes personajes, que viven muy ocupados; y sera peor desazonar uno de ellos que todo lo restante del mundo. Lo bien dicho se dice presto.

174. No vivir aprisa. El saber repartir las cosas es saberlas gozar. A muchos les sobra la vida y se les acaba la felicidad; malogran los contentos, que no los gozan, y querran despus volver atrs cuando se hallan tan adelante; postillones2 del vivir, que, a ms del comn correr del tiempo, aaden ellos su atropellamiento genial; querran devorar en un da lo que apenas podran digerir en toda la vida; viven adelantados en las felicidades, cmense los aos por venir y, como van con tanta prisa, acaban presto con todo. Aun en el querer saber ha de haber modo, para no saber las cosas mal sabidas. Son ms los das que las dichas. En el gozar, a espacio; en el obrar, aprisa. Las hazaas, bien estn hechas; los contentos, mal acabados.

195. Saber estimar. Ninguno hay que no pueda ser maestro de otro en algo; ni hay quien no exceda al que excede. Saber disfrutar a cada uno es til saber. El sabio estima a todos, porque reconoce lo bueno en cada uno, y sabe lo que cuestan las cosas de hacerse bien. El necio desprecia a todos por ignorancia de lo bueno y por eleccin de lo peor.

MATERIAL FOTOCOPIABLE / Oxford University Press Espaa, S. A.

246. Nunca dar satisfaccin a quien no la peda. Y aunque se pida, es especie de delito si es sobrada. El excusarse antes de ocasin es culparse, y el sangrarse en salud es hacer del ojo 3 al mal y a la malicia. La excusa anticipada despierta al recelo que dorma. Ni se ha de dar el cuerdo por entendido de la sospecha ajena, que es salir a buscar el agravio; entonces le ha de procurar desmentir con la entereza de su proceder.

299. Dejar con hambre. Hase de dejar en los labios aun con el nctar. Es el deseo medida de la estimacin. Hasta la material sed es treta de buen gusto picarla, pero no acabarla: lo bueno, si poco, dos veces bueno. Es grande la baja de la segunda vez 4. Hartazgos de agrado son peligrosos, que ocasionan desprecios a la ms eterna eminencia. nica regla de agradar, coger el apetito picado con el hambre con que se qued. Si se ha de irritar, sea antes por impaciencia del deseo que por enfado de la fruicin; gstase al doble de la felicidad penada.
Baltasar GRACIN Orculo manual y arte de prudencia Planeta

1 2

negocio: asunto, ocupacin. postilln: mozo que iba a caballo, delante de las postas para guiar y ensear el camino.

3 4

hacer del ojo: avisar, llamar la atencin. es grande la baja de la segunda vez: la segunda vez que algo se aprecia baja en estimacin.

1 El primer aforismo contiene un enunciado muy difundido, cul es? A qu situacin lo aplica el texto? Pon ejemplos de otros casos en que tambin podra utilizarse esta expresin. 2 Establece relaciones temticas entre los aforismos 105 y 299 y localiza en ambos una expresin casi idntica.

3 Cules son los argumentos que fundamentan los consejos de No vivir aprisa y Saber estimar? 4 Por qu conviene no excusarse de lo que uno no ha sido acusado? Qu consecuencias trae? 5 Redacta un texto sobre el vivir deprisa en la sociedad actual (cmo se manifiesta y en quines, cules son sus motivos y justificaciones, qu juicios te merecen, etc.).

16.

La poesa y la prosa barrocas

283

Evaluacin 16

Alegora de una poca fugaz, por Antoni da Pereda.

Huye sin percibirse lento el da


Huye sin percibirse lento el da, y la hora secreta1 y recatada con silencio se acerca, y despreciada2 lleva tras s la edad lozana ma. La vida nueva, que en niez arda, la juventud robusta y engaada, en el postrer invierno3 sepultada, yace entre negra sombra y nieve fra. No sent resbalar mudos los aos; hoy los lloro pasados y los veo riendo de mis lgrimas y daos. Mi penitencia deba a mi deseo, pues me deben la vida mis engaos4 y espero el mal que paso y no le creo.

1. Reconoce el gnero al que pertenece el texto y justifica


los motivos de esa adscripcin.

2. Explica el contenido del poema y determina el o los


temas que desarrolla.

3. Qu motivo propio de la poesa de Quevedo aparece


5 en este texto? En qu tipos de obras lo trata?

4. Realiza el anlisis mtrico de la composicin. Qu otras


formas mtricas se cultivaron en el barroco?

5. Analiza los recursos expresivos ms destacados del texto.


10

6. Enuncia y explica otros tipos de poesa que haya cultivado


Quevedo y menciona las caractersticas de su estilo.

7. Realiza una sntesis de la obra potica de Gngora y


de las caractersticas estilsticas ms importantes.

Francisco DE QUEVEDO Poesa completa, Planeta

8. Menciona las obras principales de Lope de Vega, los


temas tratados en general y la particularidad de su estilo.

9. Relaciona los temas de la poesa y la prosa barrocas con


1 2 3 4

secreta: ignorada, escondida. despreciada: la juventud (la edad lozana). postrer invierno: la vejez. me deben la vida mis engaEos: los engaos le han quitado la vida.

el contexto histrico, social y literario de su produccin.

10. Redacta un texto en que reflexiones, en general, sobre


la brevedad de la vida y su final inevitable (por ejemplo, si has pensado en ello alguna vez o es una realidad percibida como muy lejana, cmo afrontarla, si compartes la actitud de los poetas barrocos ). 275 16.
La poesa y la prosa barrocas

ACTIVIDADES

AMPLIACIN

17

1. Auto de los Reyes Magos


El Auto o Representacin de los Reyes Magos, perteneciente al ciclo del Ordo Stellae (escenificado en las fiestas de la Epifana), es el ms antiguo drama sacro en romance que se conserva. Consta de los monlogos de los Reyes, su encuentro y la decisin de peregrinar juntos, la visita a Herodes y el dilogo de este con sus consejeros.

Encuentro de los Magos con Herodes


GASPAR.(Dirigindose a HERODES, que se acaricia la negra y breve barba con gesto impaciente.) Gurdete el Creador. Dios te libre de mal. Te entretendremos poco Un instante no ms. (HERODES, apresurado, le estimula a continuar. GASPAR considera prudente repetir el saludo.) Dios te d larga vida, y te libre de mal. Vamos en romera, para a un rey adorar, que ha nacido en la tierra No logrmosle hallar. HERODES.(Sbitamente interesado.) Qu decs? Dnde vais? A quin vais a buscar? De qu tierra vens? Dnde intentis llegar? Decidme vuestros nombres, no los queris celar. GASPAR.Yo me llamo Gaspar; este que ves, Melchor; y aquel es Baltasar. (GASPAR no interpreta bien el inters de HERODES. Atropellado e ingenuo le comunica la nueva.) Rey, un rey ha nacido que es seor de la tierra, que mandar en el mundo con gran paz y sin guerra! HERODES.Es as en verdad? GASPAR.S, rey, por caridad. HERODES.Y cmo lo sabis? Probado lo tenis? GASPAR.Rey, verdad te diremos: probado lo tenemos. MELCHOR.(Sin poder contenerse, seala hacia el cielo.) Esto es gran maravilla: un astro nuevo brilla! BALTASAR.Seal de que ha nacido y que en carne ha venido! (Nueva pausa. La voz de HERODES tiembla levemente, mientras mira a lo alto.) HERODES.Cunto hace que le visteis y que lo percibisteis? GASPAR.Trece das son ya. (Volvindose a sus compaeros.) Creo que ms no har desde que est advertido y muy bien percibido. HERODES.(Con tono que sera amable si no le delatara el temblor de la voz.) Pues andad y buscad, y a ese rey adorad, y por aqu tornad (Ligera pausa. El REY no mira ya a los MAGOS.) Si lo encontris, ir, y yo le adorar. (Se apaga la luz del espacio central. GASPAR sale, tras haberse inclinado ante HERODES. Este, solo, se pone en pie. Ya no tiene por qu disimular su clera.) Quin vio nunca tal mal? Sobre un rey, otro tal! An no estoy yo muerto ni bajo tierra puesto. Otro rey sobre m? Nunca tal cosa vi! El mundo va hacia atrs: no s qu me haga ya. Juro que no lo crea hasta que no lo vea.
Teatro medieval Castalia

MATERIAL FOTOCOPIABLE / Oxford University Press Espaa, S. A.

1 Explica por qu el fragmento del Auto de los Reyes Magos pertenece a un texto dramtico. 2 Resume el contenido del dilogo que mantienen los personajes. Cul es el conflicto que presenta? Explcalo brevemente. 3 Caracteriza los personajes que intervienen en este pasaje atendiendo especialmente a sus palabras.

4 Las acotaciones fueron incorporadas por el profesor Fernando Lzaro Carreter, en la edicin que prepar del Auto. Seala a qu aspectos de los cdigos visuales y auditivos de la representacin hacen referencia. 5 Reconoce en las intervenciones de los personajes que participan en el dilogo, ejemplos de las funciones del lenguaje expresiva, apelativa y referencial.

17.

El teatro: de los orgenes a la comedia nacional

297

ACTIVIDADES

AMPLIACIN

17

2. Representacin del nacimiento de Nuestro Seor


El escritor Gmez Manrique compuso hacia 1460 la Representacin del nacimiento de Nuestro Seor, para su hermana doa Mara Manrique, vicaria en el convento de clarisas de Calabazanos (Palencia). Para esta composicin dramtica, el poeta se basa en los evangelios cannicos y emplea formas mtricas de la poesa cancioneril del siglo XV.

La desconfianza de Jos
LO QUE DICE JOS, SOSPECHANDO DE NTRA. SEORA: Oh viejo desventurado! Negra dicha fue la ma, en casarme con Mara por quien fuese deshonrado. Yo la veo bien preada; no s de quin ni de cunto. Dicen que d Espritu Santo, mas yo d esto no s nada. LA ORACIN QUE HACE LA GLORIOSA: Mi solo Dios verdadero, cuyo ser es inmovible, a quien es todo posible, fcil y bien facedero1! T que sabes la pureza de la mi virginidad, alumbra la ceguedad de Jos y su simpleza. EL NGEL A JOS: Oh viejo de muchos das, en el seso de muy pocos, el principal de los locos! T no sabes que Isaas dijo: Virgen parir, lo cual escribi por esta doncella gentil, honesta, cuyo par nunca ser? 10 LA QUE REPRESENTA A LA GLORIOSA, CUANDO LE DIEREN EL NIO: Adrote, rey del cielo, verdadero Dios y hombre; adoro tu santo nombre, mi salvacin y consuelo. Adrote, hijo y padre, a quien sin dolor par, porque quisiste de m hacer de sierva tu madre. Bien podr decir aqu aquel salmo gloroso que dije, hijo precioso, cuando yo te conceb: que mi nima2 engrandece a ti, mi solo seor, y en ti, mi salvador, mi espritu florece. Mas este mi gran placer en dolor ser tornado, pues t eres enviado para muerte padecer por salvar los pecadores en la cual yo pasar, no mengundome la fe, innumerables dolores. 25

30

35

15

40

45

20

Gmez MANRIQUE Representacin del nacimiento de Nuestro Seor en Ronald E. SURTZ, Teatro castellano de la Edad Media Taurus (actualizacin)

1 Explica en qu consiste el conflicto que se plantea en el inicio de este fragmento de la Representacin de nacimiento de Nuestro Seor. 2 Cmo caracteriza a Jos la Virgen Mara en su primera intervencin? Qu le reprocha el ngel?

3 Identifica el tipo de estrofa empleado en este fragmento dramtico. 4 Incorpora al texto algunas acotaciones que se refieran a distintos elementos de los cdigos que intervienen en una representacin teatral (escenografa, gestos, movimientos).

298 17.

El teatro: de los orgenes a la comedia nacional

MATERIAL FOTOCOPIABLE / Oxford University Press Espaa, S. A.

facedero: hacedero; que se puede hacer.

nima: alma.

ACTIVIDADES

AMPLIACIN

17

3. Una gloga de Juan del Encina


En el siguiente fragmento corresponde al comienzo de la gloga en la que Mengo y un escudero pugnan por el amor de la pastorcilla Pascuala.

gloga representada en requesta1 de unos amores


MINGO.Pascuala, Dios te mantenga. PASCUALA.Nora buena vengas, Mingo. Oy, qu es da de domingo, no ests con tu esposa Menga? MINGO.No ay quien all me detenga, quel cario que te tengo me pone un quejo2 tan luengo que me acossa que me venga. PASCUALA.Y no praga3 a Dios contigo, y aun con tu esposa Menguilla! Cmo dexas tu esposilla por venirte ac comigo? MINGO.Soncas4, soncas, no te digo que eres, zagala, tan bella que te quiero ms que a ella? Dios lo sabe, qu es testigo. PASCUALA.Miaf 5 , Mingo, no te creo que de m ests namorado. Pues eres ya desposado, tu querer no lo desseo. MINGO.Ay, Pascuala, que te veo tan lozana y tan garrida, que yo te juro a mi vida que deslumbro si te oteo! Y porque eres tan hermosa te quiero; mira, vers, quireme, quireme ms, pues por ti dejo a mi esposa. Y toma, toma esta rosa que para ti la cog, aunque no curas de m ni por m se te da cosa. PASCUALA.O, qu chapados olores! Mingo, Dios te d salud, y gozes la juventud ms que todos los pastores.
1 2 3 4

10

15

20

25

30

MATERIAL FOTOCOPIABLE / Oxford University Press Espaa, S. A.

35

MINGO.Y t dasme mil dolores. Dame, dame una manija, o siquiera essa sortija, que traya por tus amores. PASCUALA.Tirte6, tirte all, Minguillo, no te quellotres7 de vero! Hete8, viene un escudero; vea que eres pastorcillo. Sacude tu caramillo9, tu hondijo10 y tu cayado; haz que aballas11 el ganado, silva, hurria12, da gritillo. ESCUDERO.Pastora, slvete Dios. PASCUALA.Dios os d, seor, buen da. ESCUDERO.Guarde Dios tu galana. PASCUALA.Escudero, ass haga a vos. ESCUDERO.Tienes ms gala que dos de las de mayor beldad. PASCUALA.Essos que sois de ciudad perchufis13 huerte14 de nos. ESCUDERO.Desso no tengas temor. Por mi vida, pastorcica, que te haga presto rica PASCUALA.Essas trnicas15, seor, all para las de villa. ESCUDERO.Vete comigo, carilla16. Dexa, dexa esse pastor. Dxalo, que Dios te vala; no te pene su penar, que no te sabe tratar segn requiere tu gala. MINGO.Estate queda, Pascuala, no te engae este traidor, palaciego, burlador, que ha burlado otra zagala.

40

45

50

55

60

65

70

Juan DEL ENCINA Teatro y poesa, Taurus


9

requesta: desafo. quejo: dolor, pena. praga: agrade. soncas: en verdad, a fe.

5 6 7 8

miaf: por mi fe, s. tirte: aprtate. quellotres: lamentes. hete: mira.

caramillo: flauta de caa. hondijo: honda. aballas: arreas. hurria: arrea.

13 14 15 16

perchufis: burlis. huerte: fuerte, mucho. trnicas: retricas. carilla: compaera.

10 11 12

1 Por qu puede adscribirse este texto de Juan del Encina al gnero dramtico? 2 Resume el contenido del fragmento. 3 Caracteriza los personajes y sus actitudes. 4 Analiza la mtrica empleada.

5 Caracteriza el habla de los tres personajes de esta gloga. 6 Reconoce en el texto un recurso caracterstico de la poesa cancioneril; quin lo emplea? 7 Compara estos pastores con los de las glogas de Garcilaso de la Vega.

17.

El teatro: de los orgenes a la comedia nacional

299

ACTIVIDADES

AMPLIACIN

17

4. Un paso de Lope de Rueda


Lope de Rueda fue el principal creador de entremeses, escenas cmicas con autonoma dramtica, conocidas en su poca como pasos. Estas obras breves se escribieron para ser representadas en un espectculo ms amplio, como una comedia. De Lope de Rueda se conservan ms de veinte pasos, que se suelen clasificar en cuatro grupos, segn el tipo de protagonista: los de simple, los de negra, los de lacayo fanfarrn y los de otras figuras. El siguiente es un fragmento del paso Las aceitunas.

Las aceitunas
ALOJA.Qus esto, vecinos? Por qu me maltratis ans la mochacha? GUEDA.Ay, seor! Este mal hombre que me quiere dar las cosas a menos precio y quiere echar a perder mi casa. Unas aceitunas que son como nueces! TORUVIO.Yo juro a los huesos de mi linaje que no son ni aun como piones. GUEDA.S son! TORUVIO.No son! ALOJA.Ora, seora vecina, haceme tamao placer que os entris all dentro, que yo lo averiguar todo. GUEDA.Averige o pngase todo del quebranto1. ALOJA.Seor vecino, qu son de las aceitunas? Sacaldas ac fuera, que yo las comprar, aunque sean veinte hanegas2. TORUVIO.Que no, seor, que no es desa manera que vuesa merced se piensa; que no estn las aceitunas aqu en casa, sino en la heredad3. ALOJA.Pues traeldas aqu, que yos las comprar todas al precio que justo fuere. MENCIGELA.A dos reales quiere mi madre que se venda el celemn4. ALOJA.Cara cosa es esa. TORUVIO.No le paresce a vuesa merced?
1

MENCIGELA.Y mi padre a quince dineros. ALOJA.Tenga yo una muestra dellas. TORUVIO.Vlame Dios, seor! Vuesa merced no me quiere entender. Hoy he yo plantado un renuevo5 de aceitunas y dice mi mujer que de aqu a seis o siete aos llevar cuatro o cinco hanegas de aceituna y quella la cogera y que yo la acarrease y la mochacha la vendiese. Y que, a la fuerza de drecho6, haba de pedir a dos reales por cada celemn. Yo, que no, y ella, que s. Y sobre esto ha sido la cuistin. ALOJA.Oh, qu graciosa cuistin! Nunca tal se ha visto. Las aceitunas no estn plantadas y ha llevado la mochacha tarea sobre ellas. MENCIGELA.Qu le paresce, seor? TORUVIO.No llores, rapaza; la mochacha, seor, es como un oro. Ora andad, hija, y ponedme la mesa, que yos prometo de hacer un ayuelo de las primeras aceitunas que se vendieren. ALOJA.Ora, andad, vecino; entraos all dentro y ten paz con vuestra mujer. TORUVIO.Adis, seor. ALOJA.Ora, por cierto, qu cosas vemos en esta vida que ponen espanto! Las aceitunas no estn plantadas, ya las habemos visto reidas. Razn ser que d fin a mi embajada7.
Lope DE RUEDA Pasos completos, Espasa Calpe
MATERIAL FOTOCOPIABLE / Oxford University Press Espaa, S. A.

averige del quebranto: esta frase se ha interpretado como averige o djese todo a favor de la prdida econmica. hanegas: fanegas, medida de capacidad equivalente a 55 litros y medio. heredad: posesin en el campo.

celemn: medida de algo ms de cuatro litros y medio. renuevo: vstago que echa una planta podada. a la fuerza de drecho: por derecho. embajada: se llamaba as a la presentacin y embajador al presentador.

5 6 7

1 Justifica por qu el fragmento de Lope de Rueda pertenece a un texto dramtico. 2 Identifica los personajes que intervienen en este dilogo sealando la relacin que existe entre ellos. 3 Por qu acude Aloja? Explica cul es el conflicto.

4 Clasifica el paso Las aceitunas en uno de los grupos sealados en la explicacin que aparece ms arriba. 5 Atendiendo a las palabras finales de Aloja, sobre qu crees que tratar la comedia que se puede representar despus de este paso? Razona tu respuesta.

300 17.

El teatro: de los orgenes a la comedia nacional

ACTIVIDADES

REFUERZO

17

5. Rasgos de la comedia nacional


Repasa los rasgos de la comedia nacional y luego lee atentamente el siguiente fragmento de La dama duende, de Caldern de la Barca.

El honor familiar
LUIS.No hay accin que me suceda bien, Rodrigo. Si una dama veo airosa, y conocerla solicito, me detienen un necio y una pendencia; que no s cual es peor; si rio, y mi hermano llega, es mi enemigo su amigo; si por disculpa me deja de una dama, es una dama que mil pesares me cuesta: de suerte que una tapada me huye, un necio me atormenta, un forastero me mata, y un hermano me le lleva a ser mi husped a casa, y otra dama me desprecia De mal anda mi fortuna! RODRIGO.Qu de todas esas penas que s la que sientes ms? LUIS.No sabes. RODRIGO. Que la que llegas a sentir ms, son los celos de tu hermana y Beatriz bella? LUIS.Engaste. RODRIGO. Pues, cul es? 25 LUIS.Si tengo de hablar de veras de ti solo me fiara, lo que ms siento es que sea mi hermano tan poco atento, que llevar a casa quiera un hombre mozo, teniendo, Rodrigo, una hermana en ella, viuda y moza; y, como sabes, tan de secreto, que apenas sabe el sol que vive en casa; porque Beatriz, por ser deuda, solamente la visita. RODRIGO.Ya s que su esposo era administrador de puertos de mar en unas reales rentas, y qued debiendo al rey grande cantidad de hacienda; y ella a la corte se vino de secreto, donde intenta, escondida y retirada, componer mejor sus deudas. Y esto disculpa a tu hermano, pues, si mejor consideras que su estado no le da ni permisin ni licencia de que nadie la visite, y que, aunque tu husped sea don Manuel, no ha de saber que en casa, seor, se encierra tal mujer, qu inconveniente hay en admitirle en ella? Y ms habiendo tenido tal recato y advertencia, que para su cuarto ha dado por otra calle la puerta, y la que sala a la casa, por desmentir la sospecha, de que el cuidado la haba cerrado, o porque pudiera otra vez, fabric en ella una alacena de vidrios, labrada de tal manera, que parece que jams en tal parte ha habido puerta. LUIS.Ves con lo que me aseguras? Pues con eso mismo intentas darme muerte, pues ya dices que no ha puesto por defensa de su honor ms que unos vidrios, que al primer golpe se quiebran. (Vanse y salen DOA NGELA e ISABEL.) DOA NGELA.Vulveme a dar, Isabel, esas tocas; pena esquiva!, vuelve a amortajarme viva, ya que mi suerte crel lo quiere as. ISABEL. Toma presto; porque si tu hermano viene y alguna sospecha tiene, no la confirme con esto, de hallarte desta manera, que hoy en palacio te vio. 45

50

10

55

15

60

20

65

70

30

75

MATERIAL FOTOCOPIABLE / Oxford University Press Espaa, S. A.

35

80

40

Pedro CALDERN DE LA BARCA La dama duende Planeta

1 Resume la situacin planteada en la escena. 2 Analiza la mtrica empleada y caracterzala. 3 Explica cmo son los personajes.

4 Qu rasgos de la comedia nacional estn presentes en estos versos de Caldern? 5 Explica en qu consisten aquellos otros rasgos que no se evidencian en el fragmento.

17.

El teatro: de los orgenes a la comedia nacional

301

ACTIVIDADES

AMPLIACIN

17

6. Acotaciones de una comedia cortesana


En 1651, Baccio del Bianco realiz en el parque del Coliseo del Buen Retiro de Madrid un excepcional montaje de La fiera, el rayo y la piedra, de Caldern de la Barca. Las siguientes son algunas de las acotaciones incluidas en esta comedia cortesana.

Texto 1
(Oscurcese el teatro, que ser de peascos con el foro de marina; y mientras se dicen los primeros versos, se descubre la perspectiva del mar, y habr truenos y relmpagos.)

Texto 6
(Al entrarse por otra parte huyendo, salen PIGMALEN y LEBRN.)

Texto 10
(Vuela CUPIDO, mdase el teatro en el de monte, y en el foro, la fragua de VULCANO; y salen por una parte LISI, CLORI, LAURA e ISABELLA, con arcos y flechas y varios instrumentos en las manos; y por la otra ANAJARTE, en traje de cazadora, con venablo, y OTRAS.)

Texto 7 Texto 2
(Sale IRIFILE, vestida de pieles, suelto el cabello.) (Mdase el teatro en el bosque, y en el foro la gruta de LAS PARCAS.)

Texto 11
(Pnense las cuatro a las cuatro puntas del tablado, retranse ANAJARTE y las otras damas, y mientras cantan, sale IRIFILE, acechando.) (Suenan dentro los martillos de la fragua.)

Texto 3
(Atraviesan varios peces por la marina.)

Texto 8
(brese la gruta y vese en lo ms lejos de ella a LAS TRES PARCAS; la primera con una rueca, cuyo hilo va a dar a la tercera, que lo devana, dejando en medio a la segunda con unas tijeras en la mano.)

Texto 4 (Pasan algunas sirenas, cantando.)

Texto 12
(Descbrese la fragua, y los CCLOPES cantan al son de los martillos.) (Desaparece la fragua y pasa en una nube ANTEROS, atravesando el teatro con un venablo en la mano.)
Pedro CALDERN DE LA BARCA La fiera, el rayo y la piedra Ibero-Americana de Publicaciones

Texto 5
(Atraviesan algunos bajelillos por la marina.) (Empieza a aclarar.)

Texto 9
(Cantando, en tono muy triste.) (Cirrase la gruta.)

1 Todas estas acotaciones estn incluidas en La fiera, el rayo y la piedra, comedia cortesana de Caldern de la Barca. Seala aquellas posturas referidas a cuestiones relacionadas con los personajes (gestos, movimientos, vestuario). 2 Cules de estas acotaciones hacen mencin de elementos relacionados con la utilera de los personajes? Y con la utilera de escena?

3 Seala qu indicaciones se ofrecen acerca de la ambientacin de la obra. Diferencia entre: Elementos visuales. Elementos auditivos. 4 Despus de analizar estas acotaciones, relaciona su informacin con lo que sabes acerca del teatro barroco, especialmente el teatro cortesano.

302 17.

El teatro: de los orgenes a la comedia nacional

MATERIAL FOTOCOPIABLE / Oxford University Press Espaa, S. A.

ACTIVIDADES

AMPLIACIN

17

7. Los autos sacramentales


Recuerda las caractersticas de los autos sacramentales y luego lee el siguiente fragmento, que corresponde al final de El gran teatro del mundo, de Pedro Caldern de laBarca.

El autor y sus personajes


RICO.Si el poder y la hermosura por aquella vanagloria que tuvieron, con haber llorado, tanto te asombran, y el labrador que a gemidos enterneciera una roca est temblando de ver la presencia poderosa de la vista del Autor, cmo oso mirarla ahora? Mas es preciso llegar, pues no hay adonde me esconda de su riguroso juicio. Autor! AUTOR. Cmo as me nombras? Que aunque soy tu Autor, es bien que de decirlo te corras, pues que ya en mi compaa no has de estar. De ella te arroja mi poder. Desciende adonde te atormente tu ambiciosa condicin eternamente entre penas y congojas. RICO.Ay de m! Que envuelto en fuego caigo arrastrando mi sombra donde ya que no me vea yo a m mismo, duras rocas sepultarn mis entraas en tenebrosas alcobas. DISCRECIN.Infinita gloria tengo. HERMOSURA.Tenerla espero dichosa. LABRADOR.Hermosura, por deseos no me llevars la joya. RICO.No la espero eternamente. NIO.No tengo, para m, gloria. AUTOR.Las cuatro postrimeras son las que presentes notan vuestros ojos, y porque destas cuatro se conozca que se ha de acabar la una, suba la Hermosura ahora con el Labrador, alegres, a esta mesa misteriosa, pues que ya por sus fatigas merecen grados de gloria.

10

15

(Suben los dos.) HERMOSURA.Qu ventura! LABRADOR. RICO.Qu desdicha! Qu consuelo! Qu victoria! Qu alivio! Qu ponzoa! 50 45

20

REY. DISCRECIN. POBRE.Qu dulzura!

RICO.Qu sentimiento!

25

RICO.

NIO.Gloria y pena hay, pero yo no tengo pena ni gloria. AUTOR.Pues el ngel en el cielo, en el mundo las personas y en el infierno el demonio todos a este Pan se postran; en el infierno, en el cielo y mundo a un tiempo se oigan dulces voces que le alaben acordadas y sonoras. (Tocan chirimas, cantando el Tantum ergo muchas veces.) MUNDO.Y pues representaciones en aquesta vida toda, merezca alcanzar perdn de las unas y las otras.

30

55

MATERIAL FOTOCOPIABLE / Oxford University Press Espaa, S. A.

35

40

60

Pedro CALDERN DE LA BARCA El gran teatro del mundo Ctedra

1 Explica el contenido de este fragmento. 2 Qu caracteriza a los personajes? 3 Analiza la mtrica empleada en estos versos de El gran teatro del mundo.

4 Identifica las figuras estilsticas del texto y explica su funcin. 5 Qu caractersticas de los autos sacramentales observas en el fragmento?

17.

El teatro: de los orgenes a la comedia nacional

303

ACTIVIDADES

AMPLIACIN

17

8. El burlador de Sevilla
El burlador de Sevilla, atribuida a Tirso de Molina, constituye una creacin dramtica original que, aunque utiliza componentes previos, crea uno de los personajes ms importantes de la literatura: el don Juan, que gener una importante descendencia. En este fragmento, tras engaar a una mujer noble, don Juan seduce a Tisbea, una pescadora, prometindole casamiento; pero en verdad piensa huir una vez consumados sus propsitos.

El engao de don Juan


DON JUAN.Mientras que los pescadores van de regocijo y fiesta, t las dos yeguas apresta1, que de sus pies voladores solo nuestro engao fo. CATALINN.Al fin pretendes gozar a Tisbea? DON JUAN.Si el burlar es hbito antiguo mo, qu me preguntas, sabiendo mi condicin? CATALINN. Ya s que eres castigo de las mujeres. DON JUAN.Por Tisbea estoy muriendo, que es buena moza. CATALINN. Buen pago a su hospedaje deseas. DON JUAN.Necio, lo mismo hizo Eneas2 con la reina de Cartago. CATALINN.Los que fings, y engais las mujeres de esa suerte, lo pagaris en la muerte. DON JUAN.Qu largo me lo fiis! Catalinn3 con razn te llaman. CATALINN. Tus pareceres sigue, que en burlar mujeres quiero ser Catalinn. Ya viene la desdichada. DON JUAN.Vete, y las yeguas prevn. CATALINN.Pobre mujer, harto bien te pagamos la posada. (Vase CATALINN y sale TISBEA.)
3

TISBEA.El rato que sin ti estoy estoy ajena de m. DON JUAN.Por lo que finges as ningn crdito te doy. 5 TISBEA.Por qu? DON JUAN. Porque si me amaras mi alma favorecieras. TISBEA.Tuya soy. 10 DON JUAN. Pues di, qu esperas, o en qu, seora, reparas? TISBEA.Reparo en que fue castigo de amor el que he hallado en ti. DON JUAN.Si vivo, mi bien, en ti a cualquier cosa me obligo, aunque yo sepa perder en tu servicio la vida, la diera por bien perdida, y te prometo de ser tu esposo.

30

35

40

15

45
Tirso DE MOLINA El burlador de Sevilla Ctedra

20

25

aprestar: aparejar, preparar, disponer lo necesario para algo.


MATERIAL FOTOCOPIABLE / Oxford University Press Espaa, S. A.

Eneas: legendario hroe de la guerra de Troya que abandon a la reina Dido para seguir rumbo a Italia, dejndola enamorada y desesperada. Catalinn: probablemente, cobarde.

1 Resume el contenido de estos versos de El burlador de Sevilla y relacinalo con lo que sabes sobre el personaje de don Juan. 2 Explica la actitud que mantiene el criado Catalinn ante su amo y la respuesta que este le da.

3 Qu mtrica emplea en estos versos Tirso de Molina? A qu tipo de estructura estrfica corresponde? 4 Analiza los principales recursos estilsticos presentes en este fragmento de El burlador de Sevilla.

304 17.

El teatro: de los orgenes a la comedia nacional

Evaluacin

17

Representacin de El alcalde de Zalamea, dirigida por Gustavo Prez Puig y Mara Recatero (2003).

La dignidad del villano


JUAN.Que quieras, siendo t rico, vivir a estos hospedajes sujeto! PEDRO CRESPO. Pues cmo puedo excusarlos ni excusarme? JUAN.Comprando una ejecutoria. PEDRO CRESPO.Dime por tu vida, hay alguien que no sepa, que yo soy, si bien de limpio linaje, hombre llano? No por cierto. Pues, qu gano yo en comprarle una ejecutoria al Rey, si no le compro la sangre? Dirn entonces, que soy mejor que ahora? No, es dislate. Pues qu dirn? Que soy noble por cinco o seis mil reales; y esto es dinero y no es honra; que honra no la compra nadie. Quieres, aunque sea trivial, un ejemplillo escucharme? Es calvo un hombre mil aos, y al cabo de ellos se hace una cabellera. Este, en opiniones vulgares, deja de ser calvo? No. Pues qu dicen al mirarle? Bien puesta la cabellera trae Fulano. Pues qu hace, si, aunque no le vean la calva, todos que la tiene saben? JUAN.Enmendar su vejacin, remediarse de su parte, y redimir vejaciones del sol, del hielo y del aire. PEDRO CRESPO.Yo no quiero honor postizo, que el defecto ha de dejarme en casa. Villanos fueron mis abuelos y mis padres; sean villanos mis hijos.

25

30

10

35

15

20

Pedro CALDERN DE LA BARCA El alcalde de Zalamea, Ctedra

1. En esta escena, Juan le propone a su padre Pedro Crespo


que compre una ejecutoria carta que exima de pagar tributos al rey , para evitar alojar a don lvaro. Resume los argumentos que esgrime el alcalde para negarse.

6. Cul era el orden que se segua normalmente en las


representaciones de los corrales de comedia? Caracteriza las piezas cortas que se representaban.

2. Qu tema del teatro barroco observas en este dilogo? 3. Analiza la mtrica empleada en este fragmento y reconoce los principales recursos expresivos utilizados.

7. Seala los rasgos propios de los autos sacramentales. 8. Establece diferencias entre el teatro de Lope de Vega
y el de Caldern. Menciona obras de uno y otro.

4. Enuncia los rasgos de la comedia nacional y explcalos.


Cules de ellos reconoces en el fragmento?

9. Cules son los principales hitos del teatro renacentista


que influyeron en el teatro barroco? Recuerda autores y obras del siglo & VI.

5. Explica las diferencias entre el teatro cortesano y las


representaciones de los corrales de comedia.

10. Redacta una exposicin sobre las manifestaciones


dramticas en la Edad Media. 295 17.
El teatro: de los orgenes a la comedia nacional

ACTIVIDADES

REFUERZO

18

1. Poesa dieciochesca
En las primeras dcadas del siglo XVIII se sigui cultivando una poesa de rasgos barrocos, pero despus esta prctica evolucion en distintas direcciones. A continuacin, puedes leer algunas composiciones representativas de las distintas manifestaciones que adopt la poesa en el denominado Siglo de las Luces.

De mi gusto
Retrico molesto, deja de persuadirme que ocupe bien el tiempo y a mi Dorila olvide. Ni t tampoco quieras con rplicas sutiles. del nctar de Leo1 hacer que me desve. Ni t, que al feroz Marte muy ms errado sigues, me angusties con pintarme lo horrendo de sus lides. Empero habladme todos de bailes y de brindis, de juegos y de amores, de olores y convites, que tras la edad florida corre la vejez triste, y antes que llegue quiero holgarme y divertirme. 5

A la muerte de Filis
En lgubres cipreses he visto convertidos los pmpanos de Baco y de Venus los mirtos2; cual ronca voz del cuervo hiere mi triste odo el siempre dulce tono del tierno jilguerillo; ni murmura el arroyo con delicioso trino; resuena cual peasco con olas combatido. En vez de los corderos de los montes vecinos rebaos de leones bajar con furia he visto; del sol y de la luna los carros fugitivos esparcen negras sombras mientras dura su giro; las pastoriles flautas, que taen mis amigos, resuenan como truenos del que reina en Olimpo. Pues Baco, Venus, aves, arroyos, pastorcillos, sol, luna, todos juntos miradme compasivos, y a la ninfa3 que amaba al infeliz Narciso, mandad que diga al orbe la pena de Dalmiro.

El len y el ratn
Estaba un ratoncillo aprisionado en las garras de un len; el desdichado en tal ratonera no fue preso por ladrn de tocino ni de queso, sino porque con otros molestaba al len, que en su retiro descansaba. Pide perdn, llorando su insolencia; al or implorar la real clemencia, responde el Rey en majestuoso tono (no dijera ms Tito): Te perdono. Poco despus, cazando, el len tropieza en una red oculta en la maleza; quiere salir, mas queda prisionero; atronando la selva ruge fiero. El libre ratoncillo, que lo siente, corriendo llega; roe diligente los nudos de la red de tal manera que al fin rompi los grillos de la fiera. Conviene al poderoso para los infelices ser piadoso; tal vez se pueda ver necesitado del auxilio de aquel ms desdichado.

10

10

10

15

15

15

20

20

20

Juan MELNDEZ VALDS Poesa y prosa, Planeta

Flix Mara SAMANIEGO en Poesa del siglo XVIII, Castalia

A la invencin de la imprenta
Lleg, pues, el gran da en que un mortal divino, sacudiendo de entre la mengua universal la frente, con voz omnipotente dijo a la faz del mundo: EL HOMBRE [ES LIBRE. Y esta sagrada aclamacin saliendo, no en los estrechos lmites hundida se vio de una regin: el eco grande que invent GUTENBERG la alza [en sus alas; y en ellas conducido se mira en un momento salvar los montes, recorrer los mares, ocupar la extensin del vago viento, y sin que el trono o su furor la asombre, por todas partes el valiente grito sonar de la razn: LIBRE ES EL [HOMBRE.

25

30

Jos CADALSO

4 Por qu se puede considerar prerromntico el texto de Jos Cadalso? Explica su proceso comunicativo interno y el contenido del mensaje. 5 A qu tipo de poesa del siglo XVIII pertenecen los versos de Quintana? Justifica tu respuesta. 6 Razona a qu gnero y subgnero pertenece el texto de Samaniego. En qu grupo de la poesa del siglo XVIII se incluye? 7 Analiza la mtrica y la rima de El len y el ratn y divdelo en partes. 8 Seala los recursos expresivos que observes en los cuatro textos.

MATERIAL FOTOCOPIABLE / Oxford University Press Espaa, S. A.

10

1 Justifica la adscripcin del texto De mi gusto a la poesa rococ.


15

Manuel Jos QUINTANA

nctar de Leo: referencia al vino; Leo (el que relaja) es uno de los apelativos que reciba el dios griego Dioniso. 2mirto: arrayn. Arbusto de dos o tres metros de altura, oloroso, con ramas flexibles y hojas opuestas de color verde vivo. 3ninfa: se refiere a la ninfa Eco.
1

2 Analiza la mtrica y la rima del texto de Juan Melndez Valds (caractersticas de la anacrentica) y explica el proceso comunicativo interno del poema. 3 Compara el tono de la anacrentica de Cadalso con la de Melndez Valds.

18.

La literatura del siglo

XVIII

323

ACTIVIDADES

AMPLIACIN

18

2. La tragedia neoclsica: Raquel


Raquel, de Vicente Garca de la Huerta, se estren con gran xito en 1778 en el teatro Prncipe de Madrid, pero ya se haba representado en Orn en 1772, donde estaba desterrado su autor. La obra, que trata de los amores de Alfonso VIII con una juda de Toledo, sirve a Garca de la Huerta para expresar su opinin acerca de cmo deba ser una monarqua en su poca. Raquel responde a las caractersticas de la tragedia neoclsica: respeta las unidades de tiempo, espacio y accin, adems del principio de verosimilitud, posee un nmero reducido de personajes y pretende brindar una enseanza ejemplar.

Despreciada
RAQUEL.Ay de m! qu he escuchado? T, [Alvar Fez, explcame este arcano. ALVAR FEZ. Pues te avisan que eres la ocasin de tantos males, la respuesta te puedes dar t misma RAQUEL. (A Rubn.) Estoy despierta o sueo [por ventura? RUBN.No s, Raquel: la misma duda agita mi discurso y razn, imaginando qu es cuanto he visto, sueo o fantasa. RAQUEL.Qu especie de dolor tan inhumano es este, oh corazn, que por primicias de los males y sustos que me aguardan, me ofrece la tirana suerte ma? Quin de tanto favor se prometiera tan no esperada, tan mortal cada? Y quin hecha, fortuna, a tus halagos pudiera recelarse tal desdicha? Alfonso me aborrece; sus desvos de mis temores la verdad confirman: pues cmo podr ser ya venturosa, la que se ve de Alfonso aborrecida? Qu necio quien se fa de la suerte, sin advertir que el tiempo y que los das, que ciudades destruyen y edificios, favores y privanzas aniquilan! Qu causa puede haber, amado Alfonso, para tanto desvo? Mis caricias en qu te han ofendido, que por premio solo odio y desagrado se concilian? Mas ay de m! que en vano me desvelo en buscar la ocasin de mis fatigas; pues la suerte que empieza a perseguirme, por doblarme el dolor, querr encubrirla. 10 5 RUBN.As, Raquel, tu corazn desmaya en tan fuerte ocasin, donde es precisa la constancia mayor? En los principios si un mal, aunque sea leve, se descuida, fuerzas del abandono va cobrando, que el remedio despus inutilizan. Reciente es este mal; an se est en tiempo de poderle acudir; quien averigua la causa de un dolor, con ms acierto aplicarle podr la medicina. Inquirase, Raquel, de esta desgracia la ocasin; que despus de conocida, si no cede a remedios ordinarios, buscar los extremos mi malicia. RAQUEL.Bien, Rubn, me aconsejas: en qu dudas? Al yugo vuelva la cerviz altiva, segunda vez Alfonso: el fin se logre, y el medio sea cualquiera, que t elijas. Lcito es cuanto sea conveniente: propia moral de la venganza ma (Ruido dentro.) Mas, ay de m! Qu estrpito confuso orse deja? Al alma pronostica el corazn, latiendo apresurado, algn cercano mal.

35

40

45

50

15

55

20

25

RUBN. Ya ms distintas se perciben las voces: nunca pruebas mayores dio de s la cobarda, que al escuchar rumor tan temeroso. (Voz dentro.) Muera Raquel, para que Alfonso viva. 60
MATERIAL FOTOCOPIABLE / Oxford University Press Espaa, S. A.

30

RAQUEL.No es delirio: verdad es la que toco: y esto sufre mi enojo?, esto mis iras? Espera, vulgo brbaro, atrevido, que si mi sangre a derramar conspiras, vers que a costa de la tuya sabe defender y guardar Raquel su vida.

65

Vicente GARCA DE LA HUERTA Raquel, Compaa Ibero-Americana de Publicaciones

1 Lee el fragmento y explica qu ocurre en l. 2 Cules son los sentimientos que embargan a Raquel? Para responder, atiende especialmente a su tercera intervencin. 3 Cmo reacciona Rubn? Qu le aconseja? 4 Se produce un cambio en la actitud de la protagonista hacia el final del fragmento? Argumenta tu respuesta.

5 Analiza la mtrica y la rima empleada: es la que se emplea en toda la obra. 6 Encuentra en el texto ejemplos de discurso valorativo y de discurso universal, y de las funciones del lenguaje. 7 Reconoce en el fragmento recursos expresivos. 8 Establece diferencias entre la tragedia y la comedia neoclsicas.

324 18.

La literatura del siglo

XVIII

ACTIVIDADES

AMPLIACIN

18

3. La comedia sentimental: El delincuente honrado


En 1787 se estrena El delincuente honrado, de Gaspar Melchor de Jovellanos, obra cumbre de la comedia sentimental. La obra desarrolla una historia melodramtica centrada en un personaje, Torcuato, que aos atrs haba participado en un duelo en el que haba matado al anterior marido de su esposa. El fragmento que tienes a continuacin corresponde a la parte inicial de la comedia.

Errores antiguos
ANSELMO.A fe, amigo mo, que me has hecho bien mala obra. Dejar la cama a las siete de la maana! Hombre, no lo hara ni por una duquesa; mas tu recado fue tan ejecutivo (Despus de alguna pausa.) Pero, Torcuato, t ests triste Tus ojos Vaya, apostemos a que has llorado? TORCUATO.En mi dolor apenas he tenido ese pequeo desahogo. ANSELMO.Desahogo las lgrimas? No lo entiendo. Pues qu, un hombre como t no se correra1? TORCUATO.Si las lgrimas son efecto de la sensibilidad del corazn, desdichado de aquel que no es capaz de derramarlas! ANSELMO.Como quiera que sea, yo no te comprendo. Torcuato, tus ojos estn hinchados, tu semblante triste, y de algunos das a esta parte noto que has perdido tu natural alegra. Qu es esto? Cuando debieras Hombre, vamos claros; quieres que te diga lo que he pensado? T acabas de casarte con Laura, y por ms que la quieras, tener una mujer para toda la vida, sufrir a un suegro viejo e impertinente, empezar a sentir la falta de la dulce libertad y el peso de las obligaciones del matrimonio, son sin duda para un joven graves motivos de tristeza, y ve aqu a lo que atribuyo la tuya. Pero si esta es la causa, t no tienes disculpa, amigo mo, porque te la has buscado por tu mano. Por otra parte, Laura es virtuosa, es linda, tiene un genio dcil y amable, te quiere mucho; y t, que has sido siempre derretido, creo que no la vas en zaga. Sobre todo (viendo que no le responde), Torcuato, t no debes afligirte por frioleras; goza, con sosiego, de las dulzuras del matrimonio, que ya llegar el da en que cada cual tome su partido.
MATERIAL FOTOCOPIABLE / Oxford University Press Espaa, S. A.
1

TORCUATO.Ay, Anselmo! Esas dulzuras, que pudieran hacerme tan dichoso, se van a cambiar en pena y desconsuelo; yo las voy a perder para siempre. ANSELMO.A perderlas? Pues qu? Ah! (Dndose una palmada en la frente.) Ahora me acuerdo de que tu criada me dijo no s qu de un viaje Pero yo estaba tan dormido TORCUATO.T eres mi amigo, Anselmo, y voy a darte ahora la ltima prueba de mi confianza. ANSELMO.Pues sea sin prembulos, porque los aborrezco. Puedo servirte en algo? Mi caudal, mis fuerzas, mi vida, todo es tuyo; di lo que quieras, y si es preciso TORCUATO.Ya sabes que fui autor de la muerte del marqus de Montilla, y que este funesto 2 secreto, que hoy llena mi vida de amargura, se conserva entre los dos. ANSELMO.Es verdad; pero en cuanto al secreto no hay que recelar. T sabes tambin cunto hice con Juanillo, el criado del marqus, para alejar toda sospecha; pues aunque solo tena algunos antecedentes del desafo, yo le gratifiqu, le traspuse a Madrid, donde nadie le conoce, y mi amigo, el marqus de la Fuente, est encargado de observar sus pasos. No; lejos de pensar en ti ese bribn, tal vez creer Pero no hablemos de eso, porque no es posible TORCUATO.Ay, Anselmo, cunto te engaas! Ese criado est ya en las crceles de Segovia.
Gaspar Melchor DE JOVELLANOS Poesa. Teatro. Prosa Taurus

no se correra: no se avergonzara.

funesto: aciago, que es origen de pesares o de ruina.

1 En qu estado se encuentra Torcuato al inicio de la escena? 2 A qu atribuye Anselmo lo que le ocurre a su amigo? 3 Explica por qu cree Anselmo que no hay que preocuparse por el suceso acaecido y por qu, por del contrario, Torcuato cree que s.

4 Encuentra en el fragmento ejemplos de las funciones expresiva, apelativa y referencial. 5 Busca en las palabras de Torcuato un ejemplo de discurso universal y explica su sentido. 6 A qu hace referencia el texto secundario? 7 Establece diferencias entre la comedia sentimental y la comedia neoclsica.

18.

La literatura del siglo

XVIII

325

ACTIVIDADES

AMPLIACIN

18

4. La prosa dieciochesca: Vida


Diego de Torres Villarroel (Salamanca, 1694-1770) aprovecha la estructura de la novela picaresca, gnero que diverta a los lectores, para narrar su vida. Pero Villarroel cuenta, a diferencia del pcaro que medra para mejorar su condicin, cmo progresa gracias a su esfuerzo y mritos. Adems, sus orgenes no son miserables, sino que sus familiares, honrados, se ocupan de su instruccin. En su relato emplear los recursos de la burla y la irona, tpicamente quevedescos. Los sucesos narrados son pocos y su finalidad es aportar datos para ir configurando la personalidad de su autor, objetivo de la obra, como puedes leer en el siguiente fragmento que corresponde a la Introduccin.

Un hombre de novela
Por lo mismo que ha tardado mi muerte, ya no puede tardar; y quiero, antes de morirme, desvanecer con mis confesiones y verdades, los enredos y mentiras que me han abultado los crticos y los embusteros. La pobreza, la mocedad, lo desentonado de mi aprensin, lo ridculo de mi estudio, mis almanaques, mis coplas y mis enemigos me han hecho hombre de novela, un estudiantn extravagante y un escolar1 entre brujo y astrlogo, con visos2 de diablo y perspectivas3 de hechicero. Los tontos que pican en eruditos me sacan y me meten en sus conversaciones, y en los estrados y las cocinas, detrs de un aforismo4 del calendario, me injieren una ridcula quijotada y me pegan un par de aventuras descomunales; y, por mi desgracia y por su gusto, ando entre las gentes hecho un mamarracho, cubierto con el sayo que se les antoja, y con los parches e hisopadas5 de sus negras noticias. Paso, entre los que me conocen y me ignoran, me abominan y me saludan, por un Guzmn de Alfarache, un Gregorio de Guadaa6 y un Lzaro de Tormes; y ni soy este, ni aquel, ni el otro; y por vida ma, que se ha de saber quin soy. Yo quiero meterme en corro; y ya que cualquier monigote presumido se toma de mi murmuracin, murmuremos a medias, que yo lo puedo hacer con ms verdad y con menos injusticia y escndalo que todos. Sgase la conversacin, y crea despus el mundo a quien quisiere. No me mueve a confesar en el pblico mis verdaderas liviandades el deseo de sosegar los chismes y las parleras con que anda alborotado mi nombre y forajida mi opinin7, porque el espritu no se altera con el aire de las alabanzas, ni con el ruido de los vituperios. A todo el mundo le dejo garlar 9 y decidir sobre lo que sabe o lo que ignora, sobre m o sobre quien agarra el vuelo su voluntad, su rabia o su costumbre. Desde muy nio conoc que de las gentes no se puede pretender ni esperar ms justicia ni ms misericordia que la que no le haga falta a su amor propio. En los empeos de poca o mucha consideracin, cada uno sigue su comodidad y sus ideas. Al que me alaba, no se lo agradezco, porque, si me alaba, es porque le conviene a su modestia o su hipocresa, y a ellas puede pedir las gracias que yo no debo darle. Al que me corrige, le oigo y lo dejo descabezar10; rome mucho de ver cmo presume de consejero muy repotente11 y gustoso de sus propias satisfacciones.
Diego DE TORRES VILLARROEL Vida, ascendencia, nacimiento, crianza y aventuras DeBolsillo
1

escolar: posible juego de palabras entre la acepcin de atribuirse tanto a profesores como estudiantes que convivan en una escuela y a la de nigromntico y embustero que finge ser un pobre estudiante y mendiga. viso: apariencia de las cosas. perspectivas: falsas apariencias. aforismo: sentencia breve y doctrinal que se propone como regla. hisopadas: rociadas de agua hechas con el hisopo. Gregorio de Guadaa: protagonista de la novela picaresca El siglo pitagrico y vida de Don Gregorio Guadaa (1644), de Antonio Enrquez Gmez. opinin: aqu fama, honra. vituperio: baldn u oprobio que se dice a uno. garlar: hablar mucho y con poca discrecin. descabezar: (coloq.) empezar a vencer la dificultad o tropiezo que se encuentre en algo. repotente: engredo, prepotente.
MATERIAL FOTOCOPIABLE / Oxford University Press Espaa, S. A.

2 3 4

5 6

7 8 9

10

11

1 Qu objetivo manifiesta el yo narrador al comienzo? Qu visin presenta de su persona? A qu la atribuye? 2 Justifica la presencia de los nombres propios citados.

3 Explica la finalidad del tercer prrafo en relacin con lo dicho en el segundo. 4 Encuentra en el texto ejemplos de discurso universal. 5 Seala los recursos expresivos empleados.

326 18.

La literatura del siglo

XVIII

ACTIVIDADES

AMPLIACIN

18

5. Ensayo e Ilustracin
El ensayo espaol est estrechamente vinculado a la difusin de las ideas de la Ilustracin. El autor del siguiente texto reflexiona sobre este fenmeno y brinda algunos de los rasgos ms importantes que caracterizaron este gnero en el siglo XVIII.

Autores ilustrados
Sin necesidad de establecer una ecuacin perfecta, se puede afirmar que el gnero ensayo est ntimamente vinculado al fenmeno cultural que se conoce con el nombre de Ilustracin. Gran parte del saber erudito, antes expresado en latn, se vierte ahora a la lengua verncula1: ideas, antes solo asequibles a los que haban aprendido una lengua culta o un idioma extranjero, llegan a un amplio pblico, que se arroga, adems, la facultad de discurrir y sentenciar sobre todo lo divino y lo humano. Este fenmeno histrico condiciona esencialmente la escritura y modifica el lenguaje, hacindolo ms gil y familiar. Aquellos modelos textuales que se adaptan mejor al espritu y al contenido de mensajes dirigidos al conjunto de los ciudadanos atraen especialmente la atencin de los escritores. Pero la tenaz curiosidad por conocer novedades, tanto a nivel cientfico como poltico, desplaza a segundo plano cuestiones meramente formales o retricas. Cuando sobre el tapete se ponen temas como la libertad de pensamiento, los privilegios estamentales 2 del clero y la nobleza, el derecho a opinar en cuestiones acadmicas o gubernamentales, la reforma universitaria y multitud de otros asuntos de inters pblico, da hasta cierto punto igual si sobre ello se escribe en forma de dilogo amistoso, de apasionada oracin o de sosegada exposicin didctica. Los autores se concentran ms en el fondo que en la forma. Indudablemente, lo que el ambiente cultural de la poca no favorece es que alguien prescinda de la tertulia de amigos o que simplemente haga abstraccin del pblico opinante y se encierre en su biblioteca privada a elucubrar 3 sobre el ms all o a departir con autores difuntos. El hombre dieciochesco se considera ante todo miembro de una colectividad contempornea y, por tanto, siente responsabilidad frente a ella. No muestra, pues, demasiada propensin a rumiar solitariamente sobre la brevedad de la vida o sobre consuelos que alivien la aciaga4 fortuna individual, sino que prefiere pensar en plural afrontando sus deberes de patriota y de ciudadano y proponiendo soluciones tiles para el comn. [] Los pensadores del siglo XVIII meditan sobre intereses colectivos, basndose en una concepcin cosmopolita de la razn. [] El ilustrado no pierde nunca de vista el horizonte de la humanidad aun cuando proponga reformas especficas para la propia patria. Ciertamente, muchos ilustrados retienen todava el principio barroco de contribuir al esplendor de la monarqua y del trono, pero no todos creen que el gobierno monrquico sea el nico legtimo o necesariamente el mejor. En este punto, como en otros, no suelen hacer profesin de fe. Un escepticismo generalizado domina la actitud intelectual y hace que todo se someta, primero, a examen y, despus, a la prueba de la experiencia. Por eso se tratan preferentemente aquellos temas que admiten solucin inmediata: tcnicas agrcolas, mejoras de las manufacturas o reformas administrativas. Se confa en el progreso del conocimiento y en el creciente predominio de la naturaleza. El mtodo emprico se aplica a todos los campos del saber. En consecuencia surge una extraordinaria demanda de tratados de fsica, botnica, qumica e incluso zoologa, es decir, de todas las ciencias que pueden contribuir al progreso de la ganadera, de la agricultura y de la industria. Pero ese inters cognoscitivo est subordinado a aumentar la felicidad pblica y privada, lo cual implica tambin mejorar el ordenamiento de la sociedad. Aunque la discusin recuerda las causas de la decadencia de Espaa, el planteamiento de las reformas se enfoca desde una perspectiva cosmopolita, es decir, alzando la mirada por encima de la barrera de los Pirineos y observando lo que hacen las dems naciones.
Francisco SNCHEZ BLANCO Prlogo a El ensayo espaol: el siglo XVIII, Crtica

MATERIAL FOTOCOPIABLE / Oxford University Press Espaa, S. A.

vernculo: domstico, nativo, de nuestra casa o pas. estamental: perteneciente o relativo a un estrato social determinado. 3elucubrar: elaborar una divagacin complicada y con apariencia profunda. 4aciago: infeliz, desgraciado.
1 2

1 Explica la importancia que tuvo, segn el texto de Francisco Snchez Blanco, la irrupcin del ensayo en el siglo XVIII. 2 Por qu con este gnero dejaron de tener relevancia las cuestiones formales? En qu se fijaron, sobre todo, los escritores? 3 Indica qu circunstancias favorecieron el ambiente cultural de la poca que nos ocupa. 4 Qu tiene siempre en cuenta el escritor ilustrado?

5 Seala qu consecuencias tuvo en el pensamiento dieciochesco el escepticismo reinante y a qu campos se aplic el mtodo emprico. 6 Por qu en el perodo ilustrado se publicaron numerosos tratados de determinadas ciencias? Qu se buscaba finalmente? 7 Desde qu perspectiva se abordaron las reformas? 8 Escribe un resumen del texto de Francisco Snchez Blanco.

18.

La literatura del siglo

XVIII

327

Evaluacin 18
Falsas creencias
Levntase el rumor de algn portento1 sucedido en un pueblo, a que dio principio o un embustero o un alucinado, y no pocos, que tienen igual ligereza en la creencia que en la pluma, despachan por la estafeta2 la noticia a otras partes. Aclrase despus la verdad y viene el desengao para aquel pueblo, mas no para los otros donde se comunic la especie, porque los que la escribieron o no se acuerdan (especialmente si medi espacio de tiempo considerable) de escribir el desengao, o no quieren hacerlo, porque no se les atribuya el primer asenso3 a ligereza, con que queda estampada en los otros pueblos la patraa4, porque no lleg el caso de ratificacin a los testigos y deshacer en la plenaria5 el engao padecido en la sumaria6. En todo el pueblo de Llanes (distante de esta capital dieciocho leguas) corri uno de estos aos pasados por indubitable la existencia de un duende, gran enredador, que se deca infestaba continuamente una de las casas de aquella villa. Llegaron aqu repetidas noticias del caso, tan circunstanciadas y citando tantos testigos de vista que aun los ms incrdulos de duendes llegaron a dar asenso, y de m confieso que estuve harto inclinado a lo mismo. Sin embargo, despus, por muchos y segursimos informes se supo que el duende haba salido fingido, y que dos muchachas, con un enredillo bien poco artificioso, haban puesto a todo el pueblo en aquella creencia. Pero, quin duda que el engao que con facilidad pudo venir aqu por el continuo comercio que hay entre los dos lugares no llegara a otros muchos adonde se haba escrito el embuste?
Benito Jernimo FEIJOO Obras (Seleccin), Taurus
1

portento: cosa, accin o suceso singular que por su extraeza o novedad causa admiracin o terror. estafeta: casa u oficina del correo. asenso: asentimiento; aquello a lo que se da crdito. patraEa: mentira o noticia fabulosa, de pura invencin. plenaria: general, plena. sumaria: reducida; parte de algo.
Las brujas, por Francisco de Goya.

2 3 4 5 6

1. Resume el contenido de este texto de Benito Jernimo


Feijoo.

6. Comenta las principales vertientes poticas del siglo &


y nombra los principales autores.

VIII

2. Indica cul es el tema que trata el autor en este fragmento y menciona qu otros asuntos suele abordar en sus escritos.

7. Cul es la finalidad del teatro neoclsico? Explica las


caractersticas de sus dos principales manifestaciones. Ejemplifica cada una de ellas con una obra y su autor.

3. Reconoce en el texto las expresiones valorativas y los


principales recursos estilsticos.

8. Seala otras orientaciones dramticas del siglo


nombra los dramaturgos ms destacados.

& VIII

4. Seala la diferencia que existe entre Benito Jernimo


Feijoo y Gaspar Melchor de Jovellanos en cuanto a los destinatarios de sus escritos. Qu temas aborda este ltimo autor? Menciona obras de ambos escritores.

9. Menciona algunas de las principales instituciones ilustradas.

10. Escribe un texto sobre la literatura predominante en la


primera mitad del siglo & VIII y la que comienza a cultivarse en la segunda mitad de la centuria. No olvides citar las obras y los autores ms importantes. 309 18.
La literatura del siglo
XVIII

5. En qu obra analiza Cadalso la situacin de Espaa?


Explica cmo la estructura.

ACTIVIDADES

REFUERZO

19

1. Las acotaciones en el teatro romntico


La escenografa tuvo un desarrollo muy importante en el siglo XIX. Al levantarse el teln, la escena se converta en un lugar de ilusin que se procuraba que reprodujese fielmente los espacios reales y los imaginados por los dramaturgos, y en los cuales tuvo gran relevancia la iluminacin.

Jornada primera Escena I


La escena representa la entrada del antiguo puente de barcas de Triana, el que estar practicable a la derecha. En primer trmino, al mismo lado, un aguaducho1 o barraca de tablas y lonas, con un letrero que diga: Agua de Tomares; dentro habr un mostrador rstico con cuatro grandes cntaros, macetas de flores, vasos, un anafre2 con una cafetera de hojalata y una bandeja con azucarillos. Delante del aguaducho habr bancos de pino. Al fondo se descubrir de lejos, parte del arrabal de Triana, la huerta de los Remedios con sus altos cipreses, el ro y varios barcos en l, con flmulas3 y gallardetes4. A la izquierda se ver en lontananza la Alameda. Varios habitantes de Sevilla cruzarn en todas direcciones durante la escena. El cielo demostrar el ponerse del sol en una tarde de julio [].

Jornada segunda Escena I


Es de noche, y el teatro representa la cocina de un mesn de la villa de Hornachuelos5. Al frente estar la chimenea y el hogar. A la izquierda, la puerta de entrada; a la derecha, dos puertas practicables. A un lado, una mesa larga de pino, rodeada de asientos toscos, y alumbrado todo por un gran candiln6. El MESONERO y el ALCALDE aparecern sentados gravemente al fuego. La MESONERA, de rodillas, guisando. Junto a la mesa, el ESTUDIANTE, cantando y tocando la guitarra. El ARRIERO que habla, cribando7 cebada en el fondo del teatro. El TO TRABUCO, tendido en primer trmino sobre sus jalmas7. Los dos LUGAREOS, las dos LUGAREAS, la MOZA y uno de los ARRIEROS, que no habla, estarn bailando seguidillas. []

Jornada primera Escena III


Empieza a anochecer, y se va oscureciendo el teatro. DON LVARO sale embozado en una capa de seda, con un gran sombrero blanco, botines y espuelas; cruza lentamente la escena, mirando con dignidad y melancola a todos los lados, y se va por el puente. Todos lo observan en gran silencio.

Jornada quinta Escena X


El teatro representa un valle rodeado de riscos inaccesibles y de malezas, atravesado por un arroyuelo. Sobre un peasco accesible con dificultad, y colocado al fondo, habr una medio gruta, medio ermita, con puerta practicable, y una campana que pueda sonar y tocarse desde dentro; el cielo representar el ponerse del sol de un da borrascoso, se ir oscureciendo lentamente la escena y aumentndose los truenos y los relmpagos. []
Duque DE RIVAS Don lvaro o la fuerza del sino, Ctedra

MATERIAL FOTOCOPIABLE / Oxford University Press Espaa, S. A.

aguaducho: puesto donde se vende agua. anafre: hornillo. flmula: gallardete muy corto. gallardete: tira volante que disminuye hasta acabar en punta colocada en los mstiles. Hornachuelos: villa de la provincia de Crdoba. candiln: candil grande.
8 7

2 3 4

5 6

cribar: limpiar el trigo u otra semilla por medio de la criba, del polvo, tierra y dems impurezas. jalma: aparejo que se emplea en las bestias de carga.

1 Reconoce en las cuatro acotaciones reproducidas en esta pgina la presentacin de cuadros costumbristas, sealando sus elementos. 2 Cmo se presenta el protagonista en la escena III de la jornada primera? Ten en cuenta que es la primera vez que aparece en escena.

3 Seala en los textos elementos relacionados con los cdigos visuales, detallando a qu elementos de la representacin hacen referencia. 4 Localiza ahora los elementos referidos a los cdigos auditivos. 5 Qu tipo de atmsfera se pretende crear con la ltima acotacin?

19.

La literatura del romanticismo

339

ACTIVIDADES

REFUERZO

19

2. Don lvaro o la fuerza del sino, del duque de Rivas


El siguiente fragmento corresponde a la escena VII de la jornada primera de una obra cumbre del teatro romntico, Don lvaro o la fuerza del sino, de ngel de Saavedra, duque de Rivas. El protagonista se encuentra con su amada: estn dispuestos a huir juntos, pero Leonor se encuentra muy abatida.

La huida
DOA LEONOR. Don lvaro! Leonor! (Pausa.) DON LVARO. Fuerza bastante hay para todo en m Desventurado! La conmocin conozco que te agita, inocente Leonor. Dios no permita que por debilidad en tal momento sigas mis pasos y mi esposa seas. Renuncio a tu palabra y juramento; hachas de muerte las nupciales teas fueran para los dos Si no me amas como te amo yo a ti Si arrepentida DOA LEONOR.Mi dulce esposo, con el alma y vida es tuya tu Leonor; mi dicha fundo en seguirte hasta el fin del ancho mundo. Vamos; resuelta estoy, fij mi suerte, separarnos podr solo la muerte. (Van hacia el balcn, cuando de repente se oye ruido, ladridos y abrir y cerrar de puertas.) DOA LEONOR.Dios mo! Qu ruido es ese? Don lvaro! CURRA.Parece que han abierto las puertas del patio y la de la escalera DOA LEONOR.Se habr puesto malo mi padre? CURRA.Qu! No, seora; el ruido viene de otra parte. DOA LEONOR.Habr llegado alguno de mis hermanos? DON LVARO.Vamos, vamos, Leonor; no perdamos ni un instante. (Vuelven hacia el balcn y de repente se ve por l el resplandor de hachones de viento y se oye el galopar de caballos.) DOA LEONOR.Somos perdidos! Estamos descubiertos Imposible es la fuga. DON LVARO.Serenidad es necesario en todo caso. CURRA.La Virgen del Rosario nos valga y las nimas benditas! Qu ser de mi pobre Antonio? (Se asoma al balcn y grita.) Antonio! Antonio! DON LVARO.Calla, maldita! No llames la atencin hacia este lado; entorna el balcn. (Se acerca el ruido de puertas y pisadas.) DOA LEONOR.Ay, desdichada de m! Don lvaro, escndete aqu en mi alcoba DON LVARO.(Resuelto.) No, yo no me escondo No te abandono en tal conflicto. (Prepara una pistola.) Defenderte y salvarte es mi obligacin. DOA LEONOR.(Asustadsima.) Qu intentas? Ay! Retira esa pistola que me hiela la sangre Por Dios, sultala! La disparars contra mi buen padre? Contra alguno de mis hermanos? Para matar a alguno de los fieles y antiguos criados de esta casa? DON LVARO.(Profundamente confundido.) No, no, amor mo La emplear en dar fin a mi desventurada vida. DOA LEONOR. Qu horror! Don lvaro!
MATERIAL FOTOCOPIABLE / Oxford University Press Espaa, S. A.

Duque DE RIVAS Don lvaro o la fuerza del sino, Ctedra

1 Explica por qu podemos decir que estamos en presencia de un texto literario y adscrbelo al gnero al que pertenece. 2 Cmo reacciona don lvaro al ver abatida a doa Leonor? Qu responde ella? Reconoce en sus palabras caractersticas tpicas de la mujer en el teatro romntico.

3 Qu peculiaridad presenta el texto principal? Reconoce en l enunciados caractersticos del drama romntico y algunos recursos expresivos. 4 A qu elementos de la representacin hacen referencia las acotaciones? 5 Realiza un resumen del fragmento.

340 19.

La literatura del romanticismo

ACTIVIDADES

AMPLIACIN

19

3. El seor de Bembibre, de Gil y Carrasco


Enrique Gil y Carrasco (1815, Villafranca del Bierzo provincia de Len; 1846, Berln) realiz estudios en Ponferrada con los monjes agustinos, en el monasterio benedictino de San Andrs, de Vega, y en el seminario de Astorga. En 1832 se instal en Valladolid y en 1836 se traslad a Madrid, donde entr en contacto con literatos de la poca, como Espronceda, Larra, Zorrilla y Rivas. En 1844 public su novela El seor de Bembibre y viaj a Berln como secretario de Legacin, donde falleci dos aos despus. Gil y Carrasco fue autor tambin de poesa, artculos periodsticos y relatos de viajes. El seor de Bembibre narra los trgicos amores de don lvaro y doa Beatriz vinculados a la desaparicin de la Orden del Temple.

En edad de casarse
Don Alonso Ossorio, seor de Arganza, haba tenido dos hijos y una hija; pero de los primeros muri uno antes de salir de la infancia, y el otro muri peleando como bueno en su primera campaa contra los moros de Andaluca. As, pues, todas sus esperanzas haban venido a cifrarse en su hija doa Beatriz, que entonces tena pocos aos, pero que ya prometa tanta belleza como talento y generosa ndole. Haba en su carcter una mezcla de la energa que distingua a su padre y de la dulzura y melancola de doa Blanca de Balboa, su madre, santa seora cuya vida haba sido un vivo y constante ejemplo de bondad, de resignacin y de piedad cristiana. Aunque con la prdida temprana de sus dos hijos su complexin, harto delicada por desgracia, se haba arruinado enteramente, no fue esto obstculo para que en la crianza esmerada de su hija emplease una instruccin poco comn en aquella poca, y fecundase las felices disposiciones de que la haba dotado prdigamente la naturaleza. Sin ms esperanza que aquella criatura tan querida y hermosa, sobre ella amontonaba su ternura, todas las ilusiones del deseo y los sueos del porvenir. As creca doa Beatriz como una azucena gentil y fragante al calor del cario maternal, defendida por el nombre y poder de su padre y cercada por todas partes del respeto y amor de sus vasallos, que contemplaban en ella una medianera segura para aliviar sus males y una constante dispensadora de beneficios.
MATERIAL FOTOCOPIABLE / Oxford University Press Espaa, S. A.

Los aos pasaban en tanto rpidos como suelen, y con ellos vol la infancia de aquella joven tan noble, agraciada y rica, a quien por lo mismo pens buscar su padre un esposo digno de su clase y elevadas prendas. En el Bierzo no haba entonces ms de dos casas cuyos estados y vasallos estuviesen al nivel: una la de Arganza, otra la de la antigua familia de los Yez, cuyos dominios comprendan la frtil ribera de Bembibre y la mayor parte de las montaas comarcanas. Este linaje haba dado dos maestres al orden del Temple y era muy honrado y acatado en el pas. Por una rara coincidencia a la manera que el apellido Ossorio penda de la frgil existencia de una mujer, el de Yez estaba vinculado en la de un solo hombre no menos frgil y deleznable en aquellos tiempos de desdicha y turbulencias. Don lvaro Yez y su to don Rodrigo1, maestre del Temple en Castilla, eran los dos nicos miembros que quedaban de aquella raza ilustre y numerosa; rama seca y estril el uno, por su edad y sus votos, y vstago el otro, lleno de savia y lozana, que prometa larga vida y sonados frutos.
Enrique GIL Y CARRASCO El seor de Bembibre Ctedra
1Don

Rodrigo: personaje histrico real, ltimo maestre del Temple en Castilla y Len.

1 Justifica por qu el fragmento corresponde a un texto literario puede adscribirse a la narrativa. 2 Analiza en el texto los elementos correspondientes al nivel de la historia. Personajes Accin Tiempo Espacio

3 Atendiendo al tiempo de la historia y al tiempo del discurso, qu movimiento narrativo se observa? 4 Caracteriza al narrador y seala los tipos de discurso empleados. 5 Comenta la adjetivacin y los recursos expresivos en el fragmento.

19.

La literatura del romanticismo

341

ACTIVIDADES

REFUERZO

19

4. Artculos, de Larra
En su empeo por la reforma cultural y social del pas, Larra critic actitudes y comportamientos de sus contemporneos y, a la vez, se opuso a su tendencia a la visin negativa y denigrante de Espaa. En este artculo reflexiona sobre la frase hecha que le da ttulo y que la mayora repite, y exhorta a sus connacionales a cambiar ciertos hbitos y comprometerse con el bien comn.

En este pas
En este pas Esta es la frase que todos repetimos a porfa, frase que sirve de clave para toda clase de explicaciones, cualquiera que sea la cosa que a nuestros ojos choque en mal sentido. Qu quiere usted? decimos, en este pas!. Cualquier acontecimiento desagradable que nos suceda, creemos explicarle perfectamente con la frasecilla: Cosas de este pas!, que con vanidad pronunciamos y sin pudor alguno repetimos. Nace esta frase de un atraso reconocido en toda la nacin? No creo que pueda ser este su origen, porque solo puede conocer la carencia de una cosa el que la misma cosa conoce: de donde se infiere que si todos los individuos de un pueblo conociesen su atraso, no estaran realmente atrasados. Es la pereza de imaginacin o de raciocinio, que nos impide investigar la verdadera razn de cuanto nos sucede, y que se goza en tener una muletilla siempre a mano con que responderse a sus propios argumentos, hacindose cada uno la ilusin de no creerse cmplice de un mal, cuya responsabilidad descarga sobre el estado del pas en general? Esto parecera ms ingenioso que cierto. Creo entrever la causa verdadera de esta humillante expresin. Cuando se halla un pas en aquel crtico momento en que se acerca a una transicin, y en que, saliendo de las tinieblas, comienza a brillar a sus ojos un ligero resplandor, no conoce todava el bien, empero ya conoce el mal, de donde pretende salir para probar cualquiera otra cosa que no sea lo que hasta entonces ha tenido. Sucdele lo que a una joven bella que sale de la adolescencia; no conoce el amor todava ni sus goces; su corazn, sin embargo, o la naturaleza, por mejor decir, le empieza a revelar una necesidad que pronto ser urgente para ella, y cuyo germen y cuyos medios de satisfaccin tiene en s misma, si bien los desconoce todava; la vaga inquietud de su alma, que busca y ansa, sin saber qu, la atormenta y la disgusta de su estado actual y del anterior en que viva; y vsela despreciar y romper aquellos mismos sencillos juguetes que formaban poco antes el encanto de su ignorante existencia. Este es acaso nuestro estado, y este, a nuestro entender, el origen de la fatuidad que en nuestra juventud se observa: el medio saber reina entre nosotros; no conocemos el bien, pero sabemos que existe y que podemos llegar a poseerle, si bien sin imaginar an el cmo. Afectamos, pues, hacer ascos de lo que tenemos para dar a entender a los que nos oyeron que conocemos cosas mejores, y nos queremos engaar miserablemente unos a otros, estando todos en el mismo caso. Este medio saber nos impide gozar de lo bueno que realmente tenemos, y aun nuestra ansia de obtenerlo todo de una vez nos ciega sobre los mismos progresos que vamos insensiblemente haciendo. [] Olvidemos, lo repetimos, esa funesta expresin que contribuye a aumentar la injusta desconfianza que de nuestras propias fuerzas tenemos. Hagamos ms favor o justicia a nuestro pas, y cremosle capaz de esfuerzos y felicidades. Cumpla cada espaol con sus deberes de buen patricio, y en vez de alimentar nuestra inaccin con la expresin de desaliento: Cosas de Espaa! contribuya cada cual a las mejoras posibles. Entonces este pas dejar de ser tan mal tratado de los extranjeros, a cuyo desprecio nada podemos oponer, si de l les damos nosotros mismos el vergonzoso ejemplo.
MATERIAL FOTOCOPIABLE / Oxford University Press Espaa, S. A.

Mariano Jos DE LARRA Artculos, Castalia

1 Qu preguntas se hace Larra sobre el origen de la frase? 2 Qu respuesta aventura? Con qu comparacin la desarrolla? 3 En qu estado considera que se encuentra el pas en su poca?

4 Qu consejos da Mariano Jos de Larra a modo de conclusin? 5 Realiza un resumen del texto. 6 Reconoce las modalidades textuales presentes en este fragmento de En este pas.

342 19.

La literatura del romanticismo

ACTIVIDADES

AMPLIACIN

19

5. El reo de muerte, de Espronceda


En sus canciones Espronceda cuestiona y denuncia realidades y actitudes de su poca: la hipocresa, el egosmo, la indiferencia frente al dolor, la impiedad. Estas crticas estn presentes, sobre todo, en El reo de muerte, El verdugo y El mendigo. En la cancin que presentamos aqu, el joven reo, en la noche, desesperado, vive sus ltimas horas; desde la celda oye el bullicio de la calle y la frase que se cita; luego reina el silencio: todos duermen, indiferentes ante su muerte.

El reo de muerte
[] Loca y confusa la encendida mente, sueos de angustia y fiebre y devaneo, el alma envuelven del confuso reo, que inclina al pecho la abatida frente. Y en sueos confunde la muerte, la vida. Recuerda y olvida, suspira, respira con hrrido afn. Y en un mundo de tinieblas vaga y siente miedo y fro, y en su horrible desvaro palpa en su cuello el dogal1; y cuanto ms forcejea, cuanto ms lucha y porfa, tanto ms en su agona aprieta el nudo fatal. Y oye ruidos, voces, gentes, y aquella voz que dir: Para hacer bien por el alma del que van a ajusticiar!2
MATERIAL FOTOCOPIABLE / Oxford University Press Espaa, S. A.

O ya libre se contempla, y el aire puro respira, y oye de amor que suspira la mujer que a un tiempo am, bella y dulce cual sola, tierna flor de primavera, el amor de la pradera que el abril galn mim. Y gozoso a verla vuela, y alcanzarla intenta en vano, que al tender la ansiosa mano su esperanza a realizar, su ilusin la desvanece de repente el sueo impo, y halla un cuerpo mudo y fro y un cadalso en su lugar. Y oye a su lado en son triste lgubre voz resonar: Para hacer bien por el alma del que van a ajusticiar!

30

10

35

40

15

20

45

Jos DE ESPRONCEDA Poesas lricas y fragmentos picos, Castalia


1 2

dogal: cuerda para ahorcar a un reo. Para hacer [] ajusticiar!: frase que decan los cofrades de Paz y Caridad, que asistan a los condenados, cuando pedan limosnas para el reo.

25

1 Qu situacin se describe hasta el verso 25? Y del verso 26 hasta el final? 2 Explica el contraste que se establece entre las dos partes.

3 En qu se insiste al final del texto? 4 Analiza la mtrica y los principales recursos estilsticos empleados por Espronceda en estos versos.

19.

La literatura del romanticismo

343

ACTIVIDADES

REFUERZO

19

6. Rimas, de Bcquer
En las Rimas encontramos textos que definen la interioridad, la conciencia de la voz potica y tambin otros que definen el yo y el t femenino, claves en la obra del autor, y las relaciones imposibles entre ambos.

Rima 15 (II)
Saeta que voladora cruza, arrojada al azar, y que no se sabe dnde temblando se clavar; hoja que del rbol seca arrebata el vendaval, y que no hay quien diga el surco donde el polvo volver. Gigante ola que el viento riza y empuja en el mar y rueda y pasa y se ignora qu playa buscando va. Luz que en cercos temblorosos brilla prxima a expirar, y que no se sabe de ellos cul el ltimo ser. Eso soy yo que al acaso cruzo el mundo sin pensar de dnde vengo ni a dnde mis pasos me llevarn. 5

Rima 60 (XV)
Cendal flotante de leve bruma, rizada cinta de blanca espuma, rumor sonoro de arpa de oro, beso del aura, onda de luz, eso eres t. T, sombra area, que cuantas veces voy a tocarte te desvaneces. Como la llama, como el sonido, como la niebla, como el gemido del lago azul! En mar sin playas onda sonante, en el vaco cometa errante, largo lamento del ronco viento, ansia perpetua de algo mejor, eso soy yo. Yo, que a tus ojos en mi agona los ojos vuelvo de noche y da; yo, que incansable corro y demente tras una sombra, tras la hija ardiente de una visin!

10

10

15

15

20

20

Gustavo Adolfo BCQUER Libro de los gorriones, Planeta

Gustavo Adolfo BCQUER Libro de los gorriones, Planeta

1 Identifica los elementos que se nombran en las cuatro primeras estrofas de la Rima 15 y menciona: Qu movimientos realizan. Quin o qu los causa. Cules son sus destinos. 2 La ltima estrofa aclara que los elementos que has nombrado en la actividad 1 se identifican con el yo potico, qu similitudes se establecen? 3 Determina el tema del poema y analiza los recursos expresivos que lo estructuran. 4 Qu elementos se identifican con el t en la Rima 60? A qu mbito pertenecen?

5 Consigue comunicarse el yo potico con la destinataria? 6 En la tercera estrofa se describe el yo potico, qu elementos aparecen y qu rasgos suyos transmiten? 7 Explica el contenido de la ltima estrofa; qu persigue la voz potica? 8 Reconoce los recursos expresivos principales con que se desarrolla el contenido. 9 Realiza el anlisis mtrico de ambas composiciones. 10 Relaciona los temas de estos textos con los de la obra potica del autor.

344 19.

La literatura del romanticismo

MATERIAL FOTOCOPIABLE / Oxford University Press Espaa, S. A.

ACTIVIDADES

AMPLIACIN

19

7. Las Leyendas, de Bcquer


Las Leyendas contienen narraciones muy diversas de las que se han destacado sus valores estilsticos (se ha hablado de poemas en prosa) en relacin con la prosa de la poca, especialmente sus descripciones pictricas y el hincapi que Bcquer pone en la expresin de las sensaciones. En El rayo de luna, Manrique, un noble imaginativo y poeta, ama la soledad y suea con el amor; una noche de luna se interna en las desiertas ruinas de los Templarios, ve agitarse una cosa blanca y cree que es el traje de una mujer misteriosa. Al hallarla a esa hora y en ese lugar, se convierte en la dama de sus sueos e inicia su bsqueda. Dos meses despus de buscarla en vano, se produce el descubrimiento.

El rayo de luna
Yo no s si esto es una historia que parece cuento o un cuento que parece historia; lo que puedo decir es que en su fondo hay una verdad, una verdad muy triste, de la que acaso yo ser uno de los ltimos en aprovecharme, dadas mis condiciones de imaginacin. Otro, con esta idea, tal vez hubiera hecho un tomo de filosofa lacrimosa; yo he escrito esta leyenda, que a los que nada vean en su fondo, al menos podr entretener un rato. [] La noche estaba serena y hermosa; la luna brillaba en toda su plenitud, en lo ms alto del cielo, y el viento suspiraba con un rumor dulcsimo entre las hojas de los rboles. Manrique lleg al claustro, tendi la vista por su recinto, y mir a travs de las macizas columnas de sus arcadas Estaba desierto. Sali de l, encamin sus pasos hacia la oscura alameda que conduce al Duero, y an no haba penetrado en ella cuando de sus labios se escap un grito de jbilo. Haba visto flotar un instante y desaparecer el extremo del traje blanco, del traje blanco de la mujer de sus sueos, de la mujer que ya amaba como un loco. Corre, corre en su busca; llega al sitio en que la ha visto desaparecer; pero al llegar se detiene, fija los espantados ojos en el suelo, permanece un rato inmvil; un ligero temblor nervioso agita sus miembros, un temblor que va creciendo, que va creciendo y ofrece los sntomas de una verdadera convulsin, y prorrumpe al fin en una carcajada sonora, estridente, horrible.
MATERIAL FOTOCOPIABLE / Oxford University Press Espaa, S. A.

Aquella cosa blanca, ligera, flotante, haba vuelto a brillar ante sus ojos; pero haba brillado a sus pies un instante, no ms que un instante. Era un rayo de luna, un rayo de luna que penetraba a intervalos por entre la bveda de los rboles cuando el viento mova las ramas. *** Queris que os diga una cantiga, la ltima que ha compuesto mosn 1 Arnaldo, el trovador provenzal? No! No! exclam el joven, incorporndose colrico en su sitial. No, no quiero nada es decir, s quiero quiero que me dejis solo Cantigas, mujeres, gloria, felicidad; mentira todo, fantasmas vanos que formamos en nuestra imaginacin y vestimos a nuestro antojo, y los amamos y corremos tras ellos, para qu?, para qu? Para encontrar un rayo de luna. Manrique estaba loco; por lo menos, todo el mundo lo crea as. A m, por el contrario, se me figuraba que lo que haba hecho era recuperar el juicio.
Gustavo Adolfo BCQUER Rimas y leyendas Espasa Calpe
1

mosn: ttulo que se daba a los clrigos y a los nobles de segunda clase en el antiguo reino de Aragn.

1 Justifica la adscripcin del texto al gnero narrativo. 2 Caracteriza al narrador y explica el contenido de sus intervenciones inicial y final. 3 Resume el contenido del segundo fragmento. 4 Explica los elementos caractersticos de la narracin presentes en l. Personajes Accin Tiempo Espacio

5 Justifica el cambio en el tiempo verbal que se aprecia en el relato. 6 Analiza los recursos expresivos y la adjetivacin empleada en este texto. 7 Reconoce en ambos fragmentos el empleo de los discursos referencial, descriptivo, dramtico, valorativo y universal. 8 Qu caractersticas del romanticismo observas en el texto?

19.

La literatura del romanticismo

345

Evaluacin 19

Noche y sueo, por Evelyn de Morgan.

Rima 33 (XXIV)
Dos rojas lenguas de fuego que a un mismo tronco enlazadas se aproximan, y al besarse forman una sola llama. Dos notas que del lad a un tiempo la mano arranca, y en el espacio se encuentran y armoniosas se abrazan. Dos olas que vienen juntas a morir sobre una playa y que al romper se coronan con un penacho de plata. Dos jirones1 de vapor que del lago se levantan, y al reunirse en el cielo 15 forman una nube blanca. Dos ideas que al par brotan, dos besos que a un tiempo estallan, dos ecos que se confunden, eso son nuestras dos almas. 20
Gustavo Adolfo BCQUER Rimas, Castalia
1

10

jirn: parte o porcin pequea de un todo.

1. Enumera los elementos que se mencionan en las cuatro


primeras estrofas del poema. Qu les ocurre? En qu se transforman?

6. En qu etapa del romanticismo se incluyen los poemas


de Bcquer? En qu se diferencia esta poesa de la de la primera mitad del siglo? Cita el poeta ms importante de este ltimo perodo y sus principales obras.

2. En qu se distingue de las anteriores la estructura de la


ltima estrofa? Indica qu informacin aporta acerca del emisor y del contenido del texto.

7. Explica los gneros de la prosa romntica y sita la obra


de Larra en ese contexto. Enuncia las caractersticas fundamentales de sus artculos.

3. Enuncia el tema que trata Gustavo Adolfo Bcquer en


esta composicin y relacinalo con otros motivos caractersticos de su obra lrica.

8. Menciona los rasgos principales del drama romntico


y ejemplifcalos con Don lvaro o la fuerza del sino.

4. Qu recurso estilstico propio de las Rimas aparece en


la nmero 33? Seala los principales recursos expresivos y realiza su anlisis mtrico.

9. Qu innovaciones introduce Don Juan Tenorio? Comenta


los aspectos ms relevantes de esta pieza.

5. Menciona otros temas que Bcquer haya tratado con


frecuencia en las Rimas y explica su vinculacin con el romanticismo.

10. Redacta un texto sobre la pervivencia actual del romanticismo: ten en cuenta rasgos generales, temas y motivos tratados, esttica, huella en distintos medios de expresin (literatura, cine, series televisivas ). 329 19.
La literatura del romanticismo

ACTIVIDADES

REFUERZO

20

1. Un fragmento de la primera parte de Fortunata y Jacinta


Fortunata y Jacinta es una de las obras ms importante de la narrativa realista. En este fragmento, que pertenece a la primera parte de la novela, Juan Santa Cruz, que est borracho, habla con su mujer sobre Fortunata, durante el viaje de novios. Si la hubieras visto! Fortunata tena los ojos como dos estrellas, muy semejantes a los de la Virgen del Carmen que antes estaba en Santo Toms, y ahora en San Gins. Pregntaselo a Estupi; pregntaselo si lo dudas, a ver Fortunata tena las manos bastas de tanto trabajar; el corazn lleno de inocencia Fortunata no tena educacin; aquella boca tan linda se coma muchas letras y otras las equivocaba. Deca indilugencias, golver, asn. Pas su niez cuidando el ganado. Sabes lo que es el ganado? Las gallinas. Despus criaba los palomos a sus pechos. Como los palomos no comen sino del pico de la madre, Fortunata se los meta en el seno. Y si vieras t qu seno tan bonito! Solo que tena muchos rasguos que le hacan los palomos con los garfios de sus patas. Despus coga en la boca un buche de agua y algunos granos de algarroba, y metindose el pico en la boca les daba de comer Era la paloma madre de los tiernos pichoncitos Luego les daba su calor natural, los arrullaba, les haca rorroo, les cantaba canciones de nodriza Pobre Fortunata, pobre Pitusa! Te he dicho que la llamaban la Pitusa? No? Pues te lo digo ahora. Que conste Yo la perd, s, que conste tambin; es preciso que cada cual cargue con su responsabilidad Yo la perd; la enga, le dije mil mentiras, le hice creer que me iba a casar con ella. Has visto? Si ser pilln! Djame que me ra un poco S, todas las papas1 que yo le deca se las tragaba El pueblo es muy inocente, es tonto de remate; todo se lo cree con tal que se lo digan con palabras finas La enga, le garfi2 su honor, y tan tranquilo. Los hombres, digo, los seoritos, somos unos miserables; creemos que el honor de las hijas del pueblo es cosa de juego No pongas esa cara, vida ma. Comprendo que tienes razn; soy un infame, merezco tu desprecio. Porque lo que t dirs: una mujer es siempre una criatura de Dios, verdad? Y yo, despus que me divert con ella, la dej abandonada en medio de las calles; justo, su destino es el destino de las perras Di que s. [] Me idolatraba. Crea que yo no era como los dems, que era la caballerosidad, la hidalgua, la decencia, la nobleza en persona, el acabose de los hombres Nobleza! Qu sarcasmo! Nobleza en la mentira; digo que no puede ser, y que no, y que no Qu humanidad tan farsante! El pobre siempre debajo; el rico hace lo que le da la gana. Yo soy rico Di que soy inconstante La ilusin de lo pintoresco se iba pasando. La grosera con gracia seduce algn tiempo, despus marea Cada da me pesaba ms la carga que me haba echado encima. El picor del ajo me repugnaba. Dese, puedes creerlo, que la Pitusa fuera mala para darle una puntera Pero, qui3; ni por esas Mala ella? A buena parte Si le mando echarse al fuego por m, al fuego de cabeza! [] El hasto era ya irresistible. La misma Pitusa me era odiosa, como las palabras inmundas Un da dije vuelvo, y no volv ms Lo que deca Villalonga: cortar por lo sano Yo tena algo en mi conciencia, un hilito que me tiraba hacia all Lo cort Fortunata me persigui: tuve que jugar al escondite. Ella por aqu, yo por all Yo me escurra como una anguila. No me coga, no. El ltimo a quien vi fue a Izquierdo; le encontr un da subiendo la escalera de mi casa. Me amenaz; djome que la Pitusa estaba cambr4 de cinco meses Cambr de cinco meses! Alc los hombros Dos palabras l, dos palabras yo; alargu este brazo, y plaf Izquierdo baj de golpe un tramo entero Otro estirn, y plaf, de un brinco el segundo tramo y con la cabeza para abajo
Benito PREZ GALDS Fortunata y Jacinta, Hernando
1 2 3

papa: aqu mentira. garfiar: (en germana) hurtar, robar. qui: (en germana) interjeccin utilizada para expresar desconocimiento o sorpresa. cambr: embarazada.

MATERIAL FOTOCOPIABLE / Oxford University Press Espaa, S. A.

1 Qu caractersticas del personaje de Fortunata destaca Juan Santa Cruz en este texto? Con qu la identifica? 2 Explica qu juicios formula acerca de su relacin con ella. Por qu termina? De qu se entera al final? 3 Qu visin tiene el seorito del pueblo? Por qu afirma que les atrae a los burgueses como l?

4 Seala la tcnica narrativa utilizada por Galds en este fragmento. 5 Comenta el lenguaje empleado por el personaje en relacin con su situacin y condicin social. 6 Relaciona el comentario anterior con las caractersticas de la novela realista en general y de la obra de Galds en particular.

20.

La narrativa realista

359

ACTIVIDADES

REFUERZO

20

2. Misericordia, de Benito Prez Galds


Los ambientes retratados por la novela realista abarcan toda la diversidad social de la poca: desde las clases ms acomodadas hasta las ms humildes, e incluso los ms desamparados. En este pasaje, Benina, protagonista de Misericordia, luego de mostrar su gran piedad alimentando a un nutrido grupo de indigentes, encuentra a un mendigo anciano quien le da noticias sobre su amigo Almudena. Djole despus el pobre viejo que se mora de hambre; que no haba entrado en su boca, en tres das, ms que un pedazo de bacalao crudo que le dieron en una tienda, y algunos corruscos1 de pan, que mojaba en la fuente para reblandecerlos, porque ya no tena hueso en la boca. Desde el da de San Jos que quitaron la sopa en el Sagrado Corazn, no haba ya remedio para l; en parte alguna encontraba amparo; el cielo no le quera, la tierra tampoco. Con ochenta y dos aos cumplidos el 3 de febrero, San Blas bendito, un da despus de la Candelaria, para qu quera vivir ms ni qu se le haba perdido por ac? Un hombre que sirvi al Rey doce aos; que durante cuarenta y cinco haba picado miles de miles de toneladas de piedra en esas carreteras de Dios, y que siempre fue bien mirado y puntoso, nada tena que hacer ya, ms que encomendarse al sepulturero para que le pusiera mucha tierra, mucha tierra encima, y apisonara bien. En cuanto que colocara a las dos criaturas, se acostara para no levantarse hasta el da del Juicio por la tarde y se levantara el ltimo! Traspasada de pena Benina al or la referencia de tanto infortunio, cuya sinceridad no poda poner en duda, dijo al anciano que la llevara a donde estaba la nia enferma, y pronto fue conducida a un cuarto lbrego, en la planta baja de la casa grande de corredor, donde juntos vivan, por el pago de tres pesetas al mes, media docena de pordioseros con sus respectivas proles. La mayor parte de estos hallbanse a la sazn en Madrid, buscando la santa perra2. Solo vio Benina una vieja petiseca3 y dormilona, que pareca alcoholizada, y una mujer panzuda, tumefacta, de piel vinosa y tirante, como la de un corambre4 repleto, con la cara erisipelada5, mal envuelta en trapos de distintos colores. En el suelo, sobre un colchn flaco, cubierto de pedazos de bayeta amarilla y de jirones de mantas morellanas, yaca la nia enferma, como de seis aos, el rostro lvido, los puos cerrados en la boca. Lo que tiene esta criatura es hambre dijo Benina, que habindola tocado en la frente y manos, la encontr fra como el mrmol. Puede que as sea, porque cosa caliente no ha entrado en nuestros cuerpos desde ayer. No necesit ms la bondadosa anciana para que se le desbordase la piedad, que caudalosa inundaba su alma; y llevando a la realidad sus intenciones con la presteza que era en ella caracterstica, fue al instante a la tienda de comestibles, que en el ngulo de aquel edificio existe, y compr lo necesario para poner un puchero inmediatamente, tomando adems huevos, carbn, bacalao pues ella no haca nunca las cosas a medias. A la hora, ya estaban remediados aquellos infelices, y otros que se agregaron, inducidos por el olor que por toda la parte baja de la colmena prontamente se difundi. Y el Seor hubo de recompensar su caridad, deparndole, entre los mendigos que al festn acudieron, un lisiado sin piernas que andaba con los brazos, el cual le dio por fin noticias verdicas del extraviado Almudena.
MATERIAL FOTOCOPIABLE / Oxford University Press Espaa, S. A.

Benito PREZ GALDS Misericordia , Ctedra

corrusco: parte del pan ms tostada que corresponde a los extremos o al borde. 2santa perra: dinero. 3petiseca: raqutica, rugosa.
1

corambre: conjunto de cueros o pellejos. erisipelado: que padece erisipela, infeccin microbiana de la piel.

1 En el texto se retrata el mundo de los mendigos; seala qu casos presenta y las caractersticas del espacio en el que se desarrolla la escena. 2 Cmo reacciona Benina? Qu consigue la anciana con su actitud?

3 Comenta las tcnicas narrativas utilizadas en este texto. 4 Indica las caractersticas de la novela realista que aparecen en este pasaje de Misericordia, de Benito Prez Galds, correspondiente al ciclo de novelas espiritualistas del autor.

360 20.

La narrativa realista

ACTIVIDADES

AMPLIACIN

20

3. El sombrero de tres picos, de Pedro Antonio de Alarcn


Pedro Antonio de Alarcn (1833-1891) escribi cuentos, novelas y artculos de crtica literaria. Su primera novela, El final de Norma (1850), est dentro de la tendencia romntica. Toda su obra posterior se caracteriza por la combinacin de rasgos del romanticismo y del realismo. Entre sus ms importantes creaciones pueden citarse El escndalo (1875) y El nio de la bola (1880). Se trata de novelas de tesis que evidencian un gran conservadurismo poltico y literario; la preocupacin religiosa y por el orden social establecido son dos constantes de su literatura. En La moral en el arte (1877), discurso que pronunci a propsito de su ingreso en la Real Academia Espaola, expuso sus ideas acerca de la finalidad didctica y el sentido moral y religioso que, a su juicio, deba tener la novela. Su obra ms importante es El sombrero de tres picos (1874), una narracin breve de enredo, de carcter cmico, que transcurre a principios del siglo XIX. El relato contiene animados cuadros de costumbres y desarrolla la historia, sacada de un romance tradicional, de un corregidor que intenta conseguir los favores de una molinera casada.

El molino
Por varias y diversas razones, haca ya algn tiempo que aquel molino era el predilecto punto de llegada y descanso de los paseantes ms caracterizados de la mencionada ciudad Primeramente, conduca a l un camino carretero, menos intransitable que los restantes de aquellos contornos. En segundo lugar, delante del molino, haba una plazoletilla, empedrada, cubierta por un parral enorme, debajo del cual se tomaba muy bien el fresco en verano y el sol en invierno, merced a la alternada ida y venida de los pmpanos1 En tercer lugar, el molinero era un hombre muy respetuoso, muy discreto, muy fino, que tena lo que se llama don de gentes, y que obsequiaba a los seores que solan honrarlo con su tertulia vespertina ofrecindoles lo que daba el tiempo, ora habas verdes, ora cerezas y guindas, ora lechugas en rama y sin sazonar (que estn muy buenas cuando se las acompaa de macarros2 de pan y aceite; macarros que se encargaban de enviar por delante sus seoras), ora melones, ora uvas de aquella misma parra que les serva de dosel3, ora rosetas de maz si era invierno, y castaas asadas, y almendras y nueces, y de vez en cuando, en las tardes muy fras, un trago de vino de pulso (dentro ya de la casa y al amor de la lumbre), a lo que por Pascuas se sola aadir algn pestio, algn mantecado, algn rosco o alguna lonja de jamn alpujarreo. Tan rico era el molinero, o tan imprudentes sus tertulianos exclamaris interrumpindome. Ni lo uno ni lo otro. El molinero solo tena un pasar, y aquellos caballeros eran la delicadeza y el orgullo personificados. Pero en unos tiempos en que se pagaban cincuenta y tantas contribuciones a la Iglesia y el Estado, poco arriesgaba un rstico de tan claras luces como aquel de tenerse ganada la voluntad de regidores, cannigos, frailes, escribanos y dems personas de campanillas. As es que no faltaba quien dijese que el to Lucas (tal era el nombre del molinero) se ahorraba un dineral al ao a fuerza de agasajar a todo el mundo. Vuestra merced me va a dar una puertecilla vieja de la casa que ha derribado decale a uno. Vuestra seora decale a otro va a mandar que me rebajen el subsidio, o la alcabala4, o la contribucin de frutos civiles. Vuestra reverencia me va a dejar coger en la huerta del convento una poca hoja para mis gusanos de seda. Vuestra ilustrsima me va a dar permiso para traer una poca lea del monte X.
Pedro Antonio DE ALARCN El sombrero de tres picos, Edelvives
1 2 3

MATERIAL FOTOCOPIABLE / Oxford University Press Espaa, S. A.

pmpano: nombre que reciben las hojas de la parra. macarro: panecillo de forma alargada y una libra de peso. dosel: mueble que a cierta altura cubre o resguarda un altar, sitial, lecho, etc., adelantndose en pabelln horizontal y cayendo por detrs a modo de colgadura. alcabala: tributo del tanto por ciento del precio que pagaba al fisco el vendedor en el contrato de compraventa y ambos contratantes en el de permuta.

1 La accin de El sombrero de tres picos se desarrolla en Andaluca; comenta algunos rasgos costumbristas. 2 Analiza el narrador, las perspectivas y las tcnicas narrativas utilizadas. 3 Indica las intervenciones del narrador y las apelaciones al receptor que observes.

4 Seala qu rasgos de la narrativa realista estn presentes en este pasaje de El sombrero de tres picos. 5 Cules son las caractersticas del protagonista del relato de Pedro Antonio de Alarcn? 6 Seala qu justificacin de su conducta ofrece el narrador.

20.

La narrativa realista

361

ACTIVIDADES

REFUERZO

20

4. La procesin de Viernes Santo en Vetusta


En el siguiente pasaje de La Regenta abundan las imgenes que representan percepciones sensoriales. Ana ha decidido participar en la procesin del Viernes Santo, descalza, vestida de Nazareno, como prueba de adhesin y fidelidad espiritual al Magistral, pero cuando llega el momento se arrepiente. Los ciudadanos de Vetusta no se pierden el espectculo del cortejo, admirados y escandalizados.

El va crucis de Ana Ozores


Los tambores vibraban fnebres, tristes, empeados en resucitar un dolor muerto haca diecinueve siglos; a don Vctor s le sonaba aquello a himno de muerte; se le figuraba ya que llevaban a su mujer al patbulo1. El redoble del parche se destacaba en un silencio igual y montono. En la calle estrecha, de casas oscuras, se anticipaba el crepsculo; las largas filas de hachas encendidas se perdan a lo lejos, hacia arriba, mostrando la luz amarillenta de los pabilos2, como un rosario de cuentas doradas, roto a trechos. En los cristales de las tiendas cerradas y de algunos balcones se reflejaban las llamas movibles; suban y bajaban en contorsiones fantsticas, como sombras lucientes, en confusin de aquelarre. Aquella multitud silenciosa, aquellos pasos sin ruido, aquellos rostros sin expresin de los colegiales de blancas albas que alumbraban con cera la calle triste daban al conjunto apariencia de ensueo. No parecan seres vivos aquellos seminaristas cubiertos de blanco y negro, plidos unos, con cercos morados en los ojos, otros morenos, casi negros, de pelo en matorral, casi todos cejijuntos, preocupados con la idea fija del aburrimiento, mquinas de hacer religin, reclutas de una leva3 forzosa del hambre y de la holgazanera. Iban a enterrar a Cristo, como a cualquier cristiano, sin pensar en l; a cumplir con el oficio. [] Tambin Ana pareca de madera pintada; su palidez era como un barniz. Sus ojos no vean. A cada paso crea caer sin sentido. Senta en los pies, que pisaban las piedras y el lodo, un calor doloroso; cuidaba de que no asomasen debajo de la tnica morada; pero a veces se vean. Aquellos pies desnudos eran para ella la desnudez de todo el cuerpo y de toda el alma. Ella era una loca que haba cado en una especie de prostitucin singular!; no saba por qu, pero pensaba que despus de aquel paseo a la vergenza ya no haba honor en su casa. All iba la tonta, la literata, Jorge Sandio4 la mstica, la fatua5, la loca, la loca sin vergenza. Ni un solo pensamiento de piedad vino en su ayuda en todo el camino. El pensamiento no le daba ms que vinagre en aquel calvario de su recato. Hasta recordaba textos de fray Luis de Len en La perfecta casada, que, segn ella, condenaban lo que estaba haciendo. Me ceg la vanidad, no la piedad, pensaba. Yo tambin soy cmica, soy lo que mi marido. Si alguna vez se atreva a mirar hacia atrs, a la Virgen, senta hielo en el alma. La Madre de Jess no la miraba, no haca caso de ella; pensaba en su dolor cierto; ella, Mara, iba all porque delante llevaba a su Hijo muerto, pero Ana, a qu iba?
MATERIAL FOTOCOPIABLE / Oxford University Press Espaa, S. A.

Leopoldo ALAS, CLARN La Regenta, Castalia

patbulo: tablado o lugar en que se ejecuta la pena de muerte. pabilo: mecha que esta en el centro de la vela. leva: partida de las embarcaciones. Recluta de gente.

Jorge Sandio: castellanizacin de George Sand, seudnimo de Amandine Aurore Lucille Dupin, escritora francesa. fatuo: falto de razn o entendimiento. Lleno de presuncin o vanidad infundada o ridcula.

2 3

1 Explica brevemente el argumento de este pasaje de La Regenta. 2 Relaciona los pensamientos y sentimientos de Ana Ozores con lo que conoces de la obra. 3 Analiza el narrador y su perspectiva en este texto del realismo. 4 Reconoce los casos de discurso valorativo del narrador.

5 Qu tcnicas narrativas se usan para expresar la interioridad del personaje? 6 El texto comienza con la descripcin de la procesin; comenta las imgenes que refieren percepciones acsticas y visuales. 7 Qu alusiones literarias se incorporan en el fragmento? 8 Analiza los recursos expresivos del texto.

362 20.

La narrativa realista

Evaluacin 20

Fotograma de La Regenta, serie televisiva dirigida por Fernndo Mndez-Leite. En la imagen los actores Carmelo Gmez como Fermn de Pas y Aitana Snchez-Gijn como Ana Ozores.

La cada de Ana Ozores


Servanda iba y vena como una estatua en movimiento y los dems vetustenses no entraban en el casern de los Ozores despus de la muerte de don Vctor. No entraban. Vetusta la noble estaba escandalizada, horrorizada. Unos a otros, con cara de hipcrita compuncin, se ocultaban los buenos vetustenses el ntimo placer que les causaba aquel gran escndalo que era como una novela, algo que interrumpa la monotona eterna de la ciudad triste. Pero ostensiblemente pocos se alegraban de lo ocurrido. Era un escndalo! Un adulterio descubierto! Un duelo! Un marido, un ex regente de Audiencia, muerto de un pistoletazo en la vejiga! En Vetusta, ni aun en los das de la revolucin haba habido tiros. No haba costado a nadie un cartucho la conquista de los derechos inalienables del hombre. Aquel tiro de Mesa, del que tena la culpa la Regenta, rompa la tradicin pacfica del crimen silencioso, morigerado y precavido. Ya se saba que muchas damas principales de la Encimada y de la Colonia engaaban o haban engaado o estaban a punto de engaar a sus respectivos esposos, pero no a tiros! La envidia, que hasta se haba disfrazado de admiracin, sali a la calle con toda la amarillez de sus carnes. Y result que envidiaban en secreto la hermosura y la fama de virtuosa de la Regenta, no solo Visitacin Olas de Cuervo y Obdulia Fandio y la baronesa de la Deuda Flotante, sino tambin la gobernadora, y la de Pez y la seora de Carraspique y la de Rianzares, o sea, el Gran Constantino, y las criadas de la marquesa y toda la aristocracia, y toda la clase media y hasta las mujeres del pueblo y quin lo dijera! la marquesa misma, aquella doa Rufina tan liberal que con tanta magnanimidad se absolva a s misma de las ligerezas de la juventud y otras! Hablaban mal de Ana Ozores todas las mujeres de Vetusta y hasta la envidiaban y despellejaban muchos hombres con alma como la de aquellas mujeres.
Leopoldo ALAS, CLARN La Regenta, Alianza

1. Cmo reaccionan los habitantes de Vetusta ante la


muerte de don Vctor? Cules son sus actitudes y sus sentimientos hacia Ana? Qu crticas se observan?

6. Seala rasgos generales de la novela realista y reconoce


los que estn presentes en el texto.

2. Relaciona el pasaje con el contenido general de la novela. 3. Analiza el narrador, su perspectiva y las tcnicas narrativas empleadas en el texto.

7. Cita las etapas en la novelstica de Galds, explica sus


caractersticas y menciona sus principales obras.

8. Qu conflictos se desarrollan en Fortunata y Jacinta?


Analiza el espacio y el tiempo de la accin y relacinalos con las intenciones y novelas del autor.

4. Seala las caractersticas de la protagonista y sus relaciones con Vetusta. Alude a los otros personajes.

9. Cita otros escritores y obras del realismo. 10. Escribe una descripcin realista de una escena de tu
centro escolar: puede ser la entrada a clase o el recreo. 347 20.
La narrativa realista

5. Explica los aspectos ms relevantes de esta novela:


tema, estructura, tcnicas narrativas.

Potrebbero piacerti anche